Discussion:
Was ist die genaue Ursache dafür, daß manche Gleichungen keine Elementare Funktion als Lösung haben?
(zu alt für eine Antwort)
IV
2017-02-08 20:09:16 UTC
Permalink
Hallo,

was ist die g e n a u e Ursache dafür, daß Gleichungen wie
y + e^y = x
und die Kepler-Gleichung
y - c*sin(y) = x, c eine Konstante,
keine Elementare Funktion y (siehe Wikipedia en: Elementary function) als
Lösung haben können?

Danke.
Torn Rumero DeBrak
2017-02-08 22:42:57 UTC
Permalink
Post by IV
Hallo,
was ist die g e n a u e Ursache dafür, daß Gleichungen wie
y + e^y = x
und die Kepler-Gleichung
y - c*sin(y) = x, c eine Konstante,
keine Elementare Funktion y (siehe Wikipedia en: Elementary function) als
Lösung haben können?
Danke.
Ist die Betragsfunktion

f: IR -> IR, x |-> |x|

eine elementare Funktion?

Wenn ja, dann ist folglich nicht jede elementare Funktion
differenzierbar.

Wenn nein, warum nicht?
Hans Crauel
2017-02-08 23:30:08 UTC
Permalink
Torn Rumero DeBrak schrieb
Post by Torn Rumero DeBrak
Post by IV
was ist die g e n a u e Ursache dafür, daß Gleichungen wie
y + e^y = x
und die Kepler-Gleichung
y - c*sin(y) = x, c eine Konstante,
keine Elementare Funktion y (siehe Wikipedia en: Elementary function) als
Lösung haben können?
Ist die Betragsfunktion
f: IR -> IR, x |-> |x|
eine elementare Funktion?
Fuer reelle x ist |x| = sqrt(x^2), also ist die Betragsfunktion
wohl eine elementare Funktion im Sinne von Wikipedia "Elementary
function".
Post by Torn Rumero DeBrak
Wenn ja, dann ist folglich nicht jede elementare Funktion
differenzierbar.
Wikipedia "Elementary function" meint, dass die Menge der elementaren
Funktionen "closed under differentiation" sei. Das ist offensichtlich
fuer die dort gegebene Charakterisierung nicht der Fall. Dem muesste
man wohl genauer nachgehen, wenn man ernsthaft Aussagen ueber
`elementare Funktionen' machen will.

Wikipedia "Elementare Funktion" meint wohingegen

| Dabei gibt es keine allgemeingültige Definition, wann eine Funktion
| elementar genannt wird und wann nicht. [...]
| Da es sich trotz aller Unklarheit eingebürgert hat, von "elementaren
| Funktionen" zu sprechen, und im Gefolge dessen auch tatsächliche
| mathematische Fragestellungen aufgeworfen wurden, sind wiederholt
| Versuche unternommen worden, exakte Definitionen dieses Begriffs zu
| liefern. [...]
| Der Nachweis, dass bestimmte Funktionen nicht elementar sind, ist
| dagegen erst auf Grundlage einer präzisen Definition möglich.

Eine einfache Moeglichkeit, die Frage nach der Elementaritaet der
Umkehrfunktion der durch f(x) = x + e^x gegebenen reellen Funktion
zu beantworten, gibt dabei die Definition elementarer Funktionen
nach <http://mathworld.wolfram.com/ElementaryFunction.html>.
Dort wird eine elementare Funktion charakterisiert durch

| A function built up of a finite combination of constant functions,
| field operations (addition, multiplication, division, and root
| extractions--the elementary operations)--and algebraic, exponential,
| and logarithmic functions and their inverses under repeated
| compositions [...]

Inverse elementarer Funktionen sind somit per Definition elementar.

Hans Crauel
Torn Rumero DeBrak
2017-02-09 00:11:09 UTC
Permalink
Post by Hans Crauel
Torn Rumero DeBrak schrieb
Post by Torn Rumero DeBrak
Post by IV
was ist die g e n a u e Ursache dafür, daß Gleichungen wie
y + e^y = x
und die Kepler-Gleichung
y - c*sin(y) = x, c eine Konstante,
keine Elementare Funktion y (siehe Wikipedia en: Elementary function) als
Lösung haben können?
Ist die Betragsfunktion
f: IR -> IR, x |-> |x|
eine elementare Funktion?
Fuer reelle x ist |x| = sqrt(x^2), also ist die Betragsfunktion
wohl eine elementare Funktion im Sinne von Wikipedia "Elementary
function".
Schade. Ich hoffte, Nummer 4 würde einmal selber nachdenken, und
auf den Unsinn stoßen, den er hier verzapfen will.
Post by Hans Crauel
Post by Torn Rumero DeBrak
Wenn ja, dann ist folglich nicht jede elementare Funktion
differenzierbar.
Wikipedia "Elementary function" meint, dass die Menge der elementaren
Funktionen "closed under differentiation" sei. Das ist offensichtlich
fuer die dort gegebene Charakterisierung nicht der Fall. Dem muesste
man wohl genauer nachgehen, wenn man ernsthaft Aussagen ueber
`elementare Funktionen' machen will.
Hans Crauel
2017-02-09 17:42:18 UTC
Permalink
Torn Rumero DeBrak schrieb
Post by Torn Rumero DeBrak
Post by Hans Crauel
Torn Rumero DeBrak schrieb
Post by Torn Rumero DeBrak
Ist die Betragsfunktion
f: IR -> IR, x |-> |x|
eine elementare Funktion?
Fuer reelle x ist |x| = sqrt(x^2), also ist die Betragsfunktion
wohl eine elementare Funktion im Sinne von Wikipedia "Elementary
function".
Schade. Ich hoffte, Nummer 4 würde einmal selber nachdenken, und
auf den Unsinn stoßen, den er hier verzapfen will.
Deinen Optimismus in Ehren, allein er scheint mir fehl am Platze.

Hans Crauel
IV
2017-02-09 18:10:27 UTC
Permalink
Post by Torn Rumero DeBrak
Ist die Betragsfunktion
f: IR -> IR, x |-> |x|
eine elementare Funktion?
Fuer reelle x ist |x| = sqrt(x^2), also ist die Betragsfunktion wohl eine
elementare Funktion im Sinne von Wikipedia "Elementary function".
Schade. Ich hoffte, Nummer 4 würde einmal selber nachdenken, und auf den
Unsinn stoßen, den er hier verzapfen will.
Ich weiß wieder nicht, was Ihr meint.
Meint Ihr, im Wikipedia-Artikel stimmt etwas nicht?
Ich hatte wiederholt geschrieben, was unter den Elementaren Funktionen zu
verstehen ist:
J. F. Ritt: "Elementary functions and their inverses", 1925:
"The elementary functions are understood here to be those which are obtained
in a finite number of steps by performing algebraic operations and taking
exponentials and logarithms."
Was also meint Ihr?
Torn Rumero DeBrak
2017-02-09 19:07:51 UTC
Permalink
Post by IV
Post by Torn Rumero DeBrak
Ist die Betragsfunktion
f: IR -> IR, x |-> |x|
eine elementare Funktion?
Fuer reelle x ist |x| = sqrt(x^2), also ist die Betragsfunktion wohl
eine elementare Funktion im Sinne von Wikipedia "Elementary function".
Schade. Ich hoffte, Nummer 4 würde einmal selber nachdenken, und auf
den Unsinn stoßen, den er hier verzapfen will.
Ich weiß wieder nicht, was Ihr meint.
Meint Ihr, im Wikipedia-Artikel stimmt etwas nicht?
Ich hatte wiederholt geschrieben, was unter den Elementaren Funktionen
"The elementary functions are understood here to be those which are
obtained in a finite number of steps by performing algebraic operations
and taking exponentials and logarithms."
Was also meint Ihr?
|x| = sqrt(x^2) = exp(1/2*ln(x^2))

welche andere Darstellung willst du noch?

Ist jetzt die Betragsfunktion elementar oder nicht?
IV
2017-02-09 16:58:07 UTC
Permalink
Post by Hans Crauel
Post by IV
was ist die g e n a u e Ursache dafür, daß Gleichungen wie
y + e^y = x
und die Kepler-Gleichung
y - c*sin(y) = x, c eine Konstante,
keine Elementare Funktion y (siehe Wikipedia en: Elementary function) als
Lösung haben können?
Wikipedia "Elementary function" meint, dass die Menge der elementaren
Funktionen "closed under differentiation" sei. Das ist offensichtlich fuer
die dort gegebene Charakterisierung nicht der Fall. Dem muesste
man wohl genauer nachgehen, wenn man ernsthaft Aussagen ueber 'elementare
Funktionen' machen will.
Wieso ist die Klasse der in "Wikipedia en: Elementary function" definierten
Funktionen nicht bezüglich Differentiation abgeschlossen?
Wikipedia en: Elementary function: "In mathematics, an elementary function
is a function of one variable which is the composition of a finite number of
arithmetic operations (+ – × ÷), exponentials, logarithms, constants, and
solutions of algebraic equations (a generalization of nth roots)."
"solutions of algebraic equations" sind die expliziten algebraischen
Funktionen.
Post by Hans Crauel
Eine einfache Moeglichkeit, die Frage nach der Elementaritaet der
Umkehrfunktion der durch f(x) = x + e^x gegebenen reellen Funktion zu
beantworten, gibt dabei die Definition elementarer Funktionen nach
<http://mathworld.wolfram.com/ElementaryFunction.html>.
Dort wird eine elementare Funktion charakterisiert durch
| A function built up of a finite combination of constant functions,
| field operations (addition, multiplication, division, and root
| extractions--the elementary operations)--and algebraic, exponential,
| and logarithmic functions and their inverses under repeated
| compositions [...]
Huch. Unterscheiden sich beide Definitionen? Worin denn? Ich sehe es nicht.
Post by Hans Crauel
Inverse elementarer Funktionen sind somit per Definition elementar.
Das stimmt ja nicht.
In "Mathworld: Elementary Function" ist gemeint, auch die Inversen der zuvor
genannten Funktionen sind Elementare Funktionen, sowie alle Kompositionen
dieser Funktionen.
Die Umkehrfunktion zu F(x) = x + e^x (bzw. LambertW) ist keine Elementare
Funktion.
Detlef Müller
2017-02-10 12:55:54 UTC
Permalink
Post by Hans Crauel
Post by Hans Crauel
Post by IV
was ist die g e n a u e Ursache dafür, daß Gleichungen wie
y + e^y = x
und die Kepler-Gleichung
y - c*sin(y) = x, c eine Konstante,
keine Elementare Funktion y (siehe Wikipedia en: Elementary function) als
Lösung haben können?
Wikipedia "Elementary function" meint, dass die Menge der elementaren
Funktionen "closed under differentiation" sei. Das ist offensichtlich
fuer die dort gegebene Charakterisierung nicht der Fall. Dem muesste
man wohl genauer nachgehen, wenn man ernsthaft Aussagen ueber
'elementare Funktionen' machen will.
Sicher ist hier der Zusatz "fast überall" zu machen.
Schon rationale Funktionen sind ja nicht überall
definiert.
[...]
Post by Hans Crauel
Post by Hans Crauel
Eine einfache Moeglichkeit, die Frage nach der Elementaritaet der
Umkehrfunktion der durch f(x) = x + e^x gegebenen reellen Funktion zu
beantworten, gibt dabei die Definition elementarer Funktionen nach
<http://mathworld.wolfram.com/ElementaryFunction.html>.
Dort wird eine elementare Funktion charakterisiert durch
| A function built up of a finite combination of constant functions,
| field operations (addition, multiplication, division, and root
| extractions--the elementary operations)--and algebraic, exponential,
| and logarithmic functions and their inverses under repeated
^^^^^^^^^^^^^^^^^^
Post by Hans Crauel
Post by Hans Crauel
| compositions [...]
Huch. Unterscheiden sich beide Definitionen? Worin denn? Ich sehe es nicht.
Der kleine Zusatz "and their inverses", auf den Du ja auch eingehst.
Post by Hans Crauel
Post by Hans Crauel
Inverse elementarer Funktionen sind somit per Definition elementar.
Das stimmt ja nicht.
In "Mathworld: Elementary Function" ist gemeint, auch die Inversen der
zuvor genannten Funktionen sind Elementare Funktionen, sowie alle
Kompositionen dieser Funktionen.
Diese Interpretation ist auf den zweiten Blick etwas seltsam,
denn es werden explizit aufgezählt:
- Multiplikation und Division,
Körperoperationen und damit
- Potenzieren, sowie Wurzeln ziehen
und
- Exponentialfunktion und Logarithmusfunktion.

Da sind doch die Umkehrfunktionen bereits dabei, wozu
also noch der Zusatz?

Man sollte aber statt zu Rätseln unter den angegebenen
Quellen Chow und Shanks nachschauen, was denn genau
gemeint ist.

Ob die exakte Definition dort wirklich so lang ist,
dass sie auf so flapsige Weise zusammen gefasst werden
müsste könnte man dann überprüfen (ich habe ja meine
Zweifel).

Gruß,
Detlef
--
Dr. Detlef Müller,
http://www.mathe-doktor.de oder http://mathe-doktor.de
IV
2017-02-10 22:03:35 UTC
Permalink
Post by Detlef Müller
Post by Hans Crauel
Post by IV
was ist die g e n a u e Ursache dafür, daß Gleichungen wie
y + e^y = x
und die Kepler-Gleichung
y - c*sin(y) = x, c eine Konstante,
keine Elementare Funktion y (siehe Wikipedia en: Elementary function)
als Lösung haben können?
Wikipedia "Elementary function" meint, dass die Menge der elementaren
Funktionen "closed under differentiation" sei. Das ist offensichtlich
fuer die dort gegebene Charakterisierung nicht der Fall. Dem muesste man
wohl genauer nachgehen, wenn man ernsthaft Aussagen ueber 'elementare
Funktionen' machen will.
Sicher ist hier der Zusatz "fast überall" zu machen.
Schon rationale Funktionen sind ja nicht überall definiert.
Ritt schreibt, daß die Elementaren Funktionen fast überall analytisch sind.
In der Computeralgebra ist klar, daß jede ("Liouvillesche") Elementare
Funktion differenzierbar ist, und ihre Ableitung wieder eine Elementare
Funktion ist.
Post by Detlef Müller
Man sollte aber statt zu rätseln unter den angegebenen Quellen Chow und
Shanks nachschauen, was denn genau gemeint ist.
Im Mathworld-Artikel sind als Quellen für die Definition Ritt, Chow und
Shanks angegeben.
Alle definieren dieselbe Klasse von Funktionen.
Ritt 1925: "The elementary functions are understood here to be those which
are obtained in a finite number of steps by performing algebraic operations
and taking exponentials and logarithms."
Shanks 1978, 1993: "elementary functions - that is, a finite combination of
algebraic, trigonometric and exponential functions together with their
inverses".
Chow 1999: "Recall that a function is elementary if it can be constructed
using only a finite combination of constant functions, field operations, and
algebraic, exponential, and logarithmic functions."
Bronstein ("Symbolic Integration", 1991) verwendet ebenfalls eine analoge
Definition, allerdings körpertheoretisch beschrieben.
Die algebraischen Operationen sind algebraische Funktionen.

Die Definition im MathWorld-Artikel ist also falsch. Ich werde das dort mal
melden. Früher hatten sie schon mal eine vernünftige Definition.
IV
2017-02-09 16:44:39 UTC
Permalink
Post by Torn Rumero DeBrak
Post by IV
was ist die g e n a u e Ursache dafür, daß Gleichungen wie
y + e^y = x
und die Kepler-Gleichung
y - c*sin(y) = x, c eine Konstante,
keine Elementare Funktion y (siehe Wikipedia en: Elementary function) als
Lösung haben können?
Ist die Betragsfunktion
f: IR -> IR, x |-> |x|
eine elementare Funktion?
Ist die Betragsfunktion als Komposition von exp, ln und/oder expliziten
algebraischen einwertigen Funktionen darstellbar?
Ist die Betragsfunktion eine algebraische Funktion?
Die Elementaren Funktionen sind bezüglich der Differentiation abgeschlossen.
Post by Torn Rumero DeBrak
Wenn ja, dann ist folglich nicht jede elementare Funktion differenzierbar.
Wenn nein, warum nicht?
Was ist denn nun Deine Antwort auf meine Frage von ganz oben? "Nein, darum
nicht?"
Torn Rumero DeBrak
2017-02-09 18:59:12 UTC
Permalink
Post by IV
Post by Torn Rumero DeBrak
Post by IV
was ist die g e n a u e Ursache dafür, daß Gleichungen wie
y + e^y = x
und die Kepler-Gleichung
y - c*sin(y) = x, c eine Konstante,
keine Elementare Funktion y (siehe Wikipedia en: Elementary function)
als Lösung haben können?
Ist die Betragsfunktion
f: IR -> IR, x |-> |x|
eine elementare Funktion?
Ist die Betragsfunktion als Komposition von exp, ln und/oder expliziten
algebraischen einwertigen Funktionen darstellbar?
So ist der Begriff "Elementare Funktion" auf der Wiki-Seite nicht
definiert. WEnn du den nicht verwendest,
dann gebe diese Seite nicht als Definition für
eine elementare Funktion an and zitiere einaml die von
dir verwendete.
Post by IV
Ist die Betragsfunktion eine algebraische Funktion?
Die Elementaren Funktionen sind bezüglich der Differentiation
abgeschlossen.
Post by Torn Rumero DeBrak
Wenn ja, dann ist folglich nicht jede elementare Funktion
differenzierbar.
Wenn nein, warum nicht?
Was ist denn nun Deine Antwort auf meine Frage von ganz oben? "Nein,
darum nicht?"
Nun ist aber die Betragsfunktion nicht differenzierbar, wie
jeder Anfänger schon zeigen kann. Damit ist sie nicht
elementar (nach Wiki), obwohl sie als sqrt(x^2) elementar sein
sollte.

Der Begriff ist also momentan widersprüchlich (und war es wohl schon von
Anbeginn seiner Verwendung)
Detlef Müller
2017-02-10 13:39:57 UTC
Permalink
Post by Torn Rumero DeBrak
Post by IV
Post by Torn Rumero DeBrak
Post by IV
was ist die g e n a u e Ursache dafür, daß Gleichungen wie
y + e^y = x
und die Kepler-Gleichung
y - c*sin(y) = x, c eine Konstante,
keine Elementare Funktion y (siehe Wikipedia en: Elementary function)
als Lösung haben können?
Ist die Betragsfunktion
f: IR -> IR, x |-> |x|
eine elementare Funktion?
Ist die Betragsfunktion als Komposition von exp, ln und/oder expliziten
algebraischen einwertigen Funktionen darstellbar?
So ist der Begriff "Elementare Funktion" auf der Wiki-Seite nicht
definiert.
Ich sehe gerade auf

https://en.wikipedia.org/wiki/Elementary_function


und unter

https://de.wikipedia.org/wiki/Elementare_Funktion,
"Definitionsversuche"

nicht, welche Feinheit da jetzt fehlt.

Die "Wolfram"-Definition a la "Elementar ist, was wir in
Mathematica implementiert haben" halte ich für eher
Betriebswirtschaftlich motiviert.

Die Klassifizierung nach Robert Risch sieht dagegen
für mich "natürlich" aus.

[...]
Post by Torn Rumero DeBrak
Nun ist aber die Betragsfunktion nicht differenzierbar, wie
jeder Anfänger schon zeigen kann. Damit ist sie nicht
elementar (nach Wiki), obwohl sie als sqrt(x^2) elementar sein
sollte.
Differenzierbarkeit sehe ich in der englischen Wiki nicht als
Teil der Definition, sondern als Folgerung daraus ... und
offenbar sind die Elementaren Funktionen bestenfalls "fast überall"
definiert, schon die einfachen rationalen Funktionen haben
ja Polstellen.
Somit dürfte "fast überall differenzierbar" gemeint sein, wer
in Englisch fitt ist, könnte das ja nachtragen.
Post by Torn Rumero DeBrak
Der Begriff ist also momentan widersprüchlich (und war es wohl schon von
Anbeginn seiner Verwendung)
Zumindest Risch konnte anscheinend mit seinen Definition ganz
ordentliche Resultate erreichen und von Fehlern in seiner Arbeit
habe ich nichts gehört.

Ohne da jetzt genauer hinein gesehen zu haben, wird dort sicher
in Strukturen gerechnet, deren Elemente selbst Funktionen sind,
z.B. dem Körper der rationalen Funktionen, an den dann neue Funktionen
adjungiert (z.B. log(x)) werden.

Gruß,
Detlef
--
Dr. Detlef Müller,
http://www.mathe-doktor.de oder http://mathe-doktor.de
IV
2017-02-10 22:12:17 UTC
Permalink
Post by Torn Rumero DeBrak
So ist der Begriff "Elementare Funktion" auf der Wiki-Seite nicht
definiert.
Zumindest Risch konnte anscheinend mit seinen Definition ganz ordentliche
Resultate erreichen und von Fehlern in seiner Arbeit habe ich nichts
gehört.
[Risch 1979] definiert dieselbe Funktionenklasse wie Liouville, Ritt,
Shanks, Chow und Bronstein: "The elementary functions of a complex variable
z are those functions built up from the rational functions of z by
exponentiation, taking
logarithms, and algebraic operations.".
Christian Gollwitzer
2017-02-11 20:32:32 UTC
Permalink
Post by IV
was ist die g e n a u e Ursache dafür, daß Gleichungen wie
y + e^y = x
und die Kepler-Gleichung
y - c*sin(y) = x, c eine Konstante,
keine Elementare Funktion y (siehe Wikipedia en: Elementary function) als
Lösung haben können?
Die genaue Ursache ist die, dass es nicht geht. Ja, das klingt nicht
nach einer guten Begründung, ist aber die richtige: im Grunde ist es
eine Besonderheit, wenn man eine Funktion analytisch invertieren kann.

Ähnlich könntest Du fragen, was ist die genaue Ursache dafür, dass im
Ring der ganzen Zahlen 10 nicht durch 3 teilbar ist? Es geht halt
einfach nicht. Nur wenn man den Ring zu einem Körper erweitert, kann man
durch jede Zahl außer Null teilen. Genauso bilden die elementaren
Funktionen unter der Operation "Inversion" keine abgeschlossene
Struktur. Das geht nicht mal, wenn man nur Körperoperationen zulässt,
wie schon das einfache Beispiel eines Polynoms zeigt. Eine
abgeschlossene Struktur bilden z.B. die linearen Funktionen, die kann
man immer invertieren.

Im Übrigen wird es nichts bringen, wenn Du 10 Diskussionsfäden mit
demselben Thema aufmachst. Davon kriegst Du auch keine besseren Antworten.

Christian
IV
2017-02-11 20:55:45 UTC
Permalink
Post by IV
was ist die g e n a u e Ursache dafür, daß Gleichungen wie
y + e^y = x
und die Kepler-Gleichung
y - c*sin(y) = x, c eine Konstante,
keine Elementare Funktion y (siehe Wikipedia en: Elementary function)
als Lösung haben können?
Die genaue Ursache ist die, dass es nicht geht. Ja, das klingt nicht nach
einer guten Begründung, ist aber die richtige
Du hast recht: Deine Begründung ist richtig. Und, Deine Antwort klingt nicht
nach Mathematik.
IV
2017-02-18 22:23:36 UTC
Permalink
Post by IV
was ist die g e n a u e Ursache dafür, daß Gleichungen wie
y + e^y = x
und die Kepler-Gleichung
y - c*sin(y) = x, c eine Konstante,
keine Elementare Funktion y (siehe Wikipedia en: Elementary function) als
Lösung haben können?
Es ist auch möglich, einfachere Teilprobleme zu betrachten:
x + e^x = a oder x + sin(x) = b, mit a und b Konstanten.
Was ist die genaue Ursache dafür, daß niemand hier diese einfache
mathematische Frage auch nur ansatzweise beantworten kann?

fragend

IV
Detlef Müller
2017-02-18 22:29:37 UTC
Permalink
[...]
Post by IV
x + e^x = a oder x + sin(x) = b, mit a und b Konstanten.
Was ist die genaue Ursache dafür, daß niemand hier diese einfache
mathematische Frage auch nur ansatzweise beantworten kann?
Vermutlich die Tatsache, daß einfache Fragen nicht immer
auch einfache Antworten haben.

Gruß,
Detlef
--
Dr. Detlef Müller,
http://www.mathe-doktor.de oder http://mathe-doktor.de
IV
2017-02-19 11:51:31 UTC
Permalink
Post by IV
Post by IV
was ist die g e n a u e Ursache dafür, daß Gleichungen wie
y + e^y = x
und die Kepler-Gleichung
y - c*sin(y) = x, c eine Konstante,
keine Elementare Funktion y (siehe Wikipedia en: Elementary function)
als Lösung haben können?
x + e^x = a oder x + sin(x) = b, mit a und b Konstanten.
Was ist die genaue Ursache dafür, daß niemand hier diese einfache
mathematische Frage auch nur ansatzweise beantworten kann?
Vermutlich die Tatsache, daß einfache Fragen nicht immer auch einfache
Antworten haben.
Dann bin ich also falsch hier?
Andreas Leitgeb
2017-02-19 13:01:56 UTC
Permalink
Post by IV
Post by IV
Was ist die genaue Ursache dafür, daß niemand hier diese einfache
mathematische Frage auch nur ansatzweise beantworten kann?
Vermutlich die Tatsache, daß einfache Fragen nicht immer auch einfache
Antworten haben.
Dann bin ich also falsch hier?
Mit der konkreten Frage nach der Umkehrfunktion von x+exp(x) bzw nach
konkreten Begründungen warum eine solche nicht genauso schön darstell-
bar sein sollte, bist du hier zwar nicht prinzipiell falsch, aber es
gibt (zumindest hier) halt keine Antwort *darauf*.

Ob du anderswo bessere Antworten erhalten würdest, wage ich zwar auch
zu bezweifeln, aber das hat dich schon bisher nicht davon abgehalten,
nochmal und nochmal danach zu fragen...

Nächstes mal vielleicht dann mal anderswo.

Zum Abschluss hier noch eine komplette Liste an Orten, an denen ich
eine bessere Antwort auf die Frage nach der Umkehrfunktion von x+exp(x)
erwarten würde:
IV
2017-02-19 13:20:57 UTC
Permalink
Post by Andreas Leitgeb
Post by IV
Post by IV
Was ist die genaue Ursache dafür, daß niemand hier diese einfache
mathematische Frage auch nur ansatzweise beantworten kann?
Vermutlich die Tatsache, daß einfache Fragen nicht immer auch einfache
Antworten haben.
Dann bin ich also falsch hier?
Mit der konkreten Frage nach der Umkehrfunktion von x+exp(x) bzw nach
konkreten Begründungen warum eine solche nicht genauso schön darstellbar
sein sollte, bist du hier zwar nicht prinzipiell falsch, aber es gibt
(zumindest hier) halt keine Antwort *darauf*.
Ob du anderswo bessere Antworten erhalten würdest, wage ich zwar auch zu
bezweifeln, aber das hat dich schon bisher nicht davon abgehalten, nochmal
und nochmal danach zu fragen...
Ein Wissenschaf(f)tler sollte "Wissen schaffen". Das heißt, wenn ein Problem
zu kompliziert oder zu aufwendig ist, dann sollte man das Problem in
Teilproblem zerlegen.
Und nichts anderes mache ich hier.
Post by Andreas Leitgeb
Nächstes mal vielleicht dann mal anderswo.
Sicher.
Post by Andreas Leitgeb
Zum Abschluss hier noch eine komplette Liste an Orten, an denen ich eine
bessere Antwort auf die Frage nach der Umkehrfunktion von x+exp(x)
-
-
-
...
(Daraus spricht zwar auch wieder Rainers Nachname, auf das passende
Wortspiel mit Deinem Nachnamen verzichte ich jetzt aber mal. (Sonst
antwortet Ihr wieder nur darauf und braucht Euch nicht mit dem
mathematischen Problem beschäftigen.))
Ich ergänze Deine Liste mal:
Matheboard
Mathematics Stack Exchange
MathOverflow
Matroids Matheplanet
OnlineMathe
sci.math
sci.math.research
sci.math.symbolic
Andreas Leitgeb
2017-02-19 13:37:23 UTC
Permalink
"Andreas Leitgeb" schrieb im Newsbeitrag
Post by Andreas Leitgeb
Zum Abschluss hier noch eine komplette Liste an Orten, an denen ich eine
bessere Antwort auf die Frage nach der Umkehrfunktion von x+exp(x)
-
Das ist ein verfälschtes Zitat, da mein post keine "-" Zeile enthielt.
Da das vorige die Liste jener Orte war, an denen *ich* bessere
Antworten erwarten würde, bist du nicht in der Lage, diese Liste
für mich zu "ergänzen".

Viel Spass aber beim Abarbeiten *deiner* Liste.
Matheboard
Mathematics Stack Exchange
MathOverflow
Matroids Matheplanet
OnlineMathe
sci.math
sci.math.research
sci.math.symbolic
IV
2017-02-19 13:33:03 UTC
Permalink
Post by Andreas Leitgeb
Post by IV
Post by IV
Was ist die genaue Ursache dafür, daß niemand hier diese einfache
mathematische Frage auch nur ansatzweise beantworten kann?
Vermutlich die Tatsache, daß einfache Fragen nicht immer auch einfache
Antworten haben.
Dann bin ich also falsch hier?
Zum Abschluss hier noch eine komplette Liste an Orten, an denen ich eine
bessere Antwort auf die Frage nach der Umkehrfunktion von x+exp(x)
erwarten würde
(Nichts für ungut, aber ich nehme hiermit lediglich Eure Antwort vorweg - im
Spaß: Könntest Du Deine Frage bitte in korrekter mathematischer Notation
stellen? Da dies hier ein Mathematikforum ist, verstehen wir sie sonst
nämlich nicht. Eine Frage nach einer Umkehrfunktion setzt voraus, daß man
eine Funktion angibt, zu der man eine Frage nach ihrer Umkehrfunktion
stellen kann. "x+exp(x)" ist keine Funktion, sondern ein mathematischer
Ausdruck. Eine Funktionsdefinition beinhaltet die Angabe von
Definitionsmenge, Zielmenge, Bildmenge und Abbildungsvorschrift.
Da ich weiß, daß Du kein Mathematiker bist, solltest Du hier erst Fragen
stellen, wenn Du ein oder zwei Semester Mathematik studiert hast. Das ist
doch nicht zuviel verlangt. Wir stellen ja auch keine Fragen an Dein Fach.)
H0Iger SchuIz
2017-02-20 12:10:26 UTC
Permalink
Post by IV
(Nichts für ungut, aber ich nehme hiermit lediglich Eure Antwort vorweg - im
Spaß: Könntest Du Deine Frage bitte in korrekter mathematischer Notation
stellen? Da dies hier ein Mathematikforum ist, verstehen wir sie sonst
nämlich nicht. Eine Frage nach einer Umkehrfunktion setzt voraus, daß man
eine Funktion angibt, zu der man eine Frage nach ihrer Umkehrfunktion
stellen kann. "x+exp(x)" ist keine Funktion, sondern ein mathematischer
Ausdruck. Eine Funktionsdefinition beinhaltet die Angabe von
Definitionsmenge, Zielmenge, Bildmenge und Abbildungsvorschrift.
Also ganz offensichtlich hat Nummer vier docvh schon etwas verstanden.
Warum sie das nicht umsetzt, bleibt wohl ihr Geheimnis.

hs
Christian Gollwitzer
2017-02-19 13:46:40 UTC
Permalink
Post by Andreas Leitgeb
Post by IV
Post by IV
Was ist die genaue Ursache dafür, daß niemand hier diese einfache
mathematische Frage auch nur ansatzweise beantworten kann?
Vermutlich die Tatsache, daß einfache Fragen nicht immer auch einfache
Antworten haben.
Dann bin ich also falsch hier?
Mit der konkreten Frage nach der Umkehrfunktion von x+exp(x) bzw nach
konkreten Begründungen warum eine solche nicht genauso schön darstell-
bar sein sollte, bist du hier zwar nicht prinzipiell falsch, aber es
gibt (zumindest hier) halt keine Antwort *darauf*.
Ob du anderswo bessere Antworten erhalten würdest, wage ich zwar auch
zu bezweifeln, aber das hat dich schon bisher nicht davon abgehalten,
nochmal und nochmal danach zu fragen...
Nächstes mal vielleicht dann mal anderswo.
Zum Abschluss hier noch eine komplette Liste an Orten, an denen ich
eine bessere Antwort auf die Frage nach der Umkehrfunktion von x+exp(x)
Hier:

http://www.wolframalpha.com/input/?i=solve+x%2Bexp(x)%3Dy

gibt es eine Antwort auf die konkrete Frage. x+e^x war kein gutes
Beipsiel. Warum jetzt die LambertW-Funktion nicht als elementare
Funktion geschrieben werden kann, lässt sich vermutlich einfach beweisen.

Christian
IV
2017-02-19 14:18:30 UTC
Permalink
Post by Andreas Leitgeb
Ob du anderswo bessere Antworten erhalten würdest, wage ich zwar auch zu
bezweifeln
Warum jetzt die LambertW-Funktion nicht als elementare Funktion
geschrieben werden kann, lässt sich vermutlich einfach beweisen.
Na, das läßt sich eben bisher nicht auf einfachem Wege beweisen, wie der
folgende Artikel zeigt:
Bronstein, M.; Corless, R. M.; Davenport, J. H., Jeffrey, D. J.: Algebraic
properties of the Lambert W Function from a result of Rosenlicht and of
Liouville. Integral Transforms and Special Functions 19 (2008) (10) 709-712.
Wie sich das in diesem eben genannten Artikel und in
Rosenlight, M.: On the explicit solvability of certain transcendental
equations. Publications math\'ematiques de l'IH\'ES 36 (1969) 15-22
liest (Ich habe beide bisher nur überflogen, weil ich ja seit Wochen mit dem
Antworten auf Eure vom eigentlichen Thema wegführenden Antworten beschäftigt
bin.), hat man die Analysis der jeweils gegebenen Funktion genauestens zu
untersuchen.
Mit Ritts Satz kann man die Antwort aber für nicht zu komplexe Funktionen
sofort, für komplexere Funktionen wahrscheinlich im Anschluß an einen
systematischen Transformationsalgorithmus in eine Standardform, aus der
Abbildungsvorschrift (expliziten Funktionsgleichung) ablesen.
Und ich möchte einfach nur den Beweis von Ritts Satz vereinfachen - weil der
Unterschied offenbar wirklich nur in der Stelligkeit der in der
expln-Verkettungsdarstellung ("Liouville-Darstellung") zugelassenen
algebraischen Funktionen liegt.
Jürgen R.
2017-02-19 13:55:46 UTC
Permalink
Post by Detlef Müller
[...]
Post by IV
x + e^x = a oder x + sin(x) = b, mit a und b Konstanten.
Was ist die genaue Ursache dafür, daß niemand hier diese einfache
mathematische Frage auch nur ansatzweise beantworten kann?
Vermutlich die Tatsache, daß einfache Fragen nicht immer
auch einfache Antworten haben.
Gruß,
Detlef
Selbstverständlich haben einfache Fragen immer einfache Antworten.

"There is always a well-known solution to every human problem — neat,
plausible, and wrong." H.L.Mencken
H0Iger SchuIz
2017-02-20 10:50:15 UTC
Permalink
Post by Detlef Müller
Post by IV
Was ist die genaue Ursache dafür, daß niemand hier diese einfache
mathematische Frage auch nur ansatzweise beantworten kann?
Vermutlich die Tatsache, daß einfache Fragen nicht immer
auch einfache Antworten haben.
Vermutlich versteht er unter einer "einfachen" Frage eine "einfach zu
benatwortende". Allerdings frage ich mich, wie man das beurteilen können
soll, wenn man die Antwort nicht kennt.

Die Frage nach einer "genauen Ursache" sollte aber hellhörig werden
lassen. Das geht weit über innermathematische Schlüsse hinaus und
erfordert ein sehr tiefgehendes Verständnis. Das hat hier wohl keiner,
weil sich jemand hier jemals mit dieser Thematik beschäftigt hat.

Die Idee, man könne mathematische Forschung betreiben, ohne Ahnung von
Mathematik zu haben, weil man ja nur die Sachen, die man nicht versteht
erfragen muss, um dann aus den Antworten eine "Forschungsergebnis"
zusammenzusetzen, mutet schon einigermaßen absurd an.

hs
Jens Kallup
2017-02-20 12:55:14 UTC
Permalink
Post by H0Iger SchuIz
Die Idee, man könne mathematische Forschung betreiben, ohne Ahnung von
Mathematik zu haben, weil man ja nur die Sachen, die man nicht versteht
erfragen muss, um dann aus den Antworten eine "Forschungsergebnis"
zusammenzusetzen, mutet schon einigermaßen absurd an.
Das ist richtig.
Sich mit fremden Federn schmücken ist nicht sehr toll.
Ganz und garnicht, wenn noch nicht einmal die Quellen
als Credits angegeben werden.
H0Iger SchuIz
2017-02-20 13:06:54 UTC
Permalink
Post by Jens Kallup
Post by H0Iger SchuIz
Die Idee, man könne mathematische Forschung betreiben, ohne Ahnung von
Mathematik zu haben, weil man ja nur die Sachen, die man nicht versteht
erfragen muss, um dann aus den Antworten eine "Forschungsergebnis"
zusammenzusetzen, mutet schon einigermaßen absurd an.
Das ist richtig.
Sich mit fremden Federn schmücken ist nicht sehr toll.
Ganz und garnicht, wenn noch nicht einmal die Quellen
als Credits angegeben werden.
Darum ging's nicht. Aber das wäre ja das erste Mal, das "Jens" merkt,
worum es geht.

Die Unterstellung Nummer vier werde ihre Quellen nicht angeben und
fremde Ergebnisse als eigene ausgeben, ist nicht begründet.

hs
Jens Kallup
2017-02-20 13:25:00 UTC
Permalink
Post by H0Iger SchuIz
Darum ging's nicht. Aber das wäre ja das erste Mal, das "Jens" merkt,
worum es geht.
ich hab das so interpretiert, das Nummer vier ein Studium
ohne Abschluss hat, und nun versucht Fragen zu finden.
Post by H0Iger SchuIz
Die Unterstellung Nummer vier werde ihre Quellen nicht angeben und
fremde Ergebnisse als eigene ausgeben, ist nicht begründet.
Ok.
Aber sind wir mal ehrlich: Wer hat nicht schon mal gesagt:
"Der Herr xyz hat das so gesagt, das muss so sein!"

Das Gute an den _noch_ bestehenden Gruppen hier ist doch der,
das sich gleichgesinnte treffen und plaudern.
Damit der Mathe Verein xyz weiter bestehen kann, werden Mitglieder
aufgenommen; und man kann sich um Dinge unterhalten, die Spaß machen.

Das da beim biten Bier mal was aus der Reihe gemurmelt wird, macht es
doch aus, das man sich am Stammtisch trifft und rumsabbelt - wie ich
gerade ;-) - Erkenntnis: Da schmeckt das trite Bier um so leckerer.

Mir macht es Spaß, Leuten zu helfen; auch dann, wenn man von vorne
anfangen muss...
Ich kann verstehen, das Ihr ein gewisses Niveau halten wollt.
und die Gruppen, die möglicherweise das ww3 bestehen, eine Art Grundlagen-
Archiv für die Nachkommen darstellen soll, ist schon begründet.
Rainer Hohn
2017-02-19 00:30:14 UTC
Permalink
IV schrieb
Post by IV
Post by IV
was ist die g e n a u e Ursache dafür, daß Gleichungen wie
y + e^y = x
und die Kepler-Gleichung
y - c*sin(y) = x, c eine Konstante,
keine Elementare Funktion y (siehe Wikipedia en: Elementary function) als
Lösung haben können?
x + e^x = a oder x + sin(x) = b, mit a und b Konstanten.
Zum Beispiel für a=1 und a=1+e ist die Antwort einfach, ebenso
wie für eine Vielzahl weiterer Werte von a. Oder für b = 0 und
allgemeinerer b = k pi, k eine ganze Zahl. Damit weiß man dann
schon einiges über Werte von b zwischen k pi und (k+1) pi. Für
die interdisziplinären Ansprüche reicht das oft schon völlig
aus.
Post by IV
Was ist die genaue Ursache dafür, daß niemand hier diese einfache
mathematische Frage auch nur ansatzweise beantworten kann?
Ansatzweise ist das nun wirklich sehr einfach zu beantworten.
Ob es eine mathematische Frage ist, sei dahingestellt. Siehe
dahingehend etwa <http://genes.mit.edu/burgelab/piffles.html>.

Rainer
IV
2017-02-19 11:48:10 UTC
Permalink
Post by IV
Post by IV
was ist die g e n a u e Ursache dafür, daß Gleichungen wie
y + e^y = x
und die Kepler-Gleichung
y - c*sin(y) = x, c eine Konstante,
keine Elementare Funktion y (siehe Wikipedia en: Elementary function) als
Lösung haben können?
x + e^x = a oder x + sin(x) = b, mit a und b Konstanten.
Zum Beispiel für a=1 und a=1+e ist die Antwort einfach, ebenso wie für
eine Vielzahl weiterer Werte von a. Oder für b = 0 und allgemeinerer b = k
pi, k eine ganze Zahl. Damit weiß man dann schon einiges über Werte von b
zwischen k pi und (k+1) pi. Für die interdisziplinären Ansprüche reicht
das oft schon völlig aus.
Das verbietet aber nicht, nach allen a und b, für die diese Gleichungen
auflösbar sind, zu fragen.
Post by IV
Was ist die genaue Ursache dafür, daß niemand hier diese einfache
mathematische Frage auch nur ansatzweise beantworten kann?
Ansatzweise ist das nun wirklich sehr einfach zu beantworten.
Ob es eine mathematische Frage ist, sei dahingestellt. Siehe dahingehend
etwa <http://genes.mit.edu/burgelab/piffles.html>.
(Du machst Deinem Namen wieder alle Ehre.)
IV
2017-02-19 15:15:55 UTC
Permalink
Post by IV
Post by IV
was ist die g e n a u e Ursache dafür, daß Gleichungen wie
y + e^y = x
und die Kepler-Gleichung
y - c*sin(y) = x, c eine Konstante,
keine Elementare Funktion y (siehe Wikipedia en: Elementary function) als
Lösung haben können?
x + e^x = a oder x + sin(x) = b, mit a und b Konstanten.
Ich fange endlich einfach mal an, Teilantworten zu suchen:

1.) x + e^x = 1

In der Argumentation kann man jetzt bei einer Gleichung bleiben, man kann
aber auch die übergeordnete, allgemeinere, Fragestellung nach der
Umkehrfunktion einer entsprechenden in der Gleichung enthaltenen elementaren
Funktion bearbeiten. (Keine Ahnung, ob es eine entsprechende
Gleichungstheorie gibt, aber Ritts Satz (in Ritt, J. F.: Elementary
functions and their inverses. Trans. Amer. Math. Soc. 27 (1925) 68-90.
http://www.ams.org/journals/tran/1925-027-01) beschreibt das sehr gut für
Funktionen, und ich denke, das wäre auch die Argumentation für Gleichungen.)

Ich bleibe erstmal beim Gleichungsbild.
x + e^x = 1 ist eine algebraische Gleichung in x und e^x, also in den
Unbekannten(?)/Transzendenten(?) x und e^x. Wenn die Unbekannten einer
algebraischen Gleichung alle algebraisch abhängig sind, dann kann die
Gleichung in eine algebraische Gleichung einer einzigen Unbekannten
transformiert werden. Und solche Gleichung ist elementar auflösbar - wenn
die impliziten algebraischen Funktionen zu den Elementaren Funktionen
dazugezählt werden. (Mit Ritts Methode: Die Umkehrfunktion einer
algebraischen (einstelligen - IV) Funktion ist wieder eine solche, und
solche Funktionen gehören zu den Elementaren Funktionen.)

2.) x + e^x = 0

x + e^x = 1 ist eine algebraische Gleichung in x und e^x, also in den
Unbekannten(?)/Transzendenten(?) x und e^x. x und e^x sind algebraisch
unabhängig für alle x <> 0. x = 0 ist aber keine Lösung der Gleichung.
Die Gleichung kann nicht in eine algebraische Gleichung mit nur
algebraischen Unbekannten transformiert werden. Die äußere algebraische
Funktion der Gleichung (die algebraische Funktion mit der
Abbildungsvorschrift (x,e^x) \mapsto "+"(x,e^x) = x + e^x) ist zweistellig,
kann also, wie aus Ritts Satz folgt, keine Elementare Funktion als
Umkehrfunktion haben.

Ich versuche nun das Ergebnis der Anwendung von Ritts Satz auf diese
Gleichung zu verstehen.
Durch keine elementaren Umkehrfunktionen der in der Gleichung vertretenen
elmentaren Funktionen kann die Gleichung nach x aufgelöst werden. Warum kann
sie aber nicht durch irgendwelche anderen, also nicht in der Gleichung
"steckenden" elementaren Funktionen nach x aufgelöst werden?
Das ist doch die alles entscheidende Frage.

Kann jemand helfen, beginnen, eine Antwort auf diese Frage zu finden?
Jens Kallup
2017-02-19 21:41:07 UTC
Permalink
Post by IV
Ich versuche nun das Ergebnis der Anwendung von Ritts Satz auf diese
Gleichung zu verstehen.
Durch keine elementaren Umkehrfunktionen der in der Gleichung
vertretenen elmentaren Funktionen kann die Gleichung nach x aufgelöst
werden. Warum kann sie aber nicht durch irgendwelche anderen, also
nicht in der Gleichung "steckenden" elementaren Funktionen nach x
aufgelöst werden?
Das ist doch die alles entscheidende Frage.
Kann jemand helfen, beginnen, eine Antwort auf diese Frage zu finden?
Gleichung: x + e^x = 0

e ist eine Konstante, daher = 0
x ist eine natürliche Zahl (0 bis 9)

wir starten immer bei x = 0 .
0 ist eine Konstante, daher = 0

Lösung: x + e = 0
0 + 0 = 0

zum Anfang:

f(x) = e^x oder y = e^x
e ist einer eulersche Zahl (diese Zahl entspricht etwa 2,718).

und nun noch die Wertetabelle aufzeichnen für:

y = 2,718 hoch x

x | 0 | 0,5
---+---+------ ...
y | 1 | 1,65


Ich weiß nun nicht genau, ob im obigen Text "kein" vielleicht nicht doch
falsch ist - was meint Nummer 4 dazu?

Nehmen wir mal an, x und y sind unterschiedlich Paar Socken.
Wie sollte dann von x auf y oder von y auf x geschlossen werden?

Wir befinden uns in der Funktionenlehre, wo gilt:
(das T habe ich eingeführt, um den "Term" T etwas heraus zu heben)

x = 1
T = x^2 + 4x + 2^2
y = f(T)

Lösen wir auf:

T = 1^2 + 4*1 + 2^2
T = 1 + 4 + 4
T = 9

y = f(T)
y = 9

Probe aufs Exsämpel:

y = x^2 + 4x + 2^2 | minus 4
-4 = x^2 + 4x | minus 1^2 = 1 und: -4x (-4*1)
-4 -1 -4 = 0 | plus 9
y = 9

Tada: wir haben das Ergebnis:

x = 1
y = 9
Torn Rumero DeBrak
2017-02-20 14:51:13 UTC
Permalink
Post by IV
Post by IV
Post by IV
was ist die g e n a u e Ursache dafür, daß Gleichungen wie
y + e^y = x
und die Kepler-Gleichung
y - c*sin(y) = x, c eine Konstante,
keine Elementare Funktion y (siehe Wikipedia en: Elementary function)
als Lösung haben können?
x + e^x = a oder x + sin(x) = b, mit a und b Konstanten.
1.) x + e^x = 1
In der Argumentation kann man jetzt bei einer Gleichung bleiben, man
kann aber auch die übergeordnete, allgemeinere, Fragestellung nach der
Umkehrfunktion einer entsprechenden in der Gleichung enthaltenen
elementaren Funktion bearbeiten. (Keine Ahnung, ob es eine entsprechende
Gleichungstheorie gibt, aber Ritts Satz (in Ritt, J. F.: Elementary
functions and their inverses. Trans. Amer. Math. Soc. 27 (1925) 68-90.
http://www.ams.org/journals/tran/1925-027-01) beschreibt das sehr gut
für Funktionen, und ich denke, das wäre auch die Argumentation für
Gleichungen.)
Ich bleibe erstmal beim Gleichungsbild.
x + e^x = 1 ist eine algebraische Gleichung in x und e^x, also in den
Unbekannten(?)/Transzendenten(?) x und e^x. Wenn die Unbekannten einer
algebraischen Gleichung alle algebraisch abhängig sind, dann kann die
Gleichung in eine algebraische Gleichung einer einzigen Unbekannten
transformiert werden. Und solche Gleichung ist elementar auflösbar -
wenn die impliziten algebraischen Funktionen zu den Elementaren
Funktionen dazugezählt werden. (Mit Ritts Methode: Die Umkehrfunktion
einer algebraischen (einstelligen - IV) Funktion ist wieder eine solche,
und solche Funktionen gehören zu den Elementaren Funktionen.)
2.) x + e^x = 0
x + e^x = 1 ist eine algebraische Gleichung in x und e^x, also in den
Unbekannten(?)/Transzendenten(?) x und e^x. x und e^x sind algebraisch
unabhängig für alle x <> 0. x = 0 ist aber keine Lösung der Gleichung.
Die Gleichung kann nicht in eine algebraische Gleichung mit nur
algebraischen Unbekannten transformiert werden. Die äußere algebraische
Funktion der Gleichung (die algebraische Funktion mit der
Abbildungsvorschrift (x,e^x) \mapsto "+"(x,e^x) = x + e^x) ist
zweistellig, kann also, wie aus Ritts Satz folgt, keine Elementare
Funktion als Umkehrfunktion haben.
Ich versuche nun das Ergebnis der Anwendung von Ritts Satz auf diese
Gleichung zu verstehen.
Durch keine elementaren Umkehrfunktionen der in der Gleichung
vertretenen elmentaren Funktionen kann die Gleichung nach x aufgelöst
werden. Warum kann sie aber nicht durch irgendwelche anderen, also nicht
in der Gleichung "steckenden" elementaren Funktionen nach x aufgelöst
werden?
Das ist doch die alles entscheidende Frage.
Kann jemand helfen, beginnen, eine Antwort auf diese Frage zu finden?
Da kann ich dir aber mehr Funktionswerte für die Umkehrfunktion geben:
Folgende Tabelle zeigt die Werte von F^-1(x) für -2 <= x <= 2 in
Schritten von 0.01, so dass für
F: IR -> IR, x |-> x + e^x gilt:
F(F^-1(x)) = x

x F^-1(x)
==============================
-2.000000 -2.120028
-1.990000 -2.111104
-1.980000 -2.102189
-1.970000 -2.093282
-1.960000 -2.084384
-1.950000 -2.075494
-1.940000 -2.066614
-1.930000 -2.057742
-1.920000 -2.048879
-1.910000 -2.040025
-1.900000 -2.031181
-1.890000 -2.022345
-1.880000 -2.013518
-1.870000 -2.004701
-1.860000 -1.995892
-1.850000 -1.987093
-1.840000 -1.978304
-1.830000 -1.969523
-1.820000 -1.960752
-1.810000 -1.951991
-1.800000 -1.943239
-1.790000 -1.934497
-1.780000 -1.925764
-1.770000 -1.917041
-1.760000 -1.908328
-1.750000 -1.899625
-1.740000 -1.890931
-1.730000 -1.882248
-1.720000 -1.873574
-1.710000 -1.864910
-1.700000 -1.856256
-1.690000 -1.847613
-1.680000 -1.838980
-1.670000 -1.830356
-1.660000 -1.821744
-1.650000 -1.813141
-1.640000 -1.804549
-1.630000 -1.795967
-1.620000 -1.787396
-1.610000 -1.778835
-1.600000 -1.770285
-1.590000 -1.761745
-1.580000 -1.753216
-1.570000 -1.744698
-1.560000 -1.736190
-1.550000 -1.727694
-1.540000 -1.719208
-1.530000 -1.710733
-1.520000 -1.702269
-1.510000 -1.693817
-1.500000 -1.685375
-1.490000 -1.676944
-1.480000 -1.668525
-1.470000 -1.660117
-1.460000 -1.651720
-1.450000 -1.643334
-1.440000 -1.634960
-1.430000 -1.626597
-1.420000 -1.618246
-1.410000 -1.609906
-1.400000 -1.601578
-1.390000 -1.593262
-1.380000 -1.584957
-1.370000 -1.576663
-1.360000 -1.568382
-1.350000 -1.560112
-1.340000 -1.551855
-1.330000 -1.543609
-1.320000 -1.535375
-1.310000 -1.527153
-1.300000 -1.518943
-1.290000 -1.510745
-1.280000 -1.502560
-1.270000 -1.494386
-1.260000 -1.486225
-1.250000 -1.478076
-1.240000 -1.469939
-1.230000 -1.461815
-1.220000 -1.453703
-1.210000 -1.445604
-1.200000 -1.437517
-1.190000 -1.429442
-1.180000 -1.421381
-1.170000 -1.413331
-1.160000 -1.405295
-1.150000 -1.397271
-1.140000 -1.389260
-1.130000 -1.381261
-1.120000 -1.373276
-1.110000 -1.365303
-1.100000 -1.357344
-1.090000 -1.349397
-1.080000 -1.341463
-1.070000 -1.333542
-1.060000 -1.325634
-1.050000 -1.317740
-1.040000 -1.309858
-1.030000 -1.301990
-1.020000 -1.294135
-1.010000 -1.286293
-1.000000 -1.278465
-0.990000 -1.270649
-0.980000 -1.262847
-0.970000 -1.255059
-0.960000 -1.247284
-0.950000 -1.239522
-0.940000 -1.231774
-0.930000 -1.224040
-0.920000 -1.216319
-0.910000 -1.208612
-0.900000 -1.200918
-0.890000 -1.193238
-0.880000 -1.185571
-0.870000 -1.177919
-0.860000 -1.170280
-0.850000 -1.162655
-0.840000 -1.155044
-0.830000 -1.147446
-0.820000 -1.139863
-0.810000 -1.132293
-0.800000 -1.124738
-0.790000 -1.117196
-0.780000 -1.109668
-0.770000 -1.102155
-0.760000 -1.094655
-0.750000 -1.087170
-0.740000 -1.079698
-0.730000 -1.072241
-0.720000 -1.064798
-0.710000 -1.057369
-0.700000 -1.049954
-0.690000 -1.042553
-0.680000 -1.035167
-0.670000 -1.027795
-0.660000 -1.020437
-0.650000 -1.013094
-0.640000 -1.005765
-0.630000 -0.998450
-0.620000 -0.991150
-0.610000 -0.983864
-0.600000 -0.976592
-0.590000 -0.969335
-0.580000 -0.962093
-0.570000 -0.954864
-0.560000 -0.947651
-0.550000 -0.940452
-0.540000 -0.933267
-0.530000 -0.926097
-0.520000 -0.918941
-0.510000 -0.911800
-0.500000 -0.904674
-0.490000 -0.897562
-0.480000 -0.890465
-0.470000 -0.883382
-0.460000 -0.876314
-0.450000 -0.869261
-0.440000 -0.862223
-0.430000 -0.855199
-0.420000 -0.848189
-0.410000 -0.841195
-0.400000 -0.834215
-0.390000 -0.827250
-0.380000 -0.820300
-0.370000 -0.813364
-0.360000 -0.806443
-0.350000 -0.799537
-0.340000 -0.792646
-0.330000 -0.785769
-0.320000 -0.778907
-0.310000 -0.772060
-0.300000 -0.765228
-0.290000 -0.758410
-0.280000 -0.751608
-0.270000 -0.744820
-0.260000 -0.738047
-0.250000 -0.731288
-0.240000 -0.724545
-0.230000 -0.717816
-0.220000 -0.711102
-0.210000 -0.704403
-0.200000 -0.697719
-0.190000 -0.691050
-0.180000 -0.684395
-0.170000 -0.677755
-0.160000 -0.671130
-0.150000 -0.664520
-0.140000 -0.657925
-0.130000 -0.651344
-0.120000 -0.644779
-0.110000 -0.638228
-0.100000 -0.631692
-0.090000 -0.625170
-0.080000 -0.618664
-0.070000 -0.612172
-0.060000 -0.605695
-0.050000 -0.599233
-0.040000 -0.592785
-0.030000 -0.586353
-0.020000 -0.579935
-0.010000 -0.573532
0.000000 -0.567143
0.010000 -0.560770
0.020000 -0.554411
0.030000 -0.548066
0.040000 -0.541737
0.050000 -0.535422
0.060000 -0.529122
0.070000 -0.522837
0.080000 -0.516566
0.090000 -0.510310
0.100000 -0.504068
0.110000 -0.497841
0.120000 -0.491629
0.130000 -0.485432
0.140000 -0.479249
0.150000 -0.473080
0.160000 -0.466926
0.170000 -0.460787
0.180000 -0.454662
0.190000 -0.448552
0.200000 -0.442456
0.210000 -0.436375
0.220000 -0.430308
0.230000 -0.424256
0.240000 -0.418218
0.250000 -0.412195
0.260000 -0.406186
0.270000 -0.400192
0.280000 -0.394211
0.290000 -0.388246
0.300000 -0.382294
0.310000 -0.376357
0.320000 -0.370434
0.330000 -0.364526
0.340000 -0.358632
0.350000 -0.352752
0.360000 -0.346886
0.370000 -0.341034
0.380000 -0.335197
0.390000 -0.329374
0.400000 -0.323565
0.410000 -0.317770
0.420000 -0.311989
0.430000 -0.306223
0.440000 -0.300470
0.450000 -0.294732
0.460000 -0.289007
0.470000 -0.283296
0.480000 -0.277600
0.490000 -0.271917
0.500000 -0.266249
0.510000 -0.260594
0.520000 -0.254953
0.530000 -0.249326
0.540000 -0.243713
0.550000 -0.238113
0.560000 -0.232528
0.570000 -0.226956
0.580000 -0.221398
0.590000 -0.215853
0.600000 -0.210323
0.610000 -0.204806
0.620000 -0.199302
0.630000 -0.193812
0.640000 -0.188336
0.650000 -0.182874
0.660000 -0.177424
0.670000 -0.171989
0.680000 -0.166567
0.690000 -0.161158
0.700000 -0.155762
0.710000 -0.150381
0.720000 -0.145012
0.730000 -0.139657
0.740000 -0.134315
0.750000 -0.128986
0.760000 -0.123671
0.770000 -0.118369
0.780000 -0.113080
0.790000 -0.107804
0.800000 -0.102541
0.810000 -0.097291
0.820000 -0.092055
0.830000 -0.086832
0.840000 -0.081621
0.850000 -0.076424
0.860000 -0.071239
0.870000 -0.066068
0.880000 -0.060909
0.890000 -0.055763
0.900000 -0.050630
0.910000 -0.045510
0.920000 -0.040403
0.930000 -0.035308
0.940000 -0.030226
0.950000 -0.025157
0.960000 -0.020100
0.970000 -0.015056
0.980000 -0.010025
0.990000 -0.005006
1.000000 0.000000
1.010000 0.004994
1.020000 0.009975
1.030000 0.014944
1.040000 0.019900
1.050000 0.024844
1.060000 0.029776
1.070000 0.034696
1.080000 0.039603
1.090000 0.044498
1.100000 0.049380
1.110000 0.054251
1.120000 0.059109
1.130000 0.063955
1.140000 0.068789
1.150000 0.073611
1.160000 0.078422
1.170000 0.083220
1.180000 0.088006
1.190000 0.092780
1.200000 0.097542
1.210000 0.102293
1.220000 0.107031
1.230000 0.111758
1.240000 0.116473
1.250000 0.121176
1.260000 0.125868
1.270000 0.130548
1.280000 0.135216
1.290000 0.139873
1.300000 0.144518
1.310000 0.149151
1.320000 0.153773
1.330000 0.158384
1.340000 0.162983
1.350000 0.167571
1.360000 0.172147
1.370000 0.176712
1.380000 0.181266
1.390000 0.185808
1.400000 0.190340
1.410000 0.194860
1.420000 0.199368
1.430000 0.203866
1.440000 0.208353
1.450000 0.212828
1.460000 0.217292
1.470000 0.221746
1.480000 0.226188
1.490000 0.230620
1.500000 0.235040
1.510000 0.239450
1.520000 0.243849
1.530000 0.248237
1.540000 0.252614
1.550000 0.256980
1.560000 0.261336
1.570000 0.265681
1.580000 0.270015
1.590000 0.274339
1.600000 0.278652
1.610000 0.282955
1.620000 0.287247
1.630000 0.291528
1.640000 0.295799
1.650000 0.300060
1.660000 0.304310
1.670000 0.308550
1.680000 0.312780
1.690000 0.316999
1.700000 0.321208
1.710000 0.325407
1.720000 0.329595
1.730000 0.333773
1.740000 0.337942
1.750000 0.342100
1.760000 0.346248
1.770000 0.350385
1.780000 0.354513
1.790000 0.358631
1.800000 0.362739
1.810000 0.366837
1.820000 0.370925
1.830000 0.375003
1.840000 0.379072
1.850000 0.383131
1.860000 0.387179
1.870000 0.391218
1.880000 0.395248
1.890000 0.399268
1.900000 0.403278
1.910000 0.407278
1.920000 0.411269
1.930000 0.415250
1.940000 0.419222
1.950000 0.423184
1.960000 0.427137
1.970000 0.431081
1.980000 0.435015
1.990000 0.438939
2.000000 0.442854

Für jeden Wert x aus IR gibt es so einem Funktionswert F^-1(x)
eindeutig ( x + e^x ist streng monoton wachsend ).

Ich verstehe nicht, warum du nicht dieser Umkehrfunktion
einen eigenen Namen gibst (Vorschlag: IV(x)) und sie in die
Menge der "Elementaren Funktionen" aufnimmst.
Denn warum sollten die Funktion exp(x) oder ln(x) gerade
bevorzugt als "elementar" gekennzeichnet werden und IV(x) nicht?
Diese Scheuklappenbeschränkungen sind doch nur olle historische
Kamellen, weil man früher Funktionen, die Gleichungen lösen
sollten, in Tabellenwerken wiederfinden wollte.
Heutzutage ist man aus diesem Kinderalter herausgewachsen und
arbeitet eher an Existenz- und Eindeutigkeitsbeweisen statt
an der suche nach inhaltslosen Darstellungsexzessen.

Analog kannst du auch für 0<=c<1 eine Funktion

Kepler: [0, c) x IR -> IR

definieren, die die Kepler-Gleichung y - c*sin(y) = x
eindeutig löst, so daß

Kepler(c, x) - c*sin( Kepler(c, x) ) = x gilt,
und sie als "elementar" ansehen.
b***@gmail.com
2017-02-20 15:09:32 UTC
Permalink
Schauen wir uns einmal die andere Gleichung an:

x + e^x = d (1)

Die Funktion gibt es schon. Die lambert W funktion
löst die Gleichung:

z*e^z = c (2)

Via:

z = W(c) (3)

Nubstitutieren wir x = ln(z) in (1), erhalten wir:

d = ln(z) + e^ln(z)

= ln(z) + z (4)

Wenden wir die exponential Funktion auf
beiden Seiten an in (4), erhalten wir:

e^d = z * e^z (5)

Oder oder aus (2) und (3):

W(e^d) = z (6)

Und deshalb:

x = ln(W(e^d))

= d - W(e^d) (7)

Letzteres erhält man wenn man ln auf (2)
anwendet und (3) berücksichtigt:

ln(z*e^z) = ln(z) + z

= ln(c)

Deshalb:

ln(z) = ln(c)

Oder:

ln(W(c)) = ln(c) - W(c) (8)

Wolfram Alpha kennt das auch:
https://www.wolframalpha.com/input/?i=x%2Be^x+%3D+c
Post by Torn Rumero DeBrak
Ich verstehe nicht, warum du nicht dieser Umkehrfunktion
einen eigenen Namen gibst (Vorschlag: IV(x)) und sie in die
b***@gmail.com
2017-02-20 15:13:07 UTC
Permalink
Post by b***@gmail.com
ln(z) = ln(c)
Corr.:
ln(z) = ln(c) - z
Torn Rumero DeBrak
2017-02-20 15:55:47 UTC
Permalink
Post by b***@gmail.com
Post by b***@gmail.com
ln(z) = ln(c)
ln(z) = ln(c) - z
Hab es leider zu spät gesehen.
b***@gmail.com
2017-02-20 15:22:04 UTC
Permalink
Also x + e^x = c, löst das dumme Ding wohl richtig
mit x = c - W(e^c). Aber folgende Gleichung

x + x^a = b

Kann es nicht lösen, woran liegt das?
https://www.wolframalpha.com/input/?i=x%2Bx^a+%3D+b

Obwohl dieses Trinomial auch auf Euler/Lambert zurück geht:
http://math.stackexchange.com/questions/1989034/what-common-developments-are-there-to-solve-1x-xa/2002272#2002272
Post by b***@gmail.com
https://www.wolframalpha.com/input/?i=x%2Be^x+%3D+c
IV
2017-02-21 22:12:01 UTC
Permalink
Also x + e^x = c, löst das dumme Ding wohl richtig mit x = c - W(e^c).
Aber folgende Gleichung
x + x^a = b
kann es nicht lösen, woran liegt das?
https://www.wolframalpha.com/input/?i=x%2Bx^a+%3D+b
Du mußt ihm noch Solve und ,x befehlen. Und dann fehlt noch die Angabe von a
oder b.
Von a hängt ab, ob die Gleichung eine algebraische oder eine transzendente
Gleichung ist.

Obwohl dieses Trinomial auch auf Euler/Lambert zurück geht:
http://math.stackexchange.com/questions/1989034/what-common-developments-are-there-to-solve-1x-xa/2002272#2002272
Und hier hängt ebenfalls von a ab, ob die Gleichung eine algebraische oder
eine transzendente Gleichung ist.

Mit Hilfe von Ritts Satz in [Ritt 1925] kann man an nicht zu komplexen
Funktionstermen _sofort_ ablesen, ob eine gegebene bijektive Elementare
Funktion elementar umkehrbar ist oder nicht.
Und Ritts Satz beantwortet auch die Frage nach der Lösbarkeit einer in
Termen Elementarer Funktionen gegebenen Gleichung.
Gleichung so umformen, daß auf der rechten Seite nur eine Konstante steht.
Links steht dann eine Elementare Funktion. Wenn für die keine "lineare"
Verkettungsform aus Ritts Satz exisitiert, dann ist die Gleichung nicht
durch eine Elementare Funktion auflösbar.

Aber warum willst Du partout eine symbolische Lösung? Das ist doch
klassischer Ballast und überholt. Berechne doch die Lösungen schick modern
numerisch. Ich wünsche Dir viel Zeit dafür, wenn der Parameter nicht gegeben
ist.
Torn Rumero DeBrak
2017-02-20 15:54:35 UTC
Permalink
Post by b***@gmail.com
x + e^x = d (1)
Die Funktion gibt es schon. Die lambert W funktion
z*e^z = c (2)
z = W(c) (3)
d = ln(z) + e^ln(z)
= ln(z) + z (4)
Wenden wir die exponential Funktion auf
e^d = z * e^z (5)
W(e^d) = z (6)
x = ln(W(e^d))
= d - W(e^d) (7)
Letzteres erhält man wenn man ln auf (2)
ln(z*e^z) = ln(z) + z
= ln(c)
ln(z) = ln(c)
^^^^^^^^^^^^^^^^^
ln(z) = ln(c) - z
Post by b***@gmail.com
ln(W(c)) = ln(c) - W(c) (8)
https://www.wolframalpha.com/input/?i=x%2Be^x+%3D+c
Prima gemacht. Leider erkennt Nummer 4 die Lambertsche W-Funktion
nicht als elementar an, weil es da so einen Aufsatz gibt.
b***@gmail.com
2017-02-20 16:44:39 UTC
Permalink
Von dem Herr Ritt gibts jede Menge Aufsätze. Hier
drin hats nochmal zwei erwähnt, die nicht direct
exp und log angehen:

Building counterexamples to generalizations
for rational functions of Ritt’s decomposition
Theorem, Jaime Gutierrez, David Sevilla - 2006
http://grupos.unican.es/amac/articles/ritt-rac-LAST.pdf

Disclaimer ich vermute einen Zusammenhang zwischen
alledem, habs aber nicht vertieft bis jetzt. Aber
hier einfaches Beispiel einer meromorphen

function, die aus exp(-x) ein exp(x) macht, die
meromorphe Function ist das Reziproke:

exp(x) = 1/exp(-x)

Aber ich kenne gerade kein endliches Polynom P,
sodass exp(x) = P(exp(-x)).
Post by Torn Rumero DeBrak
Prima gemacht. Leider erkennt Nummer 4 die Lambertsche W-Funktion
nicht als elementar an, weil es da so einen Aufsatz gibt.
IV
2017-02-21 21:45:33 UTC
Permalink
Von dem Herr Ritt gibts jede Menge Aufsätze. Hier drin hats nochmal zwei
Building counterexamples to generalizations for rational functions of Ritt’s
decomposition Theorem, Jaime Gutierrez, David Sevilla - 2006
http://grupos.unican.es/amac/articles/ritt-rac-LAST.pdf
Disclaimer ich vermute einen Zusammenhang zwischen alledem, habs aber
nicht vertieft bis jetzt.
Ritt ist ja u. a. berühmt für sein Decompositions-Theorem für Polynome und
sein Buch "Differentialalgebra".
Ich sehe aber keinen Zusammenhang mit seinen Elementaren Umkehrfunktionen
von 1925.
IV
2017-02-21 21:32:09 UTC
Permalink
Post by IV
was ist die g e n a u e Ursache dafür, daß Gleichungen wie
y + e^y = x
und die Kepler-Gleichung
y - c*sin(y) = x, c eine Konstante,
keine Elementare Funktion y (siehe Wikipedia en: Elementary function)
als Lösung haben können?
Folgende Tabelle zeigt die Werte von F^-1(x) für -2 <= x <= 2 in Schritten
F(F^-1(x)) = x
...
Ich verstehe nicht, warum du nicht dieser Umkehrfunktion einen eigenen
Namen gibst (Vorschlag: IV(x)) und sie in die Menge der "Elementaren
Funktionen" aufnimmst.
Dann kann man jede beliebige Funktion in die Menge der "Elementaren
Funktionen" aufnehmen, und damit auch jede.
Wir hatten hier ja schon in einem anderen Thread festgestellt, daß F^-1(x) =
x - LambertW(e^x).
Es wird eben immer wieder gefragt: "Warum kann die und die gegebene
Gleichung nicht allein durch Anwenden Elementarer Funktionen aufgelöst
werden?"
Denn warum sollten die Funktion exp(x) oder ln(x) gerade bevorzugt als
"elementar" gekennzeichnet werden und IV(x) nicht?
Der Begriff der "Elementaren Funktionen" ist ja historisch entstanden. Und
Liouvilles Verdienst ist es, gezeigt zu haben, daß genau diese
Funktionenklasse durch exp, ln und/oder algebraische Funktionen dargestellt
werden kann.
Liouville hat auch gezeigt, wie man weitere Standardfunktionen zur Menge der
Elementaren Standardfunktionen hinzunimmt und dadurch die Funktionenklasse
erweitert (Erweiterungskörper).
Diese Scheuklappenbeschränkungen sind doch nur olle historische Kamellen,
weil man früher Funktionen, die Gleichungen lösen sollten, in
Tabellenwerken wiederfinden wollte.
Heutzutage ist man aus diesem Kinderalter herausgewachsen und arbeitet
eher an Existenz- und Eindeutigkeitsbeweisen statt an der suche nach
inhaltslosen Darstellungsexzessen.
Funktionen in geschlossenen Ausdrücken sind nutzlos.
Am besten, man behandelt alle Funktionen nur noch numerisch, nicht mehr
analytisch, und nicht mehr algebraisch.
Und Liouvilles Methode der Symbolischen Integration (Integration in finite
terms, Integration in endlichen/geschlossenen Ausdrücken) ist auch nutzlos.
Ich habe mal gehört, Mathematiker seien Leute, die (wie Künstler) ein
mathematisches Problem lösen bloß weil es da ist - ganz ohne irgendeinen
Nutzen.
Torn Rumero DeBrak
2017-02-21 22:18:44 UTC
Permalink
Post by IV
Post by Torn Rumero DeBrak
Post by IV
was ist die g e n a u e Ursache dafür, daß Gleichungen wie
y + e^y = x
und die Kepler-Gleichung
y - c*sin(y) = x, c eine Konstante,
keine Elementare Funktion y (siehe Wikipedia en: Elementary
function) als Lösung haben können?
Folgende Tabelle zeigt die Werte von F^-1(x) für -2 <= x <= 2 in
F(F^-1(x)) = x
...
Ich verstehe nicht, warum du nicht dieser Umkehrfunktion einen eigenen
Namen gibst (Vorschlag: IV(x)) und sie in die Menge der "Elementaren
Funktionen" aufnimmst.
Dann kann man jede beliebige Funktion in die Menge der "Elementaren
Funktionen" aufnehmen, und damit auch jede.
Wir hatten hier ja schon in einem anderen Thread festgestellt, daß
F^-1(x) = x - LambertW(e^x).
Es wird eben immer wieder gefragt: "Warum kann die und die gegebene
Gleichung nicht allein durch Anwenden Elementarer Funktionen aufgelöst
werden?"
WER fragt das??? Der fragende zeigt durch diese Frage doch nur, dass
er eine eingeschränkte Vorstellung vom Begriff "elementare
Funktion" hat.
Post by IV
Post by Torn Rumero DeBrak
Denn warum sollten die Funktion exp(x) oder ln(x) gerade bevorzugt als
"elementar" gekennzeichnet werden und IV(x) nicht?
Der Begriff der "Elementaren Funktionen" ist ja historisch entstanden.
Korrekt.
Post by IV
Und Liouvilles Verdienst ist es, gezeigt zu haben, daß genau diese
Funktionenklasse durch exp, ln und/oder algebraische Funktionen
dargestellt werden kann.
Nein. Liouville hat willkürlich die exp und ln Funktionen in seine
Klasse der "elementaren Funktionen" aufgenommen, weil er dann zeigen
konnte, dass einige andere populäre Funktionen (NICHT ALLE Funktionen!)
darauf zurückzuführen sind und man nicht spezielle Tabellenwerke für
die Funktionen veröffentlichen muss.
Post by IV
Liouville hat auch gezeigt, wie man weitere Standardfunktionen zur Menge
der Elementaren Standardfunktionen hinzunimmt und dadurch die
Funktionenklasse erweitert (Erweiterungskörper).
Post by Torn Rumero DeBrak
Diese Scheuklappenbeschränkungen sind doch nur olle historische
Kamellen, weil man früher Funktionen, die Gleichungen lösen sollten,
in Tabellenwerken wiederfinden wollte.
Heutzutage ist man aus diesem Kinderalter herausgewachsen und arbeitet
eher an Existenz- und Eindeutigkeitsbeweisen statt an der suche nach
inhaltslosen Darstellungsexzessen.
Funktionen in geschlossenen Ausdrücken sind nutzlos.
Jetzt einmal ganz provokativ gefragt:
Man braucht sie doch heutzutage wirklich nicht mehr, oder?
Der geschlossene Ausdruck für eine Funktion ist F, wenn ihre
Existenz und Eindeutigkeit gezeigt ist.
Für exp wäre die definierende Eigenschaft: F´ = F mit F(0) = 1.

Es lebe die Offenheit!
Post by IV
Am besten, man behandelt alle Funktionen nur noch numerisch, nicht mehr
analytisch, und nicht mehr algebraisch.
Gut. Warum nicht? Für die Praxis ist das doch ausreichend.
Post by IV
Und Liouvilles Methode der Symbolischen Integration (Integration in
finite terms, Integration in endlichen/geschlossenen Ausdrücken) ist
auch nutzlos.
Richtig, wer will sowas denn noch machen? Nicht umsonst versucht
man doch, diese stupide Arbeit dem Computer aufzubürden.
Post by IV
Ich habe mal gehört, Mathematiker seien Leute, die (wie Künstler) ein
mathematisches Problem lösen bloß weil es da ist - ganz ohne irgendeinen
Nutzen.
Da musst du aber etwas ganz gehörig missverstanden haben, oder man
wollte dich aufziehen.

Der Nutzen kann zwar momentan nicht sichtbar sein, dass der Künstler
aber an einen zukünftigen Nutzen nicht glauben sollte, ist doch reine
Unterstellung.
Am Beispiel der Differentialgeometrie und ihrer späteren Anwendung in
der Relativitätstheorie, oder an der Knotentheorie
und ihrer Anwendungen in der Chemie und Biologie (mit Irrwegen von
Lord Kelvin) siehst du doch, dass ein späterer Nutzen nicht
auszuschließen ist.
IV
2017-02-21 23:06:18 UTC
Permalink
Post by IV
Es wird eben immer wieder gefragt: "Warum kann die und die gegebene
Gleichung nicht allein durch Anwenden Elementarer Funktionen aufgelöst
werden?"
WER fragt das??? Der fragende zeigt durch diese Frage doch nur, dass er
eine eingeschränkte Vorstellung vom Begriff "elementare Funktion" hat.
Na immerhin hat er eine _Vorstellung_ vom Begriff "elementare Funktion" - im
Gegensatz zu Dir.
Post by IV
Denn warum sollten die Funktion exp(x) oder ln(x) gerade bevorzugt als
"elementar" gekennzeichnet werden und IV(x) nicht?
Der Begriff der "Elementaren Funktionen" ist ja historisch entstanden.
Post by IV
Und Liouvilles Verdienst ist es, gezeigt zu haben, daß genau diese
Funktionenklasse durch exp, ln und/oder algebraische Funktionen
dargestellt werden kann.
Nein. Liouville hat willkürlich die exp und ln Funktionen in seine Klasse
der "elementaren Funktionen" aufgenommen, weil er dann zeigen konnte, dass
einige andere populäre Funktionen (NICHT ALLE Funktionen!)
darauf zurückzuführen sind und man nicht spezielle Tabellenwerke für die
Funktionen veröffentlichen muss.
Dein "Nein" paßt nicht. Ich behaupte doch nur, daß es ein glücklicher
Umstand ist, daß genau die Funktionen der Schulmathematik, eben die
einfachen/elementaren Funktionen, so systematisch dargestellt werden können.
Post by IV
Denn warum sollten die Funktion exp(x) oder ln(x) gerade bevorzugt als
"elementar" gekennzeichnet werden und IV(x) nicht?
Diese Scheuklappenbeschränkungen sind doch nur olle historische
Kamellen, weil man früher Funktionen, die Gleichungen lösen sollten, in
Tabellenwerken wiederfinden wollte.
Heutzutage ist man aus diesem Kinderalter herausgewachsen und arbeitet
eher an Existenz- und Eindeutigkeitsbeweisen statt an der Suche nach
inhaltslosen Darstellungsexzessen.
Funktionen in geschlossenen Ausdrücken sind nutzlos.
Man braucht sie doch heutzutage wirklich nicht mehr, oder?
Der geschlossene Ausdruck für eine Funktion ist F, wenn ihre Existenz und
Eindeutigkeit gezeigt ist.
Für exp wäre die definierende Eigenschaft: F´ = F mit F(0) = 1.
Es lebe die Offenheit!
Post by IV
Am besten, man behandelt alle Funktionen nur noch numerisch, nicht mehr
analytisch, und nicht mehr algebraisch.
Gut. Warum nicht? Für die Praxis ist das doch ausreichend.
Wenn Funktionsgleichungen gegeben sind, kann man
- sofort Kurvendiskussionen machen,
- differenzieren,
- integrieren,
- damit rechnen und Funktionsgleichungen anderer Funktionen berechnen.
Rechnen mit Formeln dürfte oftmals sehr viel schneller gehen als jede
Funktion nur numerisch zu behandeln.
Computeralgebra braucht gar niemand - Numerikprogramme sind vollkommen
ausreichend.
Das ganze symbolische Rechnen mit Funktionen in Schule und Studium dient nur
der Übung. Man kann es auch weglassen. Denn wo soll man es denn später
anwenden, wenn alles nur noch numerisch berechnet wird.
Und wie ist es mit Gleichungen? Da rechnet man ja auch mit einer
Funktionsgleichung. Da will man dann auch nicht mehr?
Post by IV
Und Liouvilles Methode der Symbolischen Integration (Integration in
finite terms, Integration in endlichen/geschlossenen Ausdrücken) ist auch
nutzlos.
Richtig, wer will sowas denn noch machen? Nicht umsonst versucht man doch,
diese stupide Arbeit dem Computer aufzubürden.
Genau. Mathematik wird nicht mehr gebraucht. Das macht ja alles der
Computer.
Post by IV
Ich habe mal gehört, Mathematiker seien Leute, die (wie Künstler) ein
mathematisches Problem lösen bloß weil es da ist - ganz ohne irgendeinen
Nutzen.
Da musst du aber etwas ganz gehörig missverstanden haben, oder man wollte
dich aufziehen.
Naja, vielleicht war das früher mal so. Moderne Mathematiker müssen
wahrscheinlich immer Drittmittel erwirtschaften. Deshalb machen sie auch nur
noch Angewandte Mathematik. Na, Numerik eben. Ach nein, diese Arbeit
erledigen ja heute die Computer.
Post by IV
Der Nutzen kann zwar momentan nicht sichtbar sein, dass der Künstler aber
an einen zukünftigen Nutzen nicht glauben sollte, ist doch reine
Unterstellung.
Am Beispiel der Differentialgeometrie und ihrer späteren Anwendung in der
Relativitätstheorie, oder an der Knotentheorie und ihrer Anwendungen in
der Chemie und Biologie (mit Irrwegen von Lord Kelvin) siehst du doch,
dass ein späterer Nutzen nicht auszuschließen ist.
Er ist auszuschließen, wenn man keine Mathematik mehr betreibt.
Detlef Müller
2017-02-22 15:14:00 UTC
Permalink
[...]
Post by Torn Rumero DeBrak
Post by IV
Wir hatten hier ja schon in einem anderen Thread festgestellt, daß
F^-1(x) = x - LambertW(e^x).
Es wird eben immer wieder gefragt: "Warum kann die und die gegebene
Gleichung nicht allein durch Anwenden Elementarer Funktionen aufgelöst
werden?"
WER fragt das??? Der fragende zeigt durch diese Frage doch nur, dass
er eine eingeschränkte Vorstellung vom Begriff "elementare
Funktion" hat.
Ich halte die Untersuchung ob gewisse Funktionenklassen z.B. bzgl.
Inversenbildung abgeschlossen sind für durchaus interessant.

Wer Drittmittel einwerben will, mag sich eine praktische Anwendung
ausdenken (effektive Algorithmen zur Invertierung oder
dergleichen).

Zum Glück gibt es ja noch einen Rest wirklich freier Forschung,
wenngleich der wohl zunehmend eliminiert wird.

Der Aufwand für die Argumentation, warum das, was man in einer
Arbeit macht denn nun nützlich sei steigt zunehmend ...
einfach drauf los zu spielen ist inzwischen verpönt.
Post by Torn Rumero DeBrak
Post by IV
Denn warum sollten die Funktion exp(x) oder ln(x) gerade bevorzugt als
"elementar" gekennzeichnet werden und IV(x) nicht?
Der Begriff der "Elementaren Funktionen" ist ja historisch entstanden.
[...]
Post by Torn Rumero DeBrak
Nein. Liouville hat willkürlich die exp und ln Funktionen in seine
Klasse der "elementaren Funktionen" aufgenommen, weil er dann zeigen
konnte, dass einige andere populäre Funktionen (NICHT ALLE Funktionen!)
darauf zurückzuführen sind und man nicht spezielle Tabellenwerke für
die Funktionen veröffentlichen muss.
So willkürlich scheint mir das aber nicht gewesen zu sein.
Die exp und ln - Funktionen ergeben sich sehr natürlich in
der Funktionentheorie einer Veränderlicher. Eigentlich nur
die eine Funktion exp, aus der sich der ln ja ergibt.
Da sich sin, cos etc. damit (wenn man es zum ersten mal sieht)
erstaunlich elegant mit einer einzigen, sich natürlich ergebenden
Grundfunktion ausdrücken lassen, halte ich die Wahl keineswegs für
willkürlich.
Und die Tabellenwerke werden von den Überlegungen nicht sonderlich
profitiert haben, sicher haben die nicht im Vordergrund der Arbeit
gestanden (oder gibt es dazu Zitate?).
Post by Torn Rumero DeBrak
Post by IV
Diese Scheuklappenbeschränkungen sind doch nur olle historische
Kamellen, weil man früher Funktionen, die Gleichungen lösen sollten,
in Tabellenwerken wiederfinden wollte.
Heutzutage ist man aus diesem Kinderalter herausgewachsen und arbeitet
eher an Existenz- und Eindeutigkeitsbeweisen statt an der suche nach
inhaltslosen Darstellungsexzessen.
Genau das alte Tabellen-Argument gilt mindestens im selben Maße
noch heute, denn genau die "Darstellungsexzesse", die zu Zeiten
des Handrechnens ziemlich undurchführbar und von mehr theoretischem
Wert waren, können heute erst effektiv implementiert werden.

Gröbnerbasen hatte z.B. Hilbert (natürlich ohne die Bezeichnung zu
verwenden) verwendet, aber intensiv untersucht wurden sie erst viel
später, als sich heraus stellte, daß sie nicht nur eine Art Eierlegende
Wollmilchsau für eine Fülle theoretischer Probleme sind, sondern auch
mit Computerhilfe eine effektive Berechnungsmöglichkeit in praktischen
Anwendungen ergeben (ohne Rechner sind die Mengen der zwischendurch
entstehenden Daten außer in einfachen Beispielen kaum zu bewältigen).
Post by Torn Rumero DeBrak
Post by IV
Funktionen in geschlossenen Ausdrücken sind nutzlos.
Imo ist das Erheben der Nützlichkeit zur ersten Direktive
der Tod der wirklichen Wissenschaft.
Post by Torn Rumero DeBrak
Post by IV
Am besten, man behandelt alle Funktionen nur noch numerisch, nicht mehr
analytisch, und nicht mehr algebraisch.
Gut. Warum nicht? Für die Praxis ist das doch ausreichend.
Post by IV
Ich habe mal gehört, Mathematiker seien Leute, die (wie Künstler) ein
mathematisches Problem lösen bloß weil es da ist - ganz ohne irgendeinen
Nutzen.
Da musst du aber etwas ganz gehörig missverstanden haben, oder man
wollte dich aufziehen.
Ich finde das eigentlich gar nicht ehrenrührig. Natürlich gibt
es in der Mathematik einen kreativen und künstlerischen
Aspekt.
Post by Torn Rumero DeBrak
Der Nutzen kann zwar momentan nicht sichtbar sein, dass der Künstler
aber an einen zukünftigen Nutzen nicht glauben sollte, ist doch reine
Unterstellung.
Am Beispiel der Differentialgeometrie und ihrer späteren Anwendung in
der Relativitätstheorie, oder an der Knotentheorie
und ihrer Anwendungen in der Chemie und Biologie (mit Irrwegen von
Lord Kelvin) siehst du doch, dass ein späterer Nutzen nicht
auszuschließen ist.
Ja, traurig, daß man solche "Kollateralnutzen" inzwischen immer
wieder aufzählen muß um die "reine Mathematik" zu
rechtfertigen.

Allerdings war da gewiss nicht "früher alles besser" - viele
"Freigeister" waren ohne Geldsorgen ("von Geburt an reich" oder
sicher im Kloster), andere schafften es, sich neben dem rein
Nützlichen (bezahlten) Rechnen doch noch ihr Steckenpferd zu
pflegen.

Gruß,
Detlef
--
Dr. Detlef Müller,
http://www.mathe-doktor.de oder http://mathe-doktor.de
Jens Kallup
2017-02-22 18:00:06 UTC
Permalink
Post by Detlef Müller
Zum Glück gibt es ja noch einen Rest wirklich freier Forschung,
wenngleich der wohl zunehmend eliminiert wird.
Der Aufwand für die Argumentation, warum das, was man in einer
Arbeit macht denn nun nützlich sei steigt zunehmend ...
einfach drauf los zu spielen ist inzwischen verpönt.
ja, leider.
immer mehr Abhängigkeiten bezüglich commerz und server.
Aber ich denke, wenn sich vielleicht eine Gruppe zusammen finden
würde, dann könnte man einen gemeinsamen Eigenen Server aufmachen.
Da ich in dieser Hinsicht nicht so sehr an andere gebunden bin,
ist/besteht bei mir das Problem nicht.
Das Problem aber dürfte sein, eine Domain zu halten, die an die
IpV6 Nummer gebunden ist.
IV
2017-02-20 21:30:16 UTC
Permalink
Post by IV
Post by IV
Post by IV
was ist die g e n a u e Ursache dafür, daß Gleichungen wie
y + e^y = x
und die Kepler-Gleichung
y - c*sin(y) = x, c eine Konstante,
keine Elementare Funktion y (siehe Wikipedia en: Elementary function)
als Lösung haben können?
x + e^x = a oder x + sin(x) = b, mit a und b Konstanten.
Ich fange endlich einfach mal an, Teilantworten zu suchen
Heute endlich kam ich auf das für den Beweis noch fehlende Argument.
Ich skizziere den Beweis hier kurz.

Es ist noch viel Kauderwelsch in der Argumentation. Vielleicht kann/möchte
mir ja jemand helfen, die für einen Mathematiker doch recht einfach zu
formulierenden Herleitungen mathematisch korrekt zu machen.
Nach umfangreichen Literaturrecherchen bin ich zu dem Schluß gekommen, daß
die Herleitungen neu sind und eine Innovation darstellen.
Denjenigen, die wesentliche Beiträge liefern, biete ich die Mitautorschaft
an dem zu erstellenden Artikel in einer mathematischen Fachzeitschrift (in
Deutsch und in Englisch) an.

Die Begriffe Umkehrung einer Verkettung, Verkettung und
Verkettungsdarstellung müssen noch genauer definiert und präziser eingesetzt
werden.


Definition 1:
Eine Funktion ist genau dann elementar, also eine elementare Funktion, wenn
sie einstellig und einwertig ist, ihr Definitionsbereich ein komplexes
Intervall ist, und ihre Funktionswerte sich aus ihrem Argument in einer
endlichen Anzahl von Schritten allein durch algebraische Funktionen,
Exponentialfunktionen, Logarithmusfunktionen, Winkelfunktionen, inversen
Winkelfunktionen (= Arkusfunktionen), Hyperbelfunktionen und/oder inversen
Hyperbelfunktionen (= Areafunktionen) erzeugen lassen.


Satz 1:
Ist F eine elementare Funktion, dann sind die folgenden Aussagen äquivalent.
a) Die Funktionswerte von F lassen sich aus dem Argument von F in einer
endlichen Anzahl von Schritten allein durch algebraische Funktionen,
Exponentialfunktionen, Logarithmusfunktionen, Winkelfunktionen, inversen
Winkelfunktionen (= Arkusfunktionen), Hyperbelfunktionen und/oder inversen
Hyperbelfunktionen (= Areafunktionen) erzeugen.
b) Die Funktionswerte von F lassen sich aus dem Argument von F allein durch
endlich oftes Anwenden von algebraischen Funktionen, Exponentialfunktionen,
Logarithmusfunktionen, Winkelfunktionen, inversen Winkelfunktionen (=
Arkusfunktionen), Hyperbelfunktionen und/oder inversen Hyperbelfunktionen (=
Areafunktionen) erzeugen.
c) Die Funktionswerte von F lassen sich aus dem Argument von F allein durch
endlich oftes Anwenden von algebraischen Funktionen, exp und/oder ln
erzeugen.
d) Die Funktionswerte von F lassen sich aus dem Argument von F allein durch
endlich ofte Verkettung von algebraischen Funktionen, exp und/oder ln
erzeugen.
e) Die Funktionswerte von F lassen sich aus dem Argument von F allein durch
endlich ofte Verkettung von ein- oder mehrstelligen algebraischen
Funktionen, exp und/oder ln erzeugen.
f) F ist eine Verkettung endlich vieler Funktionen, von denen jede der Menge
{ein- oder mehrstellige algebraische Funktionen, exp, ln} entstammt.

Beweis von Satz 1:
Aussage a ist in der Definition 1 enthalten.
Aussage b ist Aussage a mit etwas anderen Worten.
Aussage c ergibt sich aus Aussage b dadurch, daß die Exponentialfunktionen,
Logarithmusfunktionen, Winkelfunktionen, Arkusfunktionen, Hyperbelfunktionen
und Areafunktionen sich allein durch algebraische Operationen sowie exp
und/oder ln darstellen lassen (siehe z. B. Abramowitz/Stegun), und jede
algebraische Operation eine algebraische Funktion ist. Aussage b ist in der
Definition der elementaren Funktionen aus [Ritt 1925] enthalten.
Aussage d ergibt sich aus Aussage c durch Benennen des Anwendens von
Funktionen als Verkettung.
Aussage e ergibt sich aus Aussage d dadurch, dass die Stelligkeit der
algebraischen Funktionen Erwähnung findet.
Aussage f ist Aussage e mit etwas anderen Worten.


Satz 2:
Eine Gleichung E(y) = x, worin E eine elementare Funktion, y die komplexe
Lösungsvariable und x eine komplexe Variable sind, hat genau dann eine
elementare Funktion y als Lösung, wenn die Funktion E eine elementare
Umkehrfunktion hat.

Beweis von Satz 2:
E^-1: Umkehrfunktion der Funktion E
Zuordnungsvorschrift der elementaren Funktion E: E(x) = F(x)
(1)
Definitionsgleichung der Umkehrfunktion E^-1: E(E^-1(x)) = x
(2)
Transformation y = E^-1(x)
(3)
Einsetzen von (2) in Gleichung (1): E(y) = x
(4)
Gleichung (4) ist die Gleichung aus Satz 2.
Verwenden von Gleichung (1) in Gleichung (4): F(y) =x
(5)
Auflösen von Gleichung (2) nach der Lösungsvariablen y: y = F^-1(x) (6)
Rücktransformation (3): E^-1(x) = F^-1(x)
(7)
Die Lösung der Gleichung aus Satz 2 ist genau die Umkehrfunktion E^-1.
Daraus folgt Satz 2.


Wir wollen jetzt Ritts Satz ([Ritt 1925]) beweisen.

Satz 3 (Satz über die Form elementar umkehrbarer elementarer Funktionen
(IV)) - Ritts Satz in etwas anderer Notation:
Wenn die Funktion F und ihre Umkehrfunktion beide elementar sind, dann
existieren n Funktionen F_1, F_2, ..., F_n, wo jedes F_i mit einem ungeraden
Index i eine algebraische Funktion ist, und jedes F_i mit einem geraden
Index i entweder exp oder ln ist, so dass F(z) =
F_n(F_{n-1}(...(F_2(F_1(z)))...)).

Beweis von Satz 3 (IV):
a) Satz 4 besagt, dass jede bijektive lineare expln-Verkettung elementar
umkehrbar ist.
b) Satz 5 besagt, dass keine nicht-lineare expln-Verkettung elementar
umkehrbar ist.
c) Aus a und b folgt, dass nur lineare expln-Verkettungen elementar
umkehrbar sind.
Aus c und dem Rittschen Konstruktionsprinzip der elementaren Funktionen
folgt die Formeldarstellung in Ritts Satz (Satz 3).


Satz 4 (IV):
Ist die Funktion F eine bijektive lineare expln-Verkettung, dann ist F
elementar umkehrbar.

Beweis von Satz 4:
Satz 4 ergibt sich daraus, dass die Umkehrfunktion einer bijektiven linearen
Verkettung ebenfalls eine lineare Verkettung ist.

In Erweiterung von [Ritt 1925] läßt sich auch sofort die Umkehrfunktion
einer bijektiven linearen Verkettung angeben. Das wäre ein weiterer Satz.


Satz 5 (IV):
Ist die Funktion F eine nicht-lineare expln-Verkettung, dann ist F nicht
elementar umkehrbar.

Beweis von Satz 5:
F ist gemäß Voraussetzung von Satz 5 eine bijektive nicht-lineare
expln-Verkettung.
Ist F nicht bijektiv, dann ist F nicht umkehrbar und damit nicht elementar
umkehrbar.
Ist F bijektiv, dann existiert die Umkehrfunktion zu F.
Die Umkehrung einer Verkettung ist die Verkettung der Umkehrfunktionen der
Komponenten der Verkettung in umgekehrter Reihenfolge. Da F eine
nicht-lineare Verkettung ist, enthält die expln-Verkettungsdarstellung von F
mindestens eine mehrstellige algebraische Funktion. Die Umkehrfunktion einer
mehrstelligen algebraischen Funktion ist eine mehrwertige Funktion. Eine
mehrwertige Funktion kann laut Definition der elementaren Funktionen
(Definition 1) jedoch keine elementare Funktion sein. Deshalb kann die
Umkehrung der Verkettungsdarstellung von F keine elementare Funktion sein.
Da F eine nicht-lineare expln-Verkettung ist, besitzt F keine lineare
Verkettungsdarstellung, sondern nur nicht-lineare Verkettungsdarstellungen.
Entsprechend dem im vorigen Absatz Gesagten kann keine dieser nicht-linearen
Verkettungsdarstellungen von F eine elementare Umkehrung haben. Da F aber
keine lineare Verkettungsdarstellung besitzt, kann keine der
Verkettungsdarstellungen der Umkehrfunktion von F eine elementare Funktion
sein. Die Umkehrfunktion von F kann daher keine elementare Funktion sein.


Diese Herleitungen beziehen sich auf die elementaren Funktionen nach Ritt.
Er zählt die impliziten algebraischen Funktionen mit zu den elementaren
Funktionen. Khovanskii nennt Ritts elementare Funktionen verallgemeinerte
elementare Funktionen.

Analoge Herleitungen sind auch für die Klasse der elementaren Funktionen
möglich, die sich von der Funktionenklasse Ritts darin unterscheidet, dass
nur die expliziten algebraischen Funktionen, nicht aber die impliziten
algebraischen Funktionen dazuzählen.

Weiter definiere ich die Klasse der standardfunktionbasierten Funktionen.

Definition (IV):
Eine Funktion ist genau dann eine Standardfunktion, wenn definiert ist, dass
sie eine Standardfunktion ist.

Definition (IV):
Eine Funktion ist genau dann standardfunktionbasiert, also eine
standardfunktionbasierte Funktion, wenn sie eine Verkettung endlich vieler
Funktionen ist, von denen jede der Menge {ein- oder mehrstellige
algebraische Funktionen, {einstellige einwertige Standardfunktionen}}
entstammt.

Satz (IV):
Wenn die Funktion F und ihre Umkehrfunktion beide standardfunktionbasiert
sind, dann ist F eine Verkettung endlich vieler Funktionen, von denen jede
der Menge {einstellige algebraische Funktionen, {einstellige einwertige
Standardfunktionen}} entstammt.


Begriffserklärungen
================

algebraische Funktion: ein- oder mehrstellige einwertige algebraische
Funktion

expln-Verkettung: Verkettungsdarstellung nach Satz 1 e

lineare expln-Verkettung: elementare Funktion, die eine
Verkettungsdarstellung nach Satz 1 e besitzt, deren algebraische Funktionen
sämtlich einstellig sind

nicht-lineare expln-Verkettung: elementare Funktion, die eine
Verkettungsdarstellung nach Satz 1 e besitzt die mindestens eine
mehrstellige algebraische Funktionen enthält, und die keine lineare
expln-Verkettung ist

Umkehrung einer Verkettung: ...

Verkettung: ...

Verkettungsdarstellung: ...



Literatur
=======

[Ritt 1925]: Ritt, J. F.: Elementary functions and their inverses. Trans.
Amer. Math. Soc. 27 (1925) (1) 68-90.
http://www.ams.org/journals/tran/1925-027-01/S0002-9947-1925-1501299-9
IV
2017-02-20 21:57:37 UTC
Permalink
Post by IV
Eine Funktion ist genau dann standardfunktionbasiert, also eine
standardfunktionbasierte Funktion, wenn sie eine Verkettung endlich vieler
Funktionen ist, von denen jede der Menge {ein- oder mehrstellige
algebraische Funktionen, {einstellige einwertige Standardfunktionen}}
entstammt.
Zur Definition gehört noch dazu, daß eine Standardfunktion F nur dann eine
standardfunktionbasierte Funktion ist, wenn die Umkehrfunktion von F
ebenfalls eine standardfunktionbasierte Funktion ist.
Post by IV
Wenn die Funktion F und ihre Umkehrfunktion beide standardfunktionbasiert
sind, dann ist F eine Verkettung endlich vieler Funktionen, von denen jede
der Menge {einstellige algebraische Funktionen, {einstellige einwertige
Standardfunktionen}} entstammt.
Ähnlich wie beim Satz von Ritt lassen sich hierzu wieder Sätze formulieren:

Satz (IV):
Ist die Funktion F eine bijektive lineare standardfunktionbasierte
Verkettung, dann ist F standardfunktionbasiert umkehrbar.

Satz (IV):
Ist die Funktion F eine nicht-lineare standardfunktionbasierte Verkettung,
dann ist F nicht standardfunktionbasiert umkehrbar.

Aus beiden Sätzen folgt, daß nur lineare standardfunktionbasierte
Verkettungen standardfunktionbasiert umkehrbar sind.

Mit der Klasse der standardfunktionbasierten Funktionen hat man eine sehr
allgemeine große Funktionenklasse zur Verfügung. Durch sie kommt man dem
Begriff "Funktionen in geschlossenen Ausdrücken" sehr viel näher.

Vielleicht kann man ja die Beschränkung auf die Einstelligkeit und
Einwertigkeit der Standardfunktionen in der Definition der
standarfunktionbasierten Funktionen weglassen und sie durch m_i-Stelligkeit
und n_i-Wertigkeit für die Standardfunktionen i, für alle i, ersetzen.
Jens Kallup
2017-02-20 22:42:12 UTC
Permalink
Post by IV
Vielleicht kann man ja die Beschränkung auf die Einstelligkeit und
Einwertigkeit der Standardfunktionen in der Definition der
standarfunktionbasierten Funktionen weglassen und sie durch
m_i-Stelligkeit und n_i-Wertigkeit für die Standardfunktionen i, für
alle i, ersetzen.
kommt drauf an.
Beachte:
n = natürliche Zahl
i = imaginäre Zahl -1

würde also lauten: 1^-1 = 1

was meinst, wie viele "i'es" es gibt?
IV
2017-02-21 17:37:12 UTC
Permalink
"IV" schrieb im Newsbeitrag news:o8fooi$j7r$***@news.albasani.net...
Die Sätze bei denen es um die Auflösbarkeit oder Umkehrbarkeit geht gelten
natürlich nur für mehrelementige Definitionsbereiche.
H0Iger SchuIz
2017-02-22 17:30:04 UTC
Permalink
Post by IV
Post by IV
Eine Funktion ist genau dann standardfunktionbasiert, also eine
standardfunktionbasierte Funktion, wenn sie eine Verkettung endlich vieler
Funktionen ist, von denen jede der Menge {ein- oder mehrstellige
algebraische Funktionen, {einstellige einwertige Standardfunktionen}}
entstammt.
Zur Definition gehört noch dazu, daß eine Standardfunktion F nur dann eine
standardfunktionbasierte Funktion ist, wenn die Umkehrfunktion von F
ebenfalls eine standardfunktionbasierte Funktion ist.
Wenn das zur Definition dazugehört, warum steht es denn da nicht?
Außerdem ist "standardfunktionbasierte Funktion" ein echter
Zungenbrecher.
Post by IV
Post by IV
Wenn die Funktion F und ihre Umkehrfunktion beide standardfunktionbasiert
sind, dann ist F eine Verkettung endlich vieler Funktionen, von denen jede
der Menge {einstellige algebraische Funktionen, {einstellige einwertige
Standardfunktionen}} entstammt.
Formulieren kann man immer viel, z.B. dass eine standardfunktionen
basierte Funktion keine Bananen mag. Ich weiß aber nicht, ob ich das
beweisen kann. Ich kann mir denken, dass Ritt bei den elementaren
Funktionen deren Eigenschaften im Beweis verwendet hat. Hier soll nun
von irgendwelchen Funktionen ausgegangen werden, die man dann verknüpft.
Da würde ich ganz gerne mal ein paar Beispiele durchlaufen lassen, bevor
ich mich zu irgendwelchen Behauptungen hinreißen ließe.

[...]
Post by IV
Mit der Klasse
Den bestimmten Artikel im Singular halte ich nicht für angemessen.
Immerhin gibt es für jede beliebige Menge von Standardfunktionen eine
solche Klasse.
Post by IV
der standardfunktionbasierten Funktionen hat man eine sehr
allgemeine große Funktionenklasse zur Verfügung. Durch sie kommt man dem
Begriff "Funktionen in geschlossenen Ausdrücken" sehr viel näher.
Näher als? Komparativ ohne Vergleich macht nicht so richtig Sinn.
Post by IV
Vielleicht kann man ja die Beschränkung auf die Einstelligkeit und
Einwertigkeit der Standardfunktionen in der Definition der
standarfunktionbasierten Funktionen weglassen und sie durch m_i-Stelligkeit
und n_i-Wertigkeit für die Standardfunktionen i, für alle i, ersetzen.
Verstehe ich nicht. Was sollen i, m_i und n_i hier sein?
Jens Kallup
2017-02-22 18:11:23 UTC
Permalink
Post by H0Iger SchuIz
Formulieren kann man immer viel, z.B. dass eine standardfunktionen
basierte Funktion keine Bananen mag.
hihi, der war gut ;-)
Was es für Funktionen gibt, die als Grundlage dienen dürften
habe ich auch bei meinen Notizen.pdf beigefügt - Link dürfte jeden
bekannt sein.
IV
2017-02-22 19:52:34 UTC
Permalink
Post by H0Iger SchuIz
Post by IV
Vielleicht kann man ja die Beschränkung auf die Einstelligkeit und
Einwertigkeit der Standardfunktionen in der Definition der
standardfunktionbasierten Funktionen weglassen und sie durch
m_i-Stelligkeit und n_i-Wertigkeit für die Standardfunktionen i, für alle
i, ersetzen.
Verstehe ich nicht. Was sollen i, m_i und n_i hier sein?
Beispiel:
Standardfunktion 1 ist m1-stellig und n1-wertig, Standardfunktion 2 ist
m2-stellig und n2-wertig
Beim Bestimmen der Umkehrfunktion der Verkettung dieser Funktionen muß man
Stelligkeit und Wertigkeit der Umkehrfunktionen der Gliedfunktionen
betrachten, also m1, n1, m2 und n2.
Jens Kallup
2017-02-20 22:01:44 UTC
Permalink
Da F aber keine lineare Verkettungsdarstellung besitzt, kann keine der
Verkettungsdarstellungen der Umkehrfunktion von F eine elementare
Funktion sein. Die Umkehrfunktion von F kann daher keine elementare
Funktion sein.
ich glaube jetzt hat Nummer vier es unbewusst geschrieben???
Und ja, würde mich freuen, wenn mein Postfach nicht immer leer ist und sich
der Weg dahin lohnt - bin ja auch nicht mehr der jüngste *hust ;-)
Adresse ist ja auf der ersten Seite der Notizen-Lektüre zu entnehmen.
IV
2017-02-21 22:18:18 UTC
Permalink
Post by Jens Kallup
Da F aber keine lineare Verkettungsdarstellung besitzt, kann keine der
Verkettungsdarstellungen der Umkehrfunktion von F eine elementare
Funktion sein. Die Umkehrfunktion von F kann daher keine elementare
Funktion sein.
ich glaube jetzt hat Nummer vier es unbewusst geschrieben???
Und ja, würde mich freuen, wenn mein Postfach nicht immer leer ist und
sich der Weg dahin lohnt - bin ja auch nicht mehr der jüngste *hust ;-)
Adresse ist ja auf der ersten Seite der Notizen-Lektüre zu entnehmen.
Ich verstehe überhaupt nicht was Du meinst.
Jens Kallup
2017-02-22 00:25:37 UTC
Permalink
Ich verstehe ÃŒberhaupt nicht was Du meinst.
hatte ich doch erwÀhnt, das ich Notizen von den einen oder
anderen Thema hier mache.
Du kannst diese aus dem Netz saugen. Hier der Link:
https://github.com/paule32/books
Carlos Naplos
2017-02-20 23:12:06 UTC
Permalink
Post by IV
Post by IV
Post by IV
Post by IV
was ist die g e n a u e Ursache dafür, daß Gleichungen wie
y + e^y = x
und die Kepler-Gleichung
y - c*sin(y) = x, c eine Konstante,
keine Elementare Funktion y (siehe Wikipedia en: Elementary
function) als Lösung haben können?
x + e^x = a oder x + sin(x) = b, mit a und b Konstanten.
Ich fange endlich einfach mal an, Teilantworten zu suchen
Heute endlich kam ich auf das für den Beweis noch fehlende Argument.
Ich skizziere den Beweis hier kurz.
Es ist noch viel Kauderwelsch in der Argumentation. Vielleicht
kann/möchte mir ja jemand helfen, die für einen Mathematiker doch recht
einfach zu formulierenden Herleitungen mathematisch korrekt zu machen.
Nach umfangreichen Literaturrecherchen bin ich zu dem Schluß gekommen,
daß die Herleitungen neu sind und eine Innovation darstellen.
Denjenigen, die wesentliche Beiträge liefern, biete ich die
Mitautorschaft an dem zu erstellenden Artikel in einer mathematischen
Fachzeitschrift (in Deutsch und in Englisch) an.
Die Begriffe Umkehrung einer Verkettung, Verkettung und
Verkettungsdarstellung müssen noch genauer definiert und präziser
eingesetzt werden.
Eine Funktion ist genau dann elementar, also eine elementare Funktion,
wenn sie einstellig und einwertig ist, ihr Definitionsbereich ein
komplexes Intervall ist, und ihre Funktionswerte sich aus ihrem Argument
in einer endlichen Anzahl von Schritten allein durch algebraische
Funktionen, Exponentialfunktionen, Logarithmusfunktionen,
Winkelfunktionen, inversen Winkelfunktionen (= Arkusfunktionen),
Hyperbelfunktionen und/oder inversen Hyperbelfunktionen (=
Areafunktionen) erzeugen lassen.
Ist F eine elementare Funktion, dann sind die folgenden Aussagen äquivalent.
a) Die Funktionswerte von F lassen sich aus dem Argument von F in einer
endlichen Anzahl von Schritten allein durch algebraische Funktionen,
Exponentialfunktionen, Logarithmusfunktionen, Winkelfunktionen, inversen
Winkelfunktionen (= Arkusfunktionen), Hyperbelfunktionen und/oder
inversen Hyperbelfunktionen (= Areafunktionen) erzeugen.
b) Die Funktionswerte von F lassen sich aus dem Argument von F allein
durch endlich oftes Anwenden von algebraischen Funktionen,
Exponentialfunktionen, Logarithmusfunktionen, Winkelfunktionen, inversen
Winkelfunktionen (= Arkusfunktionen), Hyperbelfunktionen und/oder
inversen Hyperbelfunktionen (= Areafunktionen) erzeugen.
c) Die Funktionswerte von F lassen sich aus dem Argument von F allein
durch endlich oftes Anwenden von algebraischen Funktionen, exp und/oder
ln erzeugen.
d) Die Funktionswerte von F lassen sich aus dem Argument von F allein
durch endlich ofte Verkettung von algebraischen Funktionen, exp und/oder
ln erzeugen.
e) Die Funktionswerte von F lassen sich aus dem Argument von F allein
durch endlich ofte Verkettung von ein- oder mehrstelligen algebraischen
Funktionen, exp und/oder ln erzeugen.
f) F ist eine Verkettung endlich vieler Funktionen, von denen jede der
Menge {ein- oder mehrstellige algebraische Funktionen, exp, ln} entstammt.
Aussage a ist in der Definition 1 enthalten.
Aussage b ist Aussage a mit etwas anderen Worten.
Aussage c ergibt sich aus Aussage b dadurch, daß die
Exponentialfunktionen, Logarithmusfunktionen, Winkelfunktionen,
Arkusfunktionen, Hyperbelfunktionen und Areafunktionen sich allein durch
algebraische Operationen sowie exp und/oder ln darstellen lassen (siehe
z. B. Abramowitz/Stegun), und jede algebraische Operation eine
algebraische Funktion ist. Aussage b ist in der Definition der
elementaren Funktionen aus [Ritt 1925] enthalten.
Aussage d ergibt sich aus Aussage c durch Benennen des Anwendens von
Funktionen als Verkettung.
Aussage e ergibt sich aus Aussage d dadurch, dass die Stelligkeit der
algebraischen Funktionen Erwähnung findet.
Aussage f ist Aussage e mit etwas anderen Worten.
Abgesehen von allem anderen fehlt hier für den Beweis der Äquivalenz der
sechs Aussagen, der Beweis, dass (@Jens: ja das Wort mit zwei "s",
früher mit einem "ß" gibt es in der deutschen Sprache ,-) aus f) wieder
a) folgt.

Gruß CN
Jens Kallup
2017-02-20 23:21:06 UTC
Permalink
Post by Carlos Naplos
Abgesehen von allem anderen fehlt hier für den Beweis der Äquivalenz
früher mit einem "ß" gibt es in der deutschen Sprache ,-) aus f)
wieder a) folgt.
hehe, hallo Carl!

Deutsche Land Sprache war noch nie so mein Ding.
Ich habe mich immer in der Verwendung von "statischen" Paradigmen und
Schlüsselworte wie bei Assembler das "MOV AX, 13" begnügt.

kleiner Schelm Du :-)
H0Iger SchuIz
2017-02-21 06:57:07 UTC
Permalink
Post by Carlos Naplos
Abgesehen von allem anderen fehlt hier für den Beweis der Äquivalenz der
sechs Aussagen, der Beweis, dass
[...]
Post by Carlos Naplos
aus f) wieder
a) folgt.
f) => a) gilt, weil es sich ja um eine elementare Funkton ahndelt.
Obacht, Nummer vier behauptet _nicht_ die Äquivalenz der Aussagen a) bis
f), sondern nur die viel schwächere Aussage, dass a) bus f) _für_
_elementare_ _Funktionen_ äquivalent ist. Damit erreicht sie natürlich
keine äquivalente Klassifikation der elementaren Funktion.

Der "Satz" hat in etwa die folgende Struktur:

"Für eine gerade Zahl n sind die folgenden Aussagen äquivalent:

a) n ist ruch drei teilbar.

b) b ist durch 6 teilbar."

Der Beweis ist einfach und wird dem Leser als Übung überlassen.

hs
IV
2017-02-21 22:39:53 UTC
Permalink
Post by Carlos Naplos
Abgesehen von allem anderen fehlt hier für den Beweis der Äquivalenz der
sechs Aussagen, der Beweis, dass [...] aus f) wieder a) folgt.
f) => a) gilt, weil es sich ja um eine elementare Funktion handelt.
Obacht, Nummer vier behauptet _nicht_ die Äquivalenz der Aussagen a) bis
f), sondern nur die viel schwächere Aussage, dass a) bus f) _für_
_elementare_ _Funktionen_ äquivalent ist. Damit erreicht sie natürlich
keine äquivalente Klassifikation der elementaren Funktion.
Alsooo. Ich wollte mir und Euch den Teil der Definition der Elementaren
Funktionen "Eine Funktion ist genau dann elementar, also eine elementare
Funktion, wenn sie einstellig und einwertig ist, ihr Definitionsbereich ein
komplexes Intervall ist, und" ersparen.
Und wenn ich das in jede Zeile von Satz 1 geschrieben hätte, dann hättet Ihr
doch bestimmt sofort geschrien: "Das ist kein Satz, das sind verschiedene
Definitionen ...", oder?
Warum wird mit meiner bisherigen Lösung keine äquivalente Klassifikation der
elementaren Funktion erreicht?
Eigentlich will ich die Äquivalenz(?) all dieser Definitionen zeigen. Oder
anders ausgedrückt, daß durch alle diese Definitionen dieselbe
Funktionenklasse definiert wird.
a) n ist durch drei teilbar.
b) b ist durch 6 teilbar."
Der Beweis ist einfach und wird dem Leser als Übung überlassen.
Ich habe gezeigt, daß aus a b folgt, aus b c, aus c d, aus d e, und aus e f,
und zwar folgt das einfach durch Verwenden anderer Worte, die dasselbe
bedeuten.
Umgekehrt zu zeigen, daß a aus b folgt, b aus c, c aus d, d aus e, und e aus
f, und zwar wieder einfach durch Verwenden anderer Worte, die dasselbe
bedeuten, dürfte Euch doch nicht allzu schwer fallen, oder?
Christian Gollwitzer
2017-02-21 07:19:57 UTC
Permalink
Post by IV
Heute endlich kam ich auf das für den Beweis noch fehlende Argument.
Ich skizziere den Beweis hier kurz.
[...]
Eine Funktion ist genau dann elementar, also eine elementare Funktion,
wenn sie einstellig und einwertig ist, ihr Definitionsbereich ein
komplexes Intervall ist, und ihre Funktionswerte sich aus ihrem Argument
in einer endlichen Anzahl von Schritten allein durch algebraische
Funktionen, Exponentialfunktionen, Logarithmusfunktionen,
Winkelfunktionen, inversen Winkelfunktionen (= Arkusfunktionen),
Hyperbelfunktionen und/oder inversen Hyperbelfunktionen (=
Areafunktionen) erzeugen lassen.
OK. Aber was ist ein komplexes Intervall? Komplexe Zahlen sind nicht
geordnet. Dieses "erzeugen lassen" ist auch etwas ungenau formuliert,
kann man aber durchaus verstehen, was gemeint ist.
Post by IV
Ist F eine elementare Funktion, dann sind die folgenden Aussagen äquivalent.
a) Die Funktionswerte von F lassen sich aus dem Argument von F in einer
endlichen Anzahl von Schritten allein durch algebraische Funktionen,
Exponentialfunktionen, Logarithmusfunktionen, Winkelfunktionen, inversen
Winkelfunktionen (= Arkusfunktionen), Hyperbelfunktionen und/oder
inversen Hyperbelfunktionen (= Areafunktionen) erzeugen.
b) Die Funktionswerte von F lassen sich aus dem Argument von F allein
durch endlich oftes Anwenden von algebraischen Funktionen,
Exponentialfunktionen, Logarithmusfunktionen, Winkelfunktionen, inversen
Winkelfunktionen (= Arkusfunktionen), Hyperbelfunktionen und/oder
inversen Hyperbelfunktionen (= Areafunktionen) erzeugen.
c) Die Funktionswerte von F lassen sich aus dem Argument von F allein
durch endlich oftes Anwenden von algebraischen Funktionen, exp und/oder
ln erzeugen.
d) Die Funktionswerte von F lassen sich aus dem Argument von F allein
durch endlich ofte Verkettung von algebraischen Funktionen, exp und/oder
ln erzeugen.
e) Die Funktionswerte von F lassen sich aus dem Argument von F allein
durch endlich ofte Verkettung von ein- oder mehrstelligen algebraischen
Funktionen, exp und/oder ln erzeugen.
f) F ist eine Verkettung endlich vieler Funktionen, von denen jede der
Menge {ein- oder mehrstellige algebraische Funktionen, exp, ln} entstammt.
Das ist so trivial, dass es schon fast peinlich ist. Bereits beim Lesen
Deiner Definition 1 dachte ich mir, warum listest Du die Winkel- und
Hyperbelfunktionen, wenn die doch mit exp und ln schon erschlagen sind.
Mehr Inhalt hat dieser Satz jetzt nicht, als dass man die Definition auf
"algebraisch + exp & ln" reduzieren kann. Also lass die einfach in der
ersten Definition weg.
Post by IV
Eine Gleichung E(y) = x, worin E eine elementare Funktion, y die
komplexe Lösungsvariable und x eine komplexe Variable sind, hat genau
dann eine elementare Funktion y als Lösung, wenn die Funktion E eine
elementare Umkehrfunktion hat.
In wiefern ist das ein Satz? Ich würde das als Definition der
"elementaren Umkehrfunktion" auffassen, so nach dem Motto:
"Wenn sich die Gleichung E(x) = y durch x=E^-1(y) auflösen lässt, nennt
man E elementar umkehrbar" oder so ähnlich.
Post by IV
E^-1: Umkehrfunktion der Funktion E
Zuordnungsvorschrift der elementaren Funktion E: E(x) = F(x) (1)
Definitionsgleichung der Umkehrfunktion E^-1: E(E^-1(x)) = x (2)
Transformation y = E^-1(x) (3)
Einsetzen von (2) in Gleichung (1): E(y) = x (4)
Gleichung (4) ist die Gleichung aus Satz 2.
Verwenden von Gleichung (1) in Gleichung (4): F(y) =x (5)
Auflösen von Gleichung (2) nach der Lösungsvariablen y: y = F^-1(x) (6)
Rücktransformation (3): E^-1(x) = F^-1(x) (7)
Die Lösung der Gleichung aus Satz 2 ist genau die Umkehrfunktion E^-1.
Daraus folgt Satz 2.
Dieser Beweis ist genauso lustig, Du setzt voraus dass dieses E^-1
existiert (was nicht trivial ist, siehe z.B. sin(x), x € R) und freust
Dich dann, dass es die Gleichung löst. Überhaupt nicht trivial ist, ob
E^-1 elementar ist. Und da gibt es einfache Gegenbeispiele wie eben
x*exp(x).
Post by IV
Wir wollen jetzt Ritts Satz ([Ritt 1925]) beweisen.
Satz 3 (Satz über die Form elementar umkehrbarer elementarer Funktionen
Wenn die Funktion F und ihre Umkehrfunktion beide elementar sind, dann
existieren n Funktionen F_1, F_2, ..., F_n, wo jedes F_i mit einem
ungeraden Index i eine algebraische Funktion ist, und jedes F_i mit
einem geraden Index i entweder exp oder ln ist, so dass F(z) =
F_n(F_{n-1}(...(F_2(F_1(z)))...)).
Den Rest hier habe mich mir jetzt nicht mehr im Detail angeschaut.
Trotzdem drängt sich die Frage auf, wenn Ritt den Satz schon bewiesen
hat, warum Du dir die Mühe nochmal machen willst.


Christian
IV
2017-02-21 23:36:51 UTC
Permalink
Post by Christian Gollwitzer
Post by IV
Eine Funktion ist genau dann elementar, also eine elementare Funktion,
wenn sie ..., ihr Definitionsbereich ein komplexes Intervall ist, und
...
OK. Aber was ist ein komplexes Intervall? Komplexe Zahlen sind nicht
geordnet. Dieses "Erzeugenlassen" ist auch etwas ungenau formuliert, kann
man aber durchaus verstehen, was gemeint ist.
https://en.wikipedia.org/wiki/Interval_(mathematics)#Complex_intervals
"Erzeugen" stammt von Ritt.
Post by Christian Gollwitzer
Post by IV
Post by IV
Ist F eine elementare Funktion, dann sind die folgenden Aussagen äquivalent.
Das ist so trivial, dass es schon fast peinlich ist.
Manche Mathematiker hier brauchen aber noch einen Beweis.
Post by Christian Gollwitzer
Bereits beim Lesen Deiner Definition 1 dachte ich mir, warum listest Du
die Winkel- und Hyperbelfunktionen, wenn die doch mit exp und ln schon
erschlagen sind.
Die in der Definition genannten Funktionen sind die Elementaren
Standardfunktionen. Nicht jeder weiß, daß die allein durch exp, ln und/oder
algebraische Funktionen darstellbar sind.
Post by Christian Gollwitzer
Mehr Inhalt hat dieser Satz jetzt nicht, als dass man die Definition auf
"algebraisch + exp & ln" reduzieren kann. Also lass die einfach in der
ersten Definition weg.
Ritt fängt ja gleich mit der expln-Darstellung an. Deshalb wissen ja viele
nicht, was die Elementaren Funktionen sein sollen.
Man findet verschiedene Definitionen der Elementaren Funktionen. Ich will
sie alle zusammennehmen und zeigen, daß die Definitionen
äquivalent/gleichwertig(?) sind.
Post by Christian Gollwitzer
Post by IV
Eine Gleichung E(y) = x, worin E eine elementare Funktion, y die komplexe
Lösungsvariable und x eine komplexe Variable sind, hat genau dann eine
elementare Funktion y als Lösung, wenn die Funktion E eine elementare
Umkehrfunktion hat.
In wiefern ist das ein Satz? Ich würde das als Definition der "elementaren
"Wenn sich die Gleichung E(x) = y durch x=E^-1(y) auflösen lässt, nennt
man E elementar umkehrbar" oder so ähnlich.
Die Definition der Elementaren Funktionen ist doch in meiner Definition
schon gegeben.
Satz 2 geht von Gleichungen aus und soll zeigen, daß das Problem des
Gleichungsauflösens mit Gleichungsumwandlungen durch Elementare Funktionen
auf das Problem der elementaren Umkehrfunktionen elementarer Funktionen
zurückgeführt werden kann.
Post by Christian Gollwitzer
Post by IV
E^-1: Umkehrfunktion der Funktion E
Zuordnungsvorschrift der elementaren Funktion E: E(x) = F(x) (1)
Definitionsgleichung der Umkehrfunktion E^-1: E(E^-1(x)) = x (2)
Transformation y = E^-1(x) (3)
Einsetzen von (2) in Gleichung (1): E(y) = x (4)
Gleichung (4) ist die Gleichung aus Satz 2.
Verwenden von Gleichung (1) in Gleichung (4): F(y) =x (5)
Auflösen von Gleichung (2) nach der Lösungsvariablen y: y = F^-1(x)
(6)
Rücktransformation (3): E^-1(x) = F^-1(x) (7)
Die Lösung der Gleichung aus Satz 2 ist genau die Umkehrfunktion E^-1.
Daraus folgt Satz 2.
Dieser Beweis ist genauso lustig
Genauso lustig? Hm. Der Satz oben hat eine große Bedeutung.
Post by Christian Gollwitzer
Post by IV
Beweis von Satz 2
Du setzt voraus dass dieses E^-1 existiert (was nicht trivial ist, siehe
z.B. sin(x), x € R) und freust Dich dann, dass es die Gleichung löst.
Hier war ich wieder ungenau. Hier sind mit "Umkehrfunktion" alle Zweige
(Zweige wovon eigentlich?) gemeint, also alle lokalen Umkehrfunktionen.
Ich möchte die Sätze später noch für lokale Umkehrfunktionen formulieren.
Damit ich genau angeben kann, welche bijektiven Funktionen nun tatsächlich
elementar umkehrbar sind und welche nicht. Elementar umkehrbar sind
wahrscheinlich die elementaren Funktionen, für die im entsprechenden
Definitionsbereich eine lokale elementare Umkehrfunktion existiert.
Post by Christian Gollwitzer
Überhaupt nicht trivial ist, ob E^-1 elementar ist. Und da gibt es
einfache Gegenbeispiele wie eben x*exp(x).
Satz 2 führt das Gleichungsproblem auf das Umkehrfunktionsproblem von Ritt
zurück. Ob die Umkehrfunktion elementar ist oder nicht kann man mit Hilfe
des Satzes von Ritt am Funktionsterm ablesen.
Post by Christian Gollwitzer
Post by IV
Wir wollen jetzt Ritts Satz ([Ritt 1925]) beweisen.
Satz 3 (Satz über die Form elementar umkehrbarer elementarer Funktionen
Wenn die Funktion F und ihre Umkehrfunktion beide elementar sind, dann
existieren n Funktionen F_1, F_2, ..., F_n, wo jedes F_i mit einem
ungeraden Index i eine algebraische Funktion ist, und jedes F_i mit einem
geraden Index i entweder exp oder ln ist, so dass F(z) =
F_n(F_{n-1}(...(F_2(F_1(z)))...)).
Den Rest hier habe mich mir jetzt nicht mehr im Detail angeschaut.
Trotzdem drängt sich die Frage auf, wenn Ritt den Satz schon bewiesen hat,
warum Du dir die Mühe nochmal machen willst.
Unser Beweis wird kurz sein. Ritt braucht für den Beweis 23 Seiten,
Rosenlight für Vergleichbares 9 Seiten, ich vielleicht nur 23 Zeilen.
Unser Beweis wird leicht verständlich sein.
Mit unserem Beweis können wir Ritts Satz auf beinahe beliebige durch
geschlossene Funktionsterme darstellbare Funktionen
erweitern/verallgemeinern.
Unsere Sätze und Beweise werden so einfach und anschaulich sein, daß sie in
die Mathematikausbildung in Studium und Schule eingehen werden.
H0Iger SchuIz
2017-02-22 06:51:59 UTC
Permalink
Post by IV
Post by Christian Gollwitzer
Post by IV
Eine Funktion ist genau dann elementar, also eine elementare Funktion,
wenn sie ..., ihr Definitionsbereich ein komplexes Intervall ist, und
...
OK. Aber was ist ein komplexes Intervall? Komplexe Zahlen sind nicht
geordnet. Dieses "Erzeugenlassen" ist auch etwas ungenau formuliert, kann
man aber durchaus verstehen, was gemeint ist.
https://en.wikipedia.org/wiki/Interval_(mathematics)#Complex_intervals
Bist du sicher, das du solche (rechteckige oder kreisförmige)
Definitionsbereiche haben möchtest?
Post by IV
"Erzeugen" stammt von Ritt.
So hat der das vor fast 100 Jahren geschrieben. Wann möchtest du deinen
Artikel veröffentlichen.
Post by IV
Post by Christian Gollwitzer
Post by IV
Post by IV
Ist F eine elementare Funktion, dann sind die folgenden Aussagen äquivalent.
Das ist so trivial, dass es schon fast peinlich ist.
Manche Mathematiker hier brauchen aber noch einen Beweis.
Nur, wenn es etwas zu beweisen gibt. Und dann sollte es auch ein Beweis
sein.
Post by IV
Post by Christian Gollwitzer
Bereits beim Lesen Deiner Definition 1 dachte ich mir, warum listest Du
die Winkel- und Hyperbelfunktionen, wenn die doch mit exp und ln schon
erschlagen sind.
Die in der Definition genannten Funktionen sind die Elementaren
Standardfunktionen. Nicht jeder weiß, daß die allein durch exp, ln und/oder
algebraische Funktionen darstellbar sind.
Wenn's darauf ankommt, würde ich das auch so formulieren.
Post by IV
Post by Christian Gollwitzer
Mehr Inhalt hat dieser Satz jetzt nicht, als dass man die Definition auf
"algebraisch + exp & ln" reduzieren kann. Also lass die einfach in der
ersten Definition weg.
Ritt fängt ja gleich mit der expln-Darstellung an. Deshalb wissen ja viele
nicht, was die Elementaren Funktionen sein sollen.
Man findet verschiedene Definitionen der Elementaren Funktionen. Ich will
sie alle zusammennehmen und zeigen, daß die Definitionen
äquivalent/gleichwertig(?) sind.
Mach das.
Post by IV
Post by Christian Gollwitzer
Post by IV
Eine Gleichung E(y) = x, worin E eine elementare Funktion, y die komplexe
Lösungsvariable und x eine komplexe Variable sind, hat genau dann eine
elementare Funktion y als Lösung, wenn die Funktion E eine elementare
Umkehrfunktion hat.
In wiefern ist das ein Satz? Ich würde das als Definition der "elementaren
"Wenn sich die Gleichung E(x) = y durch x=E^-1(y) auflösen lässt, nennt
man E elementar umkehrbar" oder so ähnlich.
Die Definition der Elementaren Funktionen ist doch in meiner Definition
schon gegeben.
Satz 2 geht von Gleichungen aus und soll zeigen, daß das Problem des
Gleichungsauflösens mit Gleichungsumwandlungen durch Elementare Funktionen
auf das Problem der elementaren Umkehrfunktionen elementarer Funktionen
zurückgeführt werden kann.
Wo genau geht die Eigenschaft _elementare_ Funktion in den "Beweis" ein?
Post by IV
Post by Christian Gollwitzer
Post by IV
E^-1: Umkehrfunktion der Funktion E
Zuordnungsvorschrift der elementaren Funktion E: E(x) = F(x) (1)
Definitionsgleichung der Umkehrfunktion E^-1: E(E^-1(x)) = x (2)
Transformation y = E^-1(x) (3)
Einsetzen von (2) in Gleichung (1): E(y) = x (4)
Gleichung (4) ist die Gleichung aus Satz 2.
Verwenden von Gleichung (1) in Gleichung (4): F(y) =x (5)
Auflösen von Gleichung (2) nach der Lösungsvariablen y: y = F^-1(x)
(6)
Rücktransformation (3): E^-1(x) = F^-1(x) (7)
Die Lösung der Gleichung aus Satz 2 ist genau die Umkehrfunktion E^-1.
Daraus folgt Satz 2.
Dieser Beweis ist genauso lustig
Genauso lustig? Hm. Der Satz oben hat eine große Bedeutung.
Nein.
Post by IV
Post by Christian Gollwitzer
Post by IV
Beweis von Satz 2
Du setzt voraus dass dieses E^-1 existiert (was nicht trivial ist, siehe
z.B. sin(x), x € R) und freust Dich dann, dass es die Gleichung löst.
Hier war ich wieder ungenau. Hier sind mit "Umkehrfunktion" alle Zweige
(Zweige wovon eigentlich?) gemeint, also alle lokalen Umkehrfunktionen.
Unklar. Warum ist aif einmal von lokalen Umkehrfunktionen die Rede?
Warum kommst du immer wieder mit neuen Begriffen um die Ecke, wenn
jemand einen Fehler findet?
Post by IV
Ich möchte die Sätze später noch für lokale Umkehrfunktionen formulieren.
Damit ich genau angeben kann, welche bijektiven Funktionen nun tatsächlich
elementar umkehrbar sind und welche nicht. Elementar umkehrbar sind
wahrscheinlich die elementaren Funktionen, für die im entsprechenden
Definitionsbereich eine lokale elementare Umkehrfunktion existiert.
Das scheint mit wenig Substanz zu haben. Es klingt eher so, als soll es
wie Mathematik klingen.
Post by IV
Post by Christian Gollwitzer
Überhaupt nicht trivial ist, ob E^-1 elementar ist. Und da gibt es
einfache Gegenbeispiele wie eben x*exp(x).
Satz 2 führt das Gleichungsproblem auf das Umkehrfunktionsproblem von Ritt
zurück. Ob die Umkehrfunktion elementar ist oder nicht kann man mit Hilfe
des Satzes von Ritt am Funktionsterm ablesen.
Wie?
Post by IV
Post by Christian Gollwitzer
Post by IV
Wir wollen jetzt Ritts Satz ([Ritt 1925]) beweisen.
Satz 3 (Satz über die Form elementar umkehrbarer elementarer Funktionen
Wenn die Funktion F und ihre Umkehrfunktion beide elementar sind, dann
existieren n Funktionen F_1, F_2, ..., F_n, wo jedes F_i mit einem
ungeraden Index i eine algebraische Funktion ist, und jedes F_i mit einem
geraden Index i entweder exp oder ln ist, so dass F(z) =
F_n(F_{n-1}(...(F_2(F_1(z)))...)).
Den Rest hier habe mich mir jetzt nicht mehr im Detail angeschaut.
Trotzdem drängt sich die Frage auf, wenn Ritt den Satz schon bewiesen hat,
warum Du dir die Mühe nochmal machen willst.
Unser
Ah, Hoheit verwenden den zustehenden Plural. Brav!
Post by IV
Beweis wird kurz sein. Ritt braucht für den Beweis 23 Seiten,
Rosenlight für Vergleichbares 9 Seiten, ich vielleicht nur 23 Zeilen.
Wenn die letzte Zeile ist "Der Rest folgt aus dem Konstruktionsprinzip
von Ritt." kriegt man es auch in 22 Zeilen hin.

Macht es Sinn Beweislänge in Seiten und Zeilen zu messen? Velleicht doch
besser in Zeichenanzahl?
Post by IV
Unser Beweis wird leicht verständlich sein.
Oh, dann kommt noch etwas Verständliches? Da bin ich aber mal gespannt.
Post by IV
Mit unserem Beweis können wir Ritts Satz auf beinahe beliebige durch
geschlossene Funktionsterme darstellbare Funktionen
erweitern/verallgemeinern.
Unsere Sätze und Beweise werden so einfach und anschaulich sein, daß sie in
die Mathematikausbildung in Studium und Schule eingehen werden.
Ich würd' den Hafen nicht so weit aufreißen, da kommt ja schon Licht
durch. Das passt nicht zu "Ich bin ja kein Mathematiker, ich weiß nicht
wie das geht." Entscheide dich für einen Troll-Ansatz.

hs
IV
2017-02-22 17:31:33 UTC
Permalink
Post by H0Iger SchuIz
Post by IV
Post by Christian Gollwitzer
Post by IV
Eine Funktion ist genau dann elementar, also eine elementare Funktion,
wenn sie ..., ihr Definitionsbereich ein komplexes Intervall ist, und
...
OK. Aber was ist ein komplexes Intervall?
https://en.wikipedia.org/wiki/Interval_(mathematics)#Complex_intervals
Bist du sicher, das du solche (rechteckige oder kreisförmige)
Definitionsbereiche haben möchtest?
D: Definitionsbereich
Eigentlich reicht D \subseteq \mathbb{C}. Dann muß ich in den Sätzen aber
mehrelementige Definitionsbereiche voraussetzen.
Post by H0Iger SchuIz
Post by IV
Post by Christian Gollwitzer
Post by IV
Eine Gleichung E(y) = x, worin E eine elementare Funktion, y die
komplexe Lösungsvariable und x eine komplexe Variable sind, hat genau
dann eine elementare Funktion y als Lösung, wenn die Funktion E eine
elementare Umkehrfunktion hat.
Gemeint ist: "eine lokale elementare Umkehrfunktion hat".
Post by H0Iger SchuIz
Wo genau geht die Eigenschaft _elementare_ Funktion in den "Beweis" ein?
Gegeben ist eine beliebige Gleichung E(y) = x, worin E eine Elementare
Funktion ist. Diese Gleichung ist auch die Definitionsgleichung jeder
lokalen Umkehrfunktion E^-1 mit y = E^-1(x) der Elementaren Funktion E.
a) Wenn eine lokale Umkehrfunktion E^-1 existiert, dann ist die Gleichung
nach y auflösbar: y = E^-1(x). Wenn E^-1 eine Elementare Funktion ist, dann
ist auch y eine Elementare Funktion.
b) Wenn keine lokale Umkehrfunktion zu E existiert, dann ist die Gleichung
nicht nach y auflösbar. Wenn eine lokale Umkehrfunktion E^-1 existiert,
diese aber keine Elementare Funktion ist, dann ist die Gleichung zwar nach y
auflösbar, y ist dann aber keine Elementare Funktion.
Aus a und b folgt der Satz.
Post by H0Iger SchuIz
Post by IV
Satz 2
Der Satz oben hat eine große Bedeutung.
Nein.
Oh ja, ich sehe, Du hast wieder mal recht. Weil der Satz ja nur hier bekannt
ist, kann er natürlich noch keine große Bedeutung haben.
Post by H0Iger SchuIz
Post by IV
Post by Christian Gollwitzer
Post by IV
Beweis von Satz 2
Du setzt voraus dass dieses E^-1 existiert (was nicht trivial ist, siehe
z. B. sin(x), x € R) und freust Dich dann, dass es die Gleichung löst.
Hier war ich wieder ungenau. Hier sind mit "Umkehrfunktion" alle Zweige
(Zweige wovon eigentlich?) gemeint, also alle lokalen Umkehrfunktionen.
Unklar. Warum ist auf einmal von lokalen Umkehrfunktionen die Rede?
E(y) = x wird durch y = E^-1(x) nach y aufgelöst. Die Funktion E muß nicht
unbedingt bijektiv sein. Enthält E algebraische Funktionen, dann können
unter Umständen lokale Umkehrfunktionen E^-1 existieren, durch die die
Gleichung gelöst wird.
Post by H0Iger SchuIz
Warum kommst du immer wieder mit neuen Begriffen um die Ecke, wenn jemand
einen Fehler findet?
Weil ich hier vergessen hatte, daß ich beim Problem Gleichunglösen die
lokalen Umkehrfunktionen mit erwähnen wollte.
Das Problem Gleichunglösen war nur zur Einführung. Ich will es erst viel
später behandeln.
Post by H0Iger SchuIz
Post by IV
Post by Christian Gollwitzer
Überhaupt nicht trivial ist, ob E^-1 elementar ist. Und da gibt es
einfache Gegenbeispiele wie eben x*exp(x).
Satz 2 führt das Gleichungsproblem auf das Umkehrfunktionsproblem von
Ritt zurück. Ob die Umkehrfunktion elementar ist oder nicht kann man mit
Hilfe des Satzes von Ritt am Funktionsterm ablesen.
Wie?
Im Moment möchte ich mich ja nur auf den Beweis von Ritts Satz
konzentrieren.
Aber vorab: Nach Ritts Satz kann nur eine lineare Verkettung elementar
umkehrbar sein. In Erweiterung von Ritts Satz gebe ich den allgemeinen
Funktionsterm der elementaren Umkehrfunktion an: die Umkehrfunktionen der
einzelnen Glieder in der umgekehrten Reihenfolge.
Post by H0Iger SchuIz
Post by IV
Unser
Ah, Hoheit verwenden den zustehenden Plural. Brav!
(Was soll die Bemerkung?)
Üblicherweise spricht man in einem Team so.
H0Iger SchuIz
2017-02-22 18:11:53 UTC
Permalink
Post by IV
D: Definitionsbereich
Eigentlich reicht D \subseteq \mathbb{C}. Dann muß ich in den Sätzen aber
mehrelementige Definitionsbereiche voraussetzen.
Zwei isolierte Punkte reichen? Oder doch lieber ein offenes Gebiet?
Post by IV
Post by H0Iger SchuIz
Post by IV
Post by Christian Gollwitzer
Post by IV
Eine Gleichung E(y) = x, worin E eine elementare Funktion, y die
komplexe Lösungsvariable und x eine komplexe Variable sind, hat genau
dann eine elementare Funktion y als Lösung, wenn die Funktion E eine
elementare Umkehrfunktion hat.
Gemeint ist: "eine lokale elementare Umkehrfunktion hat".
Steht da aber nicht.
Post by IV
Post by H0Iger SchuIz
Wo genau geht die Eigenschaft _elementare_ Funktion in den "Beweis" ein?
Gegeben ist eine beliebige Gleichung E(y) = x, worin E eine Elementare
Funktion ist. Diese Gleichung ist auch die Definitionsgleichung jeder
lokalen Umkehrfunktion E^-1 mit y = E^-1(x) der Elementaren Funktion E.
a) Wenn eine lokale Umkehrfunktion E^-1 existiert, dann ist die Gleichung
y = E^-1(x).
Das geht mir jetzt doch ein wenig schnell. Immerhin haben wir es hier
nur mit einer lokalen Umkehrfunktion zu tun. Muss man dann nicht
vielleicht mal nachschauen für welche xse und ypsilonse das gelten soll?
Woran soll denn die Lokalität der "Umkehrfunktion" erkennen?
Post by IV
Wenn E^-1 eine Elementare Funktion ist, dann
ist auch y eine Elementare Funktion.
y sieht mir so überhaupt nicht wie eine Funktion aus, sondern eher wie
ein Wert, also eine komlexe(?) Zahl. Ich glaube, da ist in der
Formulierung des Satzes schon etwas schief gegangen.
Post by IV
Post by H0Iger SchuIz
Post by IV
Post by Christian Gollwitzer
Post by IV
Beweis von Satz 2
Du setzt voraus dass dieses E^-1 existiert (was nicht trivial ist, siehe
z. B. sin(x), x € R) und freust Dich dann, dass es die Gleichung löst.
Hier war ich wieder ungenau. Hier sind mit "Umkehrfunktion" alle Zweige
(Zweige wovon eigentlich?) gemeint, also alle lokalen Umkehrfunktionen.
Unklar. Warum ist auf einmal von lokalen Umkehrfunktionen die Rede?
E(y) = x wird durch y = E^-1(x) nach y aufgelöst. Die Funktion E muß nicht
unbedingt bijektiv sein. Enthält E algebraische Funktionen, dann können
unter Umständen lokale Umkehrfunktionen E^-1 existieren, durch die die
Gleichung gelöst wird.
Aber wohl auch nur lokal. Mir scheint, hier wird der Begriff "lokal"
irgendwie dazwischengeschoben, weil die Bedingung der Existenz einer
lokalen Umkehrfunktion einfacher zu erfüllen erscheint. Ansonsten wird
aber von ihm gar keine Notiz genommen, sondern man formuliert munter
darauf los, als gebe es eine globale Umkehrfunktion.
Post by IV
Post by H0Iger SchuIz
Warum kommst du immer wieder mit neuen Begriffen um die Ecke, wenn jemand
einen Fehler findet?
Weil ich hier vergessen hatte, daß ich beim Problem Gleichunglösen die
lokalen Umkehrfunktionen mit erwähnen wollte.
Aha.
Post by IV
Das Problem Gleichunglösen war nur zur Einführung. Ich will es erst viel
später behandeln.
Dann vergessen wir das einstweilen.
Post by IV
Post by H0Iger SchuIz
Post by IV
Post by Christian Gollwitzer
Überhaupt nicht trivial ist, ob E^-1 elementar ist. Und da gibt es
einfache Gegenbeispiele wie eben x*exp(x).
Satz 2 führt das Gleichungsproblem auf das Umkehrfunktionsproblem von
Ritt zurück. Ob die Umkehrfunktion elementar ist oder nicht kann man mit
Hilfe des Satzes von Ritt am Funktionsterm ablesen.
Wie?
Im Moment möchte ich mich ja nur auf den Beweis von Ritts Satz
konzentrieren.
Aber vorab: Nach Ritts Satz kann nur eine lineare Verkettung elementar
umkehrbar sein.
Nein. Ritt sagt, nur eine Funktion, die sich in jener Verkettungsform
darstellen lässt, ist elementar umkehrbar. Ist die Funktion z.B. durch
einen anderen Funktionsterm gegeben, weiß man gar nichts, bis man
entscheiden kann, ob eine solche Darstellung möglich ist oder nicht.

Ich meine nach wie vor unterscheidet ihr nicht genau genug zwischen
Funktion und Darstellung.
Post by IV
In Erweiterung von Ritts Satz gebe ich den allgemeinen
Funktionsterm der elementaren Umkehrfunktion an: die Umkehrfunktionen der
einzelnen Glieder in der umgekehrten Reihenfolge.
Das geht nur, wenn die Glieder bijektiv sind. Mir ist's bald, als wäre
das schon erwähnt worden.
Post by IV
Post by H0Iger SchuIz
Post by IV
Unser
Ah, Hoheit verwenden den zustehenden Plural. Brav!
(Was soll die Bemerkung?)
Üblicherweise spricht man in einem Team so.
Ah, ein Team. Da habe ich wohl etwas verwechselt. Sorry.

hs
IV
2017-02-22 19:19:47 UTC
Permalink
Post by H0Iger SchuIz
Post by IV
D: Definitionsbereich
Eigentlich reicht D \subseteq \mathbb{C}. Dann muß ich in den Sätzen aber
mehrelementige Definitionsbereiche voraussetzen.
Zwei isolierte Punkte reichen? Oder doch lieber ein offenes Gebiet?
Warum offen, warum nicht auch ab- oder halbgeschlossen?
Post by H0Iger SchuIz
Post by IV
Gegeben ist eine beliebige Gleichung E(y) = x, worin E eine Elementare
Funktion ist. Diese Gleichung ist auch die Definitionsgleichung jeder
lokalen Umkehrfunktion E^-1 mit y = E^-1(x) der Elementaren Funktion E.
a) Wenn eine lokale Umkehrfunktion E^-1 existiert, dann ist die Gleichung
y = E^-1(x).
Das geht mir jetzt doch ein wenig schnell. Immerhin haben wir es hier nur
mit einer lokalen Umkehrfunktion zu tun. Muss man dann nicht vielleicht
mal nachschauen für welche xse und ypsilonse das gelten soll?
Woran soll denn die Lokalität der "Umkehrfunktion" erkennen?
Das Problem des Gleichunglösens soll jetzt eigentlich noch nicht dran sein.
Ich hab's mir ja noch nicht genau überlegt. Ich habe nur so eine Idee.
Bestimmt findet sich in der Literatur der Zusammenhang zwischen
Gleichunglösen und Umkehrfunktionen.
Es ist doch nichts Anderes als Gleichunglösen durch Anwenden der
Umkehrtransformationen. Wenn die Umkehrtransformationen nicht bijektiv sind,
müssen die Definitions- und Wertebereiche ihrer einzelnen Umkehrfunktionen
untersucht werden.
Wenn man den Definitionsbereich der Gleichung jeweils immer so einschränkt,
daß die lokalen Umkehrfunktionen aller Glieder der Verkettungsdarstellung zu
(globalen) Umkehrfunktionen werden, dann kann man sogar mit den
nicht-lokalen Formulierungen des Satzes arbeiten.
Post by H0Iger SchuIz
Ich meine nach wie vor unterscheidet ihr nicht genau genug zwischen
Funktion und Darstellung.
Ja, Du hast recht. Danke, daß Du es auf den Punkt gebracht hast. Mit Deinen
beiden Begriffen kann man arbeiten.
Post by H0Iger SchuIz
Post by IV
In Erweiterung von Ritts Satz gebe ich den allgemeinen Funktionsterm der
elementaren Umkehrfunktion an: die Umkehrfunktionen der einzelnen Glieder
in der umgekehrten Reihenfolge.
Das geht nur, wenn die Glieder bijektiv sind. Mir ist's bald, als wäre das
schon erwähnt worden.
Der Satz über die Form des Funktionsterms der Umkehrfunktion könnte
beinhalten, daß man im Falle der elementaren Umkehrbarkeit jedes Gliedes die
Umkehrfunktion in der oben angegebenen Form angeben kann.
Keine Ahnung, ob die Umkehrung auch gilt.
Der gerade besprochene Satz geht ja schon über das in der Literatur Bekannte
hinaus.
Hier sind doch eine Reihe von Sätzen angegeben worden. Vielleicht knöpft
sich ja der eine oder andere einen dieser einfacheren Sätze vor und
versucht, Beiträge zu dessen Beweis zu liefern. Ich würde doch viel zu lange
dafür brauchen.
H0Iger SchuIz
2017-02-23 06:43:17 UTC
Permalink
Post by IV
Post by H0Iger SchuIz
Post by IV
D: Definitionsbereich
Eigentlich reicht D \subseteq \mathbb{C}. Dann muß ich in den Sätzen aber
mehrelementige Definitionsbereiche voraussetzen.
Zwei isolierte Punkte reichen? Oder doch lieber ein offenes Gebiet?
Warum offen, warum nicht auch ab- oder halbgeschlossen?
Womöglich gibt es relevante Sätze, die Offenheit voraussetzen. Aber
beruhigend zu sehen, dass ihr euch noch überhaupt keine Gedanken dazu
gemacht habt.

hs
Hans Crauel
2017-02-23 11:45:52 UTC
Permalink
H0Iger SchuIz schrieb [betr. D \subseteq \mathbb{C}]
Post by H0Iger SchuIz
"H0Iger SchuIz" schreibt
Post by H0Iger SchuIz
Zwei isolierte Punkte reichen? Oder doch lieber ein offenes Gebiet?
Warum offen, warum nicht auch ab- oder halbgeschlossen?
Was soll es heissen, dass eine Teilmenge der komplexen Zahlen
"halbgeschlossen" ist? Dito fuer reelle Zahlen?
Post by H0Iger SchuIz
Womöglich gibt es relevante Sätze, die Offenheit voraussetzen.
Oft genuegt es, nichtleeres Inneres zu haben (und bekommt die
Aussage fuer innere Punkte).

Hans Crauel
IV
2017-02-23 16:15:00 UTC
Permalink
Post by Hans Crauel
Post by IV
Post by H0Iger SchuIz
Zwei isolierte Punkte reichen? Oder doch lieber ein offenes Gebiet?
Warum offen, warum nicht auch ab- oder halbgeschlossen?
Was soll es heissen, dass eine Teilmenge der komplexen Zahlen
"albgeschlossen" ist? Dito fuer reelle Zahlen?
Mit "halbgeschlossen" meinte ich "halboffen".
Hans Crauel
2017-02-23 19:30:20 UTC
Permalink
IV schrieb
"Hans Crauel" schrieb
Post by Hans Crauel
Post by IV
Warum offen, warum nicht auch ab- oder halbgeschlossen?
Was soll es heissen, dass eine Teilmenge der komplexen Zahlen
"halbgeschlossen" ist? Dito fuer reelle Zahlen?
Mit "halbgeschlossen" meinte ich "halboffen".
Was soll denn das nun wieder?
Was meinst du mit "halboffene" Teilmenge der komplexen Zahlen?

Hans Crauel
IV
2017-02-23 16:23:28 UTC
Permalink
Post by H0Iger SchuIz
Post by H0Iger SchuIz
Post by IV
Eigentlich reicht D \subseteq \mathbb{C}. Dann muß ich in den Sätzen
aber mehrelementige Definitionsbereiche voraussetzen.
Zwei isolierte Punkte reichen? Oder doch lieber ein offenes Gebiet?
Womöglich gibt es relevante Sätze, die Offenheit voraussetzen. Aber
beruhigend zu sehen, dass ihr euch noch überhaupt keine Gedanken dazu
gemacht habt.
Warum sollte Offenheit eine Rolle für die Umkehrbarkeit spielen?

Die erste Stelle an der in Ritts Artikel das Wort "complex" vorkommt ist die
hier:
"4. Let r be any area in the complex plane, and suppose that we can continue
the above mentioned element of u with center at z0 into and all over r, so
that u has a branch which is uniform and analytic throughout r. Let C be
some curve along which u can be continued from z0 into r. Any curve which
can be obtained from C by a slight deformation will serve equally well for
the continuation of u into r. As each monomial in (2) is analytic almost
everywhere, we can take a curve close to C all along which each monomial can
be continued from z0. It is easy to see that a single curve can be taken for
all the monomials, because a curve which will do for one of them can be
shifted slightly so as to do also for another. We conclude that in any area
in which u has an analytic branch, there is an area in which all the
monomials in (2) have analytic branches which satisfy (2) together with u.
Evidently we can choose the smaller area in such a way that each of the
algebraic functions of which the monomials in (2) are exponentials or
logarithms is analytic in the smaller area."
Was ist nun der Definitionsbereich?
Torn Rumero DeBrak
2017-02-22 18:44:37 UTC
Permalink
Post by IV
Post by H0Iger SchuIz
Post by IV
Post by Christian Gollwitzer
Post by IV
Eine Funktion ist genau dann elementar, also eine elementare
Funktion, wenn sie ..., ihr Definitionsbereich ein komplexes
Intervall ist, und ...
OK. Aber was ist ein komplexes Intervall?
https://en.wikipedia.org/wiki/Interval_(mathematics)#Complex_intervals
Bist du sicher, das du solche (rechteckige oder kreisförmige)
Definitionsbereiche haben möchtest?
D: Definitionsbereich
Eigentlich reicht D \subseteq \mathbb{C}. Dann muß ich in den Sätzen
aber mehrelementige Definitionsbereiche voraussetzen.
^^^^^^^^^^^^^^

Das meinst du sicher nicht. Ein "mehrelementiger" Definitionsbereich
enthält vom sprachlichen Verständnis her mehrere Elemente, also
z.B { 1, 4^27, 7.5, sqrt(19) } im Gegensatz zu einelementige Bereiche
wie { 3.73 } .

Solltest du "mehrdimensional" meinen?

...
Post by IV
Im Moment möchte ich mich ja nur auf den Beweis von Ritts Satz
konzentrieren.
Aber vorab: Nach Ritts Satz kann nur eine lineare Verkettung elementar
Mir ist noch nicht klar, was du unter einer "linearen" Verkettung
verstehst und wie eine "nicht-lineare" Verkettung im Gegensatz dazu
aussieht. Ich gehe doch richtig in der Annahme, dass "Verkettung" bei
dir für "Komposition" steht und nicht eine eigene Bedeutung besitzt?
Wie sollte dann aber mit "o" eine nicht-lineare Struktur aufgebaut
werden?
Post by IV
umkehrbar sein. In Erweiterung von Ritts Satz gebe ich den allgemeinen
Funktionsterm der elementaren Umkehrfunktion an: die Umkehrfunktionen
der einzelnen Glieder in der umgekehrten Reihenfolge.
IV
2017-02-22 19:45:04 UTC
Permalink
Post by IV
Post by H0Iger SchuIz
Post by IV
Post by Christian Gollwitzer
Post by IV
Eine Funktion ist genau dann elementar, also eine elementare
Funktion, wenn sie ..., ihr Definitionsbereich ein komplexes
Intervall ist, und ...
OK. Aber was ist ein komplexes Intervall?
https://en.wikipedia.org/wiki/Interval_(mathematics)#Complex_intervals
Bist du sicher, das du solche (rechteckige oder kreisförmige)
Definitionsbereiche haben möchtest?
D: Definitionsbereich
Eigentlich reicht D \subseteq \mathbb{C}. Dann muß ich in den Sätzen aber
mehrelementige Definitionsbereiche voraussetzen.
mehrelementige Definitionsbereiche ^^^^^^^^^^^^^
Das meinst du sicher nicht. Ein "mehrelementiger" Definitionsbereich
enthält vom sprachlichen Verständnis her mehrere Elemente, also z.B { 1,
4^27, 7.5, sqrt(19) } im Gegensatz zu einelementige Bereichen wie {
3.73 }.
Solltest du "mehrdimensional" meinen?
Mir scheint, die Umkehrbarkeit durch eine Elementare Funktion ist lediglich
eine Eigenschaft des Funktionsterms - unabhängig vom Definitionsbereich, bis
auf die Ausnahme einelementiger Definitionsbereiche.
Wie ist der Fachbegriff für den allgemeinsten Definitionsbereich im
Komplexen, der einelementige Definitionsbereiche ausschließt? Der
Definitionsbereich braucht ja nicht zusammenhängend sein. Hm, welche Arten
von Zusammenhang braucht man hier?
Post by IV
Post by H0Iger SchuIz
Im Moment möchte ich mich ja nur auf den Beweis von Ritts Satz
konzentrieren.
Aber vorab: Nach Ritts Satz kann nur eine lineare Verkettung ...
Mir ist noch nicht klar, was du unter einer "linearen" Verkettung
verstehst und wie eine "nicht-lineare" Verkettung im Gegensatz dazu
aussieht. Ich gehe doch richtig in der Annahme, dass "Verkettung" bei dir
für "Komposition" steht und nicht eine eigene Bedeutung besitzt?
Wie sollte dann aber mit "o" eine nicht-lineare Struktur aufgebaut werden?
Verkettung = Hintereinanderausführung = Komposition von Funktionen, aber
mit ein- oder mehrstelligen einwertigen Gliedern.
(Leider kann man die Formel für solche allgemeine Verkettung schlecht
aufschreiben. Ein "Verkettungsgraph" ist wohl die übersichtlichste
Darstellung.)
Leider habe bisher noch keine Fachbegriffe für solche verallgemeinerten
Kompositionen gefunden. Superposition kommt dem wohl am nächsten, aber die
enthält wohl immer auch Summen.
expln-Darstellung ist klar? (convert(...,expln) ist übrigens eine Befehl in
MAPLE.)
exp und ln sind einstellig und einwertig. Die Verkettung lediglich
einstelliger einwertiger Funktionen ist linear.
Durch mehrstellige oder mehrwertige Glieder in einer Verkettung wird die
Verkettung "verzweigt"/"zweidimensional", jedenfalls nicht-linear.

Relationen kann man hintereinander ausführen. Läßt sich daraus vielleicht
etwas über die Hintereinanderausführung mehrstelliger und/oder mehrwertiger
Funktionen sagen?
Torn Rumero DeBrak
2017-02-22 21:53:51 UTC
Permalink
Post by IV
Post by IV
Post by H0Iger SchuIz
Post by IV
Post by Christian Gollwitzer
Post by IV
Eine Funktion ist genau dann elementar, also eine elementare
Funktion, wenn sie ..., ihr Definitionsbereich ein komplexes
Intervall ist, und ...
OK. Aber was ist ein komplexes Intervall?
https://en.wikipedia.org/wiki/Interval_(mathematics)#Complex_intervals
Bist du sicher, das du solche (rechteckige oder kreisförmige)
Definitionsbereiche haben möchtest?
D: Definitionsbereich
Eigentlich reicht D \subseteq \mathbb{C}. Dann muß ich in den Sätzen
aber mehrelementige Definitionsbereiche voraussetzen.
mehrelementige Definitionsbereiche ^^^^^^^^^^^^^
Das meinst du sicher nicht. Ein "mehrelementiger" Definitionsbereich
enthält vom sprachlichen Verständnis her mehrere Elemente, also z.B {
1, 4^27, 7.5, sqrt(19) } im Gegensatz zu einelementige Bereichen wie {
3.73 }.
Solltest du "mehrdimensional" meinen?
Mir scheint, die Umkehrbarkeit durch eine Elementare Funktion ist
lediglich eine Eigenschaft des Funktionsterms - unabhängig vom
Definitionsbereich, bis auf die Ausnahme einelementiger
Definitionsbereiche.
Nein. Die Umkehrbarkeit hängt auch vom Definitionsbereich und
Wertebereich ab, selbst wenn der Funktionsterm gleich bleibt.
Das hatte ich dir doch schon am Beispiel der Quadratfunktion gezeigt:

F1: IR -> IR, x |-> x^2 ist nicht umkehrbar, da nicht surjektiv und
nicht injektiv
F2: IR^>=0 -> IR, x |-> x^2 ist nicht umkehrbar, da nicht surjektiv
F3: IR -> IR^>=0, x |-> x^2 ist nicht umkehrbar, da nicht injektiv
F4: IR^>=0 -> IR^>=0, x|-> x^2 ist umkehrbar, da bijektiv
F5: IR^<=0 -> IR^>=0, x |-> x^2 ist umkehrbar, da bijektiv

In allen fünf Fällen ist der Funktionsterm gleich (nämlich x^2),
die Umkehrbarkeit hängt aber vom Definitions- und Wertebereich ab.

Und das hat auch nichts damit zu tun, ob der Definitionsbereich
nur ein einziges Element enthält oder unendlich viele

F6: { -1, 0, 1 } -> {0, 1}, x|-> x^2 ist nicht umkehrbar, da nicht
injektiv, und hier enthält der Definitionsbereich 3 Elemente.
Post by IV
Wie ist der Fachbegriff für den allgemeinsten Definitionsbereich im
Komplexen, der einelementige Definitionsbereiche ausschließt? Der
Definitionsbereich braucht ja nicht zusammenhängend sein. Hm, welche
Arten von Zusammenhang braucht man hier?
"offen" bezüglich der Standardtopologie in IC wäre ein guter Kandidat.
Post by IV
Post by IV
Post by H0Iger SchuIz
Im Moment möchte ich mich ja nur auf den Beweis von Ritts Satz
konzentrieren.
Aber vorab: Nach Ritts Satz kann nur eine lineare Verkettung ...
Mir ist noch nicht klar, was du unter einer "linearen" Verkettung
verstehst und wie eine "nicht-lineare" Verkettung im Gegensatz dazu
aussieht. Ich gehe doch richtig in der Annahme, dass "Verkettung" bei
dir für "Komposition" steht und nicht eine eigene Bedeutung besitzt?
Wie sollte dann aber mit "o" eine nicht-lineare Struktur aufgebaut werden?
Verkettung = Hintereinanderausführung = Komposition von Funktionen,
aber mit ein- oder mehrstelligen einwertigen Gliedern.
(Leider kann man die Formel für solche allgemeine Verkettung schlecht
aufschreiben. Ein "Verkettungsgraph" ist wohl die übersichtlichste
Darstellung.)
Leider habe bisher noch keine Fachbegriffe für solche verallgemeinerten
Kompositionen gefunden. Superposition kommt dem wohl am nächsten, aber
die enthält wohl immer auch Summen.
expln-Darstellung ist klar? (convert(...,expln) ist übrigens eine Befehl
in MAPLE.)
exp und ln sind einstellig und einwertig. Die Verkettung lediglich
einstelliger einwertiger Funktionen ist linear.
Nein: denn der komplexe Logarithmus ist mehrwertig!
Post by IV
Durch mehrstellige oder mehrwertige Glieder in einer Verkettung wird die
Verkettung "verzweigt"/"zweidimensional", jedenfalls nicht-linear.
Wie soll das gehen? Wenn folgende Funktionen gegeben sind:

F1: IR x IR X IR -> IR x IR, (x1, x2, x3) |-> (y1, y2)
F2: IR x IR -> IR x IR x IR x IR, (y1, y2) |-> (z1, z2, z3, z4)
F3: IR x IR x IR x IR -> IR, (z1, z2, z3, z4) |-> w1

dann ist doch die Komposition

F3 o F2 o F1

linear und kein Graph, der irgendwo verzweigt.

Oder gehörst du der Meinungsgruppe an, die obige Funktionen
als einwertig und einstellig betrachtet? Dann hat nämlich
F1 nur ein Argument (ein 3-Tupel) und einen Wert (ein 2-Tupel),
so dass man statt

F1(x1, x2, x3)

die Schreibweise

F1((x1, x2, x3))

benutzen sollte.

Was sind dann aber die Begriffe mehrstellig und mehrwertig bei dir?
H0Iger SchuIz
2017-02-23 06:43:17 UTC
Permalink
Post by Torn Rumero DeBrak
Post by IV
Mir scheint, die Umkehrbarkeit durch eine Elementare Funktion ist
lediglich eine Eigenschaft des Funktionsterms - unabhängig vom
Definitionsbereich,
Nein, eben nicht. Das könnte man erkennen, wenn man sich die Definition
der Bijektivität einmal durchliest.
Post by Torn Rumero DeBrak
Nein. Die Umkehrbarkeit hängt auch vom Definitionsbereich und
Wertebereich ab, selbst wenn der Funktionsterm gleich bleibt.
^^^^^

Eben. Das wurde alles schon mal erklärt. Und jetzt sind wir keinen
Schritt weiter. Wie will sich denn jemand mit Umkehrbarkeit von
Funktionen beschäftigen, wenn er noch ncht mal die Definition der
Bijektivität versteht?

hs
IV
2017-02-23 17:44:34 UTC
Permalink
Post by H0Iger SchuIz
Post by IV
Mir scheint, die Umkehrbarkeit durch eine Elementare Funktion ist
lediglich eine Eigenschaft des Funktionsterms - unabhängig vom
Definitionsbereich
Nein, eben nicht. Das könnte man erkennen, wenn man sich die Definition
der Bijektivität einmal durchliest.
Nun wirst Du wieder schimpfen, daß ich wieder neue Begriffe erfinde und es
nicht gleich geschrieben habe. Ich meine die "potentielle" elementare
Umkehrbarkeit aus Ritts Satz. Ritts Satz sagt doch: "if F(z) and its inverse
are both elementary", dann exisitiert für F eine lineare
expln-Verkettungsdarstellung. Er sagt nichts über die "tatsächliche"
elementare Umkehrbarkeit einer gegebenen Funktion.
Eine gegebene Verkettungsdarstellung existiert (bis auf Ausnahmen:
unterschiedliche Gültigkeitsbereiche der einzelnen Potenz- und
Logarithmengesetze) unabhängig vom Definitionsbereich, denn die
Verkettungsdarstellung ist ja ein Funktionsterm. Er kann zu
unterschiedlichen Funktionen gehören. Diese unterscheiden sich in ihrem
Definitionsbereich. Deshalb ist die "potentielle" elementare Umkehrbarkeit
(bis auf Ausnahmen) unabhängig vom Definitionsbereich.
Ob die gegebene _potentiell_ elementar umkehrbare Funktion dann
_tatsächlich_ elementar umkehrbar ist, hängt davon ab ob sie bijektiv ist
oder nicht. Die Bijektivität jedoch hängt vom Definitionsbereich ab: Von
zwei Funktionen mit demselben Funktionsterm aber unterschiedlichem
Definitionsbereich könnte die eine bijektiv sein und die andere nicht.
Torn Rumero DeBrak
2017-02-23 19:03:14 UTC
Permalink
Post by IV
Post by H0Iger SchuIz
Post by IV
Mir scheint, die Umkehrbarkeit durch eine Elementare Funktion ist
lediglich eine Eigenschaft des Funktionsterms - unabhängig vom
Definitionsbereich
Nein, eben nicht. Das könnte man erkennen, wenn man sich die
Definition der Bijektivität einmal durchliest.
Nun wirst Du wieder schimpfen, daß ich wieder neue Begriffe erfinde und
es nicht gleich geschrieben habe. Ich meine die "potentielle" elementare
Umkehrbarkeit aus Ritts Satz. Ritts Satz sagt doch: "if F(z) and its
inverse are both elementary", dann exisitiert für F eine lineare
expln-Verkettungsdarstellung. Er sagt nichts über die "tatsächliche"
elementare Umkehrbarkeit einer gegebenen Funktion.
Du verstehst den Satz einfach nicht:
Die Existenz einer Umkehrfunktion zu F ist NOTWENDIGE Voraussetzung,
damit die Aussage des Satzes überhaupt gültig ist.
Erst dann, wenn es diese Umkehrfunktion gibt, kann man auch
eine "lineare expln-Verkettungsdarstellung" finden.

Du hast bis jetzt immer noch nicht gesagt, was denn nun "linear"
daran sein soll. (siehe mein vorherigen Postings
Post by IV
unterschiedliche Gültigkeitsbereiche der einzelnen Potenz- und
Logarithmengesetze) unabhängig vom Definitionsbereich, denn die
Verkettungsdarstellung ist ja ein Funktionsterm. Er kann zu
unterschiedlichen Funktionen gehören. Diese unterscheiden sich in ihrem
Definitionsbereich. Deshalb ist die "potentielle" elementare
Umkehrbarkeit (bis auf Ausnahmen) unabhängig vom Definitionsbereich.
Ob die gegebene _potentiell_ elementar umkehrbare Funktion dann
_tatsächlich_ elementar umkehrbar ist, hängt davon ab ob sie bijektiv
Von zwei Funktionen mit demselben Funktionsterm aber unterschiedlichem
Definitionsbereich könnte die eine bijektiv sein und die andere nicht.
Was verstehst du dann unter "Umkehrbarkeit"?

Eine Funktion F heißt umkehrbar, wenn nicht nur jedem Argument
eindeutig ein Funktionswert zugeordnet ist, sondern auch umgekehrt zu
jedem Funktionswert genau ein Argument gehört.

D.h. man kann dann eine Umkehrfunktion F^-1 konstruieren, für die

1) F^-1 o F = id_D und
2) F o F^-1 = id_W

gelten (wobei id_D die Identitat auf dem Definitionsbereich ist
und id_W die Identität auf dem Wertebereich.

Was verstehst du da jetzt unterschiedlich?


BTW:
Da ja id_D und id_W bijektiv sind folgt aus 1) ,
daß F^-1 surjektiv und F injektiv sein müssen,
und aus 2) folgt, daß F surjektiv und F^-1 injektiv
sein müssen.
Zusammen heißt das aber, daß F und F^-1 beide bijektiv sein müssen.
Anders kann es keine Umkehrfunktion geben.
(die Umkehrfunktion ist dann sogar eindeutig bestimmt; es kann also
zu einer Funktion nicht mehrere Umkehrfunktionen geben)

Oder siehst du das anders?
Carlo XYZ
2017-02-23 19:47:00 UTC
Permalink
Post by Torn Rumero DeBrak
Eine Funktion F heißt umkehrbar, wenn nicht nur jedem Argument
eindeutig ein Funktionswert zugeordnet ist, sondern auch umgekehrt zu
jedem Funktionswert genau ein Argument gehört.
Beispiel A={a}, B={b,c}, f:A->B, a\mapsto b. Nennst du f umkehrbar?
Ich habe nicht unbedingt etwas dagegen, aber IV wird verwirrt.

H0Iger SchuIz
2017-02-21 15:46:50 UTC
Permalink
IV <***@onlinehome.de> wrote:

[...]
Post by IV
Denjenigen, die wesentliche Beiträge liefern, biete ich die Mitautorschaft
an dem zu erstellenden Artikel in einer mathematischen Fachzeitschrift (in
Deutsch und in Englisch) an.
Von welcher Zeitschrift hast du den schon eine Zusage?
Post by IV
Die Begriffe Umkehrung einer Verkettung, Verkettung und
Verkettungsdarstellung müssen noch genauer definiert und präziser eingesetzt
werden.
Aha.
Post by IV
Eine Gleichung E(y) = x, worin E eine elementare Funktion, y die komplexe
Lösungsvariable und x eine komplexe Variable sind, hat genau dann eine
elementare Funktion y als Lösung, wenn die Funktion E eine elementare
Umkehrfunktion hat.
Was ist denn nun y? Eine "komplexe Lösungsvariable" oder eine elementare
Funktion?
Post by IV
E^-1: Umkehrfunktion der Funktion E
Zuordnungsvorschrift der elementaren Funktion E: E(x) = F(x)
(1)
Definitionsgleichung der Umkehrfunktion E^-1: E(E^-1(x)) = x
(2)
Transformation y = E^-1(x)
(3)
Einsetzen von (2) in Gleichung (1): E(y) = x
(4)
Gleichung (4) ist die Gleichung aus Satz 2.
Verwenden von Gleichung (1) in Gleichung (4): F(y) =x
(5)
Auflösen von Gleichung (2) nach der Lösungsvariablen y: y = F^-1(x) (6)
Rücktransformation (3): E^-1(x) = F^-1(x)
(7)
Die Lösung der Gleichung aus Satz 2 ist genau die Umkehrfunktion E^-1.
Daraus folgt Satz 2.
Aus dem Gestammel folgt schon mal gar nichts. Ohen auf mathematische
Details einzugehen, ist der Versuch vollständig mit prädikatfreien
Sätzen formulieren zu wollen, keine gute Idee.

So wird auch die Struktur des Beweises nicht klar. Es wird eine
Äquivalenz behauptet. Um diese zu zeigen, muss man entweder durchgängig
Äquivalenzen verwenden, um von der einen Aussage zur anderen zu kommen,
oder man zeigt zwei Implikationen. Was soll das hier sein?
Post by IV
Wir wollen jetzt Ritts Satz ([Ritt 1925]) beweisen.
Satz 3 (Satz über die Form elementar umkehrbarer elementarer Funktionen
Wenn die Funktion F und ihre Umkehrfunktion beide elementar sind, dann
existieren n Funktionen F_1, F_2, ..., F_n, wo jedes F_i mit einem ungeraden
Index i eine algebraische Funktion ist, und jedes F_i mit einem geraden
Index i entweder exp oder ln ist, so dass F(z) =
F_n(F_{n-1}(...(F_2(F_1(z)))...)).
a) Satz 4 besagt, dass jede bijektive lineare expln-Verkettung elementar
umkehrbar ist.
b) Satz 5 besagt, dass keine nicht-lineare expln-Verkettung elementar
umkehrbar ist.
c) Aus a und b folgt, dass nur lineare expln-Verkettungen elementar
umkehrbar sind.
Aus c und dem Rittschen Konstruktionsprinzip
Das findet man in etwa wo? Da es wohl eine wichtige Rolle im Beweis
einnimmt, sollte man es nicht einfach so dazwischenschieben.

Wie jetzt damit der Satz bewiesen sein soll, wird mir nicht klar, da die
von Ritt behauptete Darstellung weder hier noch in den nachfolgenden
Sätzen überhaupt vorkommt. Wo wird deren Existenz gezeigt?
Post by IV
der elementaren Funktionen
folgt die Formeldarstellung in Ritts Satz (Satz 3).
Ist die Funktion F eine bijektive lineare expln-Verkettung, dann ist F
elementar umkehrbar.
Was soll das heißen, das eine Funktion eine Verkettung _ist_? Das eine
Funktion eine bestimmte Darstellung _hat_ ist durchaus verständlich.
Hier meine ich aber Funktion und Darstellung werden in unzulässiger
Weise miteinander identifiziert.
Post by IV
Satz 4 ergibt sich daraus, dass die Umkehrfunktion einer bijektiven linearen
Verkettung ebenfalls eine lineare Verkettung ist.
In Erweiterung von [Ritt 1925] läßt sich auch sofort die Umkehrfunktion
einer bijektiven linearen Verkettung angeben.
Nur wenn man die entsorechende Verkettungsdarstellung, die man
"umkehren" möchte, auch kennt. Zunächst sagt Ritts Satz etwas über die
Existenz einer bestimmten Darstellung aus. Das heißt nicht, dass man sie
konkret hinschreiben kann.
Post by IV
Das wäre ein weiterer Satz.
Ist die Funktion F eine nicht-lineare expln-Verkettung, dann ist F nicht
elementar umkehrbar.
Dass ein Funktion eine solche Darstellung hat, schließt nicht aus, dass
sie noch andere Darstellungen haben kann. Daher ist die Formulierung,
die Funktion sei eine Verkettung nicht zulässig.
Post by IV
F ist gemäß Voraussetzung von Satz 5 eine bijektive nicht-lineare
expln-Verkettung.
Wo wird da die Bijektivität vorausgesetzt?
Post by IV
Ist F nicht bijektiv, dann ist F nicht umkehrbar und damit nicht elementar
umkehrbar.
Ist F bijektiv, dann existiert die Umkehrfunktion zu F.
Die Umkehrung einer Verkettung ist die Verkettung der Umkehrfunktionen der
Komponenten der Verkettung in umgekehrter Reihenfolge. Da F eine
nicht-lineare Verkettung ist, enthält die expln-Verkettungsdarstellung von F
mindestens eine mehrstellige algebraische Funktion. Die Umkehrfunktion einer
mehrstelligen algebraischen Funktion ist eine mehrwertige Funktion. Eine
mehrwertige Funktion kann laut Definition der elementaren Funktionen
(Definition 1) jedoch keine elementare Funktion sein. Deshalb kann die
Umkehrung der Verkettungsdarstellung von F keine elementare Funktion sein.
Da F eine nicht-lineare expln-Verkettung ist, besitzt F keine lineare
Verkettungsdarstellung, sondern nur nicht-lineare Verkettungsdarstellungen.
Entsprechend dem im vorigen Absatz Gesagten kann keine dieser nicht-linearen
Verkettungsdarstellungen von F eine elementare Umkehrung haben. Da F aber
keine lineare Verkettungsdarstellung besitzt, kann keine der
Verkettungsdarstellungen der Umkehrfunktion von F eine elementare Funktion
sein. Die Umkehrfunktion von F kann daher keine elementare Funktion sein.
Dieser "Beweis" setzt voraus, dass es keine "lineare" Verkettung gibt,
nicht, dass eine Funktion als "nicht-lineare" Verkettung darstellbar
ist. Womöglich basiert die Beweisidee nur auf ungenauen Formulierungen.
Post by IV
Weiter definiere ich die Klasse der standardfunktionbasierten Funktionen.
Eine Funktion ist genau dann eine Standardfunktion, wenn definiert ist, dass
sie eine Standardfunktion ist.
Sorry, jetzt wird's albern. Das soll doch wohl keine Definition sein?
Prrr.
Post by IV
Eine Funktion ist genau dann standardfunktionbasiert, also eine
standardfunktionbasierte Funktion, wenn sie eine Verkettung endlich vieler
Funktionen ist, von denen jede der Menge {ein- oder mehrstellige
algebraische Funktionen, {einstellige einwertige Standardfunktionen}}
entstammt.
Wenn die Funktion F und ihre Umkehrfunktion beide standardfunktionbasiert
sind, dann ist F eine Verkettung endlich vieler Funktionen, von denen jede
der Menge {einstellige algebraische Funktionen, {einstellige einwertige
Standardfunktionen}} entstammt.´
Dieser "Satz" verwendet doch nur die "Definition". Was soll das?

hs
IV
2017-02-21 17:31:59 UTC
Permalink
Post by H0Iger SchuIz
Post by IV
Die Begriffe Umkehrung einer Verkettung, Verkettung und
Verkettungsdarstellung müssen noch genauer definiert und präziser
eingesetzt werden.
Aha.
Wir Nichtmathematiker und Ritt würden ja exp(x^2) bereits als Funktion
bezeichnen. Damit wären Aussagen zu "Form", "Darstellung", "Struktur" dieses
Ausdrucks sofort als Eigenschaften der Funktion erkennbar.
Wie aber nennt die moderne Mathematik den Ausdruck exp(x^2), wenn die
Funktion die Funktion F ist mit x \mapsto F(x)=exp(x^2)?
Ich muß unterscheiden zwischen der Funktion, dem Ausdruck exp(x^2),
_ihrer_/_seiner_ Verkettungsdarstellung, _einer_ Verkettungsdarstellung,
einer Verkettung, ...
Dafür muß ich erst die Begriffe definieren.
Post by H0Iger SchuIz
Post by IV
Eine Gleichung E(y) = x, worin E eine elementare Funktion, y die komplexe
Lösungsvariable und x eine komplexe Variable sind, hat genau dann eine
elementare Funktion y als Lösung, wenn die Funktion E eine elementare
Umkehrfunktion hat.
Was ist denn nun y? Eine "komplexe Lösungsvariable" oder eine elementare
Funktion?
Auch hier muß ich mich wohl entscheiden zwischen "Unbekannte",
"Lösungsvariable" und "Funktion".
Post by H0Iger SchuIz
Aus dem Gestammel folgt schon mal gar nichts.
...
Was soll das hier sein?
Der zu beweisende Satz 2 ist "Stand der Wissenschaft", auf dessen Beweis
kommt es hier im Moment nicht an.
Post by H0Iger SchuIz
Post by IV
Wir wollen jetzt Ritts Satz ([Ritt 1925]) beweisen.
...
Aus c und dem Rittschen Konstruktionsprinzip
Das findet man in etwa wo? Da es wohl eine wichtige Rolle im Beweis
einnimmt, sollte man es nicht einfach so dazwischenschieben.
Das ist in Ritts Artikel zu finden. Ich habe hier noch nicht alles haarklein
aufgeschrieben.
Post by H0Iger SchuIz
Wie jetzt damit der Satz bewiesen sein soll, wird mir nicht klar, da die
von Ritt behauptete Darstellung weder hier noch in den nachfolgenden
Sätzen überhaupt vorkommt. Wo wird deren Existenz gezeigt?
Ich habe hier noch nicht alles haarklein aufgeschrieben.
Die Verkettungsdarstellung steckt im Konstruktionsprinzip der Elementaren
Funktionen bei Ritt und Liouville. Ritt nennt es "Liouvilles Methode".
Die Verkettungsdarstellung ist bei mir in Satz 1 d -f zu finden.
Ritt konstruiert die Elementaren Funktionen in der "linearen"
Verkettungsdarstellung abwechselnd aus exp/ln-Funktionen und algebraischen
Funktionen. Ich nehme an, dadurch will er redundante Ausdrücke wie ln o exp
oder exp o ln verhindern. Und da jede algebraische Funktion algebraischer
Funktionen wieder eine algebraische Funktion ist, ist die Zusammenfassung
einer Komposition lediglich algebraischer Funktionen zu einer nicht
zusammengesetzten algebraischen Funktion als Zwischenglied legitim.
Da die Identitätsfunktion auch eine algebraische Funktion ist, entspricht
Ritts lineare Verkettungsdarstellung in seinem Satz meiner in meinem Satz 1.
Nur, die von Ritt vorgegebene Form der Verkettung ist unnötig starr, meine
Vorgaben sind dagegen auf das Nötigste beschränkt: eine lineare Verkettung
zu sein.
Post by H0Iger SchuIz
Post by IV
Ist die Funktion F eine bijektive lineare expln-Verkettung, dann ist F
elementar umkehrbar.
Was soll das heißen, dass eine Funktion eine Verkettung _ist_? Dass eine
Funktion eine bestimmte Darstellung _hat_ ist durchaus verständlich. Hier
meine ich aber Funktion und Darstellung werden in unzulässiger Weise
miteinander identifiziert.
Dieselbe Antwort wie ganz oben: Ich muß noch genauer zwischen der Funktion
und ihrer Darstellung unterscheiden, dazu muß ich mir aber erst noch die
entsprechenden Begriffe definieren.
Eine Verkettung (= Funktionskomposition) _ist_ eine Funktion.
Post by H0Iger SchuIz
Post by IV
In Erweiterung von [Ritt 1925] läßt sich auch sofort die Umkehrfunktion
einer bijektiven linearen Verkettung angeben.
Nur wenn man die entsprechende Verkettungsdarstellung, die man "umkehren"
möchte, auch kennt. Zunächst sagt Ritts Satz etwas über die Existenz einer
bestimmten Darstellung aus. Das heißt nicht, dass man sie
konkret hinschreiben kann.
Die Umkehrfunktion einer bijektiven linearen Verkettung ist doch die
Verkettung der Umkehrfunktionen der Komponentenfunktionen (Gliedfunktionen?)
der ursprünglichen Funktion in umgekehrter Reihenfolge.
Post by H0Iger SchuIz
Post by IV
Ist die Funktion F eine nicht-lineare expln-Verkettung, dann ist F nicht
elementar umkehrbar.
Dass eine Funktion eine solche Darstellung hat, schließt nicht aus, dass
sie noch andere Darstellungen haben kann. Daher ist die Formulierung, die
Funktion sei eine Verkettung nicht zulässig.
Bedenke wieder meine noch fehlenden Definitionen für diese Begriffe.
Post by H0Iger SchuIz
Post by IV
F ist gemäß Voraussetzung von Satz 5 eine bijektive nicht-lineare
expln-Verkettung.
Wo wird da die Bijektivität vorausgesetzt?
Stimmt. Danke. Es muß heißen: "F ist gemäß Voraussetzung von Satz 5 eine
nicht-lineare expln-Verkettung." Und im Anschluß daran werden im Beweis ja
die Fälle Nicht-Bijektivität und Bijektivität abgehandelt.
Post by H0Iger SchuIz
Post by IV
...
Ist F nicht bijektiv, dann ist F nicht umkehrbar und damit nicht
elementar umkehrbar.
Ist F bijektiv, dann existiert die Umkehrfunktion zu F.
Die Umkehrung einer Verkettung ist die Verkettung der Umkehrfunktionen
der Komponenten der Verkettung in umgekehrter Reihenfolge. Da F eine
nicht-lineare Verkettung ist, enthält die expln-Verkettungsdarstellung
von F mindestens eine mehrstellige algebraische Funktion. Die
Umkehrfunktion einer mehrstelligen algebraischen Funktion ist eine
mehrwertige Funktion. Eine mehrwertige Funktion kann laut Definition der
elementaren Funktionen (Definition 1) jedoch keine elementare Funktion
sein. Deshalb kann die Umkehrung der Verkettungsdarstellung von F keine
elementare Funktion sein.
Da F eine nicht-lineare expln-Verkettung ist, besitzt F keine lineare
Verkettungsdarstellung, sondern nur nicht-lineare
Verkettungsdarstellungen.
Entsprechend dem im vorigen Absatz Gesagten kann keine dieser
nicht-linearen Verkettungsdarstellungen von F eine elementare Umkehrung
haben. Da F aber keine lineare Verkettungsdarstellung besitzt, kann keine
der Verkettungsdarstellungen der Umkehrfunktion von F eine elementare
Funktion sein. Die Umkehrfunktion von F kann daher keine elementare
Funktion sein.
Dieser "Beweis" setzt voraus, dass es keine "lineare" Verkettung gibt,
nicht, dass eine Funktion als "nicht-lineare" Verkettung darstellbar ist.
Womöglich basiert die Beweisidee nur auf ungenauen Formulierungen.
Ich muß noch sauberer trennen zwischen "eine lineare Verkettung ist" und
"eine lineare Verkettungsdarstellung hat".
Eine Verkettung (die Funktion) ist genau dann nicht-linear, wenn für sie
keine lineare Verkettungsdarstellung existiert.
Post by H0Iger SchuIz
Post by IV
Weiter definiere ich die Klasse der standardfunktionbasierten Funktionen.
Eine Funktion ist genau dann eine Standardfunktion, wenn definiert ist,
dass sie eine Standardfunktion ist.
Sorry, jetzt wird's albern. Das soll doch wohl keine Definition sein?
Prrr.
Wie kann ich's besser formulieren? Es soll eine Menge von Funktionen
vorgegeben sein. Und jedes Element dieser Menge soll eine Standardfunktion
heißen.
Post by H0Iger SchuIz
Post by IV
Eine Funktion ist genau dann standardfunktionbasiert, also eine
standardfunktionbasierte Funktion, wenn sie eine Verkettung endlich
vieler Funktionen ist, von denen jede der Menge {ein- oder mehrstellige
algebraische Funktionen, {einstellige einwertige Standardfunktionen}}
entstammt.
Wenn die Funktion F und ihre Umkehrfunktion beide standardfunktionbasiert
sind, dann ist F eine Verkettung endlich vieler Funktionen, von denen
jede der Menge {einstellige algebraische Funktionen, {einstellige
einwertige Standardfunktionen}} entstammt.
Dieser "Satz" verwendet doch nur die "Definition". Was soll das?
aus der Definition: {ein- oder mehrstellige algebraische Funktionen, ...}
aus dem Satz: {einstellige algebraische Funktionen, ...}
Es geht in Analogie zu Ritts Satz um den Unterschied zwischen linearen
Verkettungen (von Standarfunktionen) und nicht-linearen Verkettungen (von
Standardfunktionen).
H0Iger SchuIz
2017-02-21 17:47:19 UTC
Permalink
Post by IV
Post by H0Iger SchuIz
Post by IV
Die Begriffe Umkehrung einer Verkettung, Verkettung und
Verkettungsdarstellung müssen noch genauer definiert und präziser
eingesetzt werden.
Aha.
Wir Nichtmathematiker
Das ist 'ne ziemlich große Gruppe von Menschen, zu deren Sprecher du
dich da machst. Allerdings ist sie nicht relevant. Es geht halt um
Mathematik.
Post by IV
und Ritt würden ja exp(x^2) bereits als Funktion
bezeichnen.
Würde er das heute auch noch tun?
Post by IV
Damit wären Aussagen zu "Form", "Darstellung", "Struktur" dieses
Ausdrucks sofort als Eigenschaften der Funktion erkennbar.
Wie aber nennt die moderne Mathematik den Ausdruck exp(x^2), wenn die
Funktion die Funktion F ist mit x \mapsto F(x)=exp(x^2)?
Einen solchen Term kann man als Funktionsterm bezeichnen. Aber Obacht,
ohne Bindung des Arguments hat so ein Funktionsterm keine Bedeutung. Was
spricht dagegen eine Funktionsvorschrift als Funktionsvorschrift zu
notieren?
Post by IV
Ich muß unterscheiden zwischen der Funktion, dem Ausdruck exp(x^2),
_ihrer_/_seiner_ Verkettungsdarstellung, _einer_ Verkettungsdarstellung,
einer Verkettung, ...
Dafür muß ich erst die Begriffe definieren.
Makeitso!
Post by IV
Post by H0Iger SchuIz
Post by IV
Eine Gleichung E(y) = x, worin E eine elementare Funktion, y die komplexe
Lösungsvariable und x eine komplexe Variable sind, hat genau dann eine
elementare Funktion y als Lösung, wenn die Funktion E eine elementare
Umkehrfunktion hat.
Was ist denn nun y? Eine "komplexe Lösungsvariable" oder eine elementare
Funktion?
Auch hier muß ich mich wohl entscheiden zwischen "Unbekannte",
"Lösungsvariable" und "Funktion".
Post by H0Iger SchuIz
Aus dem Gestammel folgt schon mal gar nichts.
...
Was soll das hier sein?
Der zu beweisende Satz 2 ist "Stand der Wissenschaft"
Aha.
Post by IV
, auf dessen Beweis
kommt es hier im Moment nicht an.
Dann würde ich auch keinen Nicht-Beweis anführen.
Post by IV
Post by H0Iger SchuIz
Post by IV
Wir wollen jetzt Ritts Satz ([Ritt 1925]) beweisen.
...
Aus c und dem Rittschen Konstruktionsprinzip
Das findet man in etwa wo? Da es wohl eine wichtige Rolle im Beweis
einnimmt, sollte man es nicht einfach so dazwischenschieben.
Das ist in Ritts Artikel zu finden. Ich habe hier noch nicht alles haarklein
aufgeschrieben.
So kann man das dann halt nicht nachvollziehen.
Post by IV
Post by H0Iger SchuIz
Wie jetzt damit der Satz bewiesen sein soll, wird mir nicht klar, da die
von Ritt behauptete Darstellung weder hier noch in den nachfolgenden
Sätzen überhaupt vorkommt. Wo wird deren Existenz gezeigt?
Ich habe hier noch nicht alles haarklein aufgeschrieben.
So kann man das dann halt nicht nachvollziehen.
Post by IV
Die Verkettungsdarstellung steckt im Konstruktionsprinzip der Elementaren
Funktionen bei Ritt und Liouville. Ritt nennt es "Liouvilles Methode".
Die Verkettungsdarstellung ist bei mir in Satz 1 d -f zu finden.
Ritt konstruiert die Elementaren Funktionen in der "linearen"
Verkettungsdarstellung abwechselnd aus exp/ln-Funktionen und algebraischen
Funktionen. Ich nehme an, dadurch will er redundante Ausdrücke wie ln o exp
oder exp o ln verhindern. Und da jede algebraische Funktion algebraischer
Funktionen wieder eine algebraische Funktion ist, ist die Zusammenfassung
einer Komposition lediglich algebraischer Funktionen zu einer nicht
zusammengesetzten algebraischen Funktion als Zwischenglied legitim.
Da die Identitätsfunktion auch eine algebraische Funktion ist, entspricht
Ritts lineare Verkettungsdarstellung in seinem Satz meiner in meinem Satz 1.
Nur, die von Ritt vorgegebene Form der Verkettung ist unnötig starr, meine
Vorgaben sind dagegen auf das Nötigste beschränkt: eine lineare Verkettung
zu sein.
Ritt behauptet in seinem Satz eine bestimmte Darstellung. Wenn man also
Ritts Satz beweisen will, sollte es um diese gehen. Je weniger "starr"
man diese Darstellung macht, umso schwächer wird die Aussage des Satzes.

Bist du dir sicher, dass du die Aussage von Ritts Satz verstanden hast?
Post by IV
Post by H0Iger SchuIz
Post by IV
Ist die Funktion F eine bijektive lineare expln-Verkettung, dann ist F
elementar umkehrbar.
Was soll das heißen, dass eine Funktion eine Verkettung _ist_? Dass eine
Funktion eine bestimmte Darstellung _hat_ ist durchaus verständlich. Hier
meine ich aber Funktion und Darstellung werden in unzulässiger Weise
miteinander identifiziert.
Dieselbe Antwort wie ganz oben: Ich muß noch genauer zwischen der Funktion
und ihrer Darstellung unterscheiden, dazu muß ich mir aber erst noch die
entsprechenden Begriffe definieren.
Eine Verkettung (= Funktionskomposition) _ist_ eine Funktion.
Post by H0Iger SchuIz
Post by IV
In Erweiterung von [Ritt 1925] läßt sich auch sofort die Umkehrfunktion
einer bijektiven linearen Verkettung angeben.
Nur wenn man die entsprechende Verkettungsdarstellung, die man "umkehren"
möchte, auch kennt. Zunächst sagt Ritts Satz etwas über die Existenz einer
bestimmten Darstellung aus. Das heißt nicht, dass man sie
konkret hinschreiben kann.
Die Umkehrfunktion einer bijektiven linearen Verkettung ist doch die
Verkettung der Umkehrfunktionen der Komponentenfunktionen (Gliedfunktionen?)
der ursprünglichen Funktion in umgekehrter Reihenfolge.
Wenn die alle Umkehrfunktionen haben, d.h. wenn die alle bijketiv sind.
Post by IV
Post by H0Iger SchuIz
Post by IV
Ist die Funktion F eine nicht-lineare expln-Verkettung, dann ist F nicht
elementar umkehrbar.
Dass eine Funktion eine solche Darstellung hat, schließt nicht aus, dass
sie noch andere Darstellungen haben kann. Daher ist die Formulierung, die
Funktion sei eine Verkettung nicht zulässig.
Bedenke wieder meine noch fehlenden Definitionen für diese Begriffe.
Also eigentlich ist noch gar nichts klar, weil noch nichts definiert
ist.

Plopp.

hs
IV
2017-02-21 19:54:40 UTC
Permalink
Post by H0Iger SchuIz
Post by IV
Die Umkehrfunktion einer bijektiven linearen Verkettung ist doch die
Verkettung der Umkehrfunktionen der Komponentenfunktionen
(Gliedfunktionen?) der ursprünglichen Funktion in umgekehrter
Reihenfolge.
Wenn die alle Umkehrfunktionen haben, d.h. wenn die alle bijketiv sind.
Wir hatten doch hier in einem anderen Thread schon festgestellt, daß aus der
Bijektivität einer (linearen) Komposition mehrerer Funktionen die
Bijektivität jedes einzelnen Gliedes folgt.
Torn Rumero DeBrak
2017-02-21 20:18:42 UTC
Permalink
Post by IV
Post by H0Iger SchuIz
Post by IV
Die Umkehrfunktion einer bijektiven linearen Verkettung ist doch die
Verkettung der Umkehrfunktionen der Komponentenfunktionen
(Gliedfunktionen?) der ursprünglichen Funktion in umgekehrter
Reihenfolge.
Wenn die alle Umkehrfunktionen haben, d.h. wenn die alle bijketiv sind.
Wir hatten doch hier in einem anderen Thread schon festgestellt, daß aus
der Bijektivität einer (linearen) Komposition mehrerer Funktionen die
Bijektivität jedes einzelnen Gliedes folgt.
Nein. Das hatten wir nicht. Das Resultat war:

Ist g ∘ f bijektiv, dann ist f injektiv und g surjektiv.

Mehr kann man über f und g nicht sagen.

Es gibt also Beispiele, wo weder f noch g bijektiv sind.
IV
2017-02-21 23:40:56 UTC
Permalink
Post by Torn Rumero DeBrak
Post by IV
Wir hatten doch hier in einem anderen Thread schon festgestellt, daß aus
der Bijektivität einer (linearen) Komposition mehrerer Funktionen die
Bijektivität jedes einzelnen Gliedes folgt.
Ist g ∘ f bijektiv, dann ist f injektiv und g surjektiv.
Mehr kann man über f und g nicht sagen.
Es gibt also Beispiele, wo weder f noch g bijektiv sind.
Was aber kann man über die Injektivität, Surjektivität und Bijektivität der
Glieder in F = f3 o f2 o f1 sagen, wenn F bijektiv ist?
Und was kann man über die Umkehrfunktion von F sagen?
Torn Rumero DeBrak
2017-02-22 00:49:44 UTC
Permalink
Post by IV
Post by Torn Rumero DeBrak
Post by IV
Wir hatten doch hier in einem anderen Thread schon festgestellt, daß
aus der Bijektivität einer (linearen) Komposition mehrerer Funktionen
die Bijektivität jedes einzelnen Gliedes folgt.
Ist g ∘ f bijektiv, dann ist f injektiv und g surjektiv.
Mehr kann man über f und g nicht sagen.
Es gibt also Beispiele, wo weder f noch g bijektiv sind.
Was aber kann man über die Injektivität, Surjektivität und Bijektivität
der Glieder in F = f3 o f2 o f1 sagen, wenn F bijektiv ist?
Na, das ist doch allgemeines Grundwissen, dass man
eine mathematische Aussage doch mehrfach anwenden kann:

Es gilt folgendes:

F = f3 o (f2 o f1) = (f3 o f2) o f1

nach dem Assoziativgesetz für Komposition.

Jetzt wende die Originalaussage auf die
einzeln stehende Funktion und die geklammerten
Funktionen (aufgefasst als eine Funktion) an.
Das ist für beliebig viele endlich Kompositionsketten durchführbar.

Das habe ich aber - wenn ich mich recht erinnere - dir
schon einmal geschrieben:

f3 ist surjektiv, (f2 o f1) ist injektiv,
(f3 o f2) ist surjektiv und f1 ist injektiv.
Folglich ist dann f2 bijektiv.


Analoges Beispiel ist die erweiterte Binomische Formel:

Was ist (a + b + c)^2, wenn (x + y)^2 = x^2 + 2xy + y^2 ist?

Setze einfach (b + c) = d, und verwende das Assozitivgesetz,
die ursprüngliche, zweikomponentige binomische Formel und das
Distributivgesetz.

(a + b + c)^2 = (a + (b + c))^2
= (a + d)^2 = a^2 + 2ad +d^2
= a^2 + 2a(b + c) + (b + c)^2
= a^2 + 2ab + 2ac + b^2 + 2bc + c^2
Post by IV
Und was kann man über die Umkehrfunktion von F sagen?
F^-1 ist bijektiv und
F^-1 o F und F o F^-1 sind die jeweiligen Identitäten auf
ihren jeweiligen Definitionsbereichen.
Carlo XYZ
2017-02-22 03:02:21 UTC
Permalink
Post by Torn Rumero DeBrak
F = f3 o (f2 o f1) = (f3 o f2) o f1
..
f3 ist surjektiv, (f2 o f1) ist injektiv,
(f3 o f2) ist surjektiv und f1 ist injektiv.
Folglich ist dann f2 bijektiv.
Gegenbeispiel A={a}, B={b,c}

f1:A->A, a\mapsto a, f1 bijektiv

f2:A->B, a\mapsto b, f2 nicht surjektiv

f3:B->A, b\mapsto a und c\mapsto a, f3 nicht injektiv

f1of2of3 und f2of3of1 (relationale Schreibweise) beide bijektiv
Torn Rumero DeBrak
2017-02-22 05:34:26 UTC
Permalink
Post by Carlo XYZ
Post by Torn Rumero DeBrak
F = f3 o (f2 o f1) = (f3 o f2) o f1
..
f3 ist surjektiv, (f2 o f1) ist injektiv,
(f3 o f2) ist surjektiv und f1 ist injektiv.
Folglich ist dann f2 bijektiv.
Gegenbeispiel A={a}, B={b,c}
f1:A->A, a\mapsto a, f1 bijektiv
f2:A->B, a\mapsto b, f2 nicht surjektiv
f3:B->A, b\mapsto a und c\mapsto a, f3 nicht injektiv
f1of2of3 und f2of3of1 (relationale Schreibweise) beide bijektiv
Sorry, du hast Recht. Ich hatte fälschlich gedacht, aus
(f2 o f1) injektiv/surjektiv folgt, f1 und f2 injektiv/surjektiv.
Es gilt jedoch nur die umgekehrte Implikation:
Wenn f1 und f2 injektiv/surjektiv, dann (f2 o f1) injektiv/surjektiv.

Richtig ist aber, dass

wenn (f2 o f1) injektiv, dann ist f1 injektiv, und
wenn (f2 o f1) surjektiv, dann ist f2 surjektiv.

Im ursprünglichen Fall, dass F = f3 o f2 o f1 bijektiv ist,
kann man nur schließen, dass f3 surjektiv und f1 injektiv ist.
Über f2 ist nichts bekannt.

BTW: Bei mir gilt durchgehend f2 o f1 (x) = f2(f1(x)) .
H0Iger SchuIz
2017-02-22 06:51:59 UTC
Permalink
Post by Torn Rumero DeBrak
Im ursprünglichen Fall, dass F = f3 o f2 o f1 bijektiv ist,
kann man nur schließen, dass f3 surjektiv und f1 injektiv ist.
Über f2 ist nichts bekannt.
Grob: Je weiter "innen" die Komponenten liegen, umso weniger weiß man.

hs
Carlo XYZ
2017-02-22 10:15:31 UTC
Permalink
Post by H0Iger SchuIz
Post by Torn Rumero DeBrak
Im ursprünglichen Fall, dass F = f3 o f2 o f1 bijektiv ist,
kann man nur schließen, dass f3 surjektiv und f1 injektiv ist.
Über f2 ist nichts bekannt.
Grob: Je weiter "innen" die Komponenten liegen, umso weniger weiß man.
So grob, dass es nutzlos ist. Auch mal was Schönes.
Detlef Müller
2017-02-22 13:37:15 UTC
Permalink
Am 20.02.2017 um 22:30 schrieb IV:

[...]
Post by IV
Analoge Herleitungen sind auch für die Klasse der elementaren Funktionen
möglich, die sich von der Funktionenklasse Ritts darin unterscheidet,
dass nur die expliziten algebraischen Funktionen, nicht aber die
impliziten algebraischen Funktionen dazuzählen.
Weiter definiere ich die Klasse der standardfunktionbasierten Funktionen.
Eine Funktion ist genau dann eine Standardfunktion, wenn definiert ist,
dass sie eine Standardfunktion ist.
Das wurde ja schon etwas präzisiert zu:

Gegeben sei eine Menge S von invertierbaren Funktionen, die
also abgeschlossen gegen die Inversenbildung ist.
Diese Elemente dieser Menge S nennen wir "Standardfunktionen".

Das könnte algebraisch fassbar sein, wenn man einen Funktionenkörper
K mit Konstantenkörper K_0 als Grundmenge hat.
Es gäbe dann eine zusätzliche Verknüpfung außer {+,*}, nämlich die
Komposition "o" und bezüglich dieser ein neutrales Element (die
identische Abbildung) - dann könnte man analog zu Einheiten in Ringen
in K die "bezüglich o invertierbaren" Elemente K^o auszeichnen
(bijektive Funktionen mit f^{-1} in K).
K^o ist dann kein Körper mehr (z.B. die 1 und andere Elemente
aus K_0 sind nicht drinn).
Post by IV
Eine Funktion ist genau dann standardfunktionbasiert, also eine
standardfunktionbasierte Funktion, wenn sie eine Verkettung endlich
vieler Funktionen ist, von denen jede der Menge {ein- oder mehrstellige
algebraische Funktionen, {einstellige einwertige Standardfunktionen}}
entstammt.
Das würde ich dann versuchen in verschiedenen Stufen zu formulieren.
SB_{-1} := {}, (für spätere Defs nützlich)
SB_0 := K_0,
SB_1 := K_0(S),
SB_2 := K_0({f o t , t o f| f in SB_1, t in S}),
SB_3 := K_0({f o t , t o f| f in SB_2, t in S}),
...

und schließlich

sei SB die Vereinigung der SB_i, i in IN_0.

Damit hat man auch eine Art Abstufung, also jedem f in SB
ist eine "Komplexität" ord(f) zugeordnet (nämlich dasjenige
ganzzahlige ord(f) mit
f in SB_{ord(f)} und f nicht in SB_{ord(f)-1}

Weiter dürfte SB ein Körper sein (wäre zu zeigen wäre) und
ist bezgl. "o" abgeschlossen (prüfen!).
Post by IV
Wenn die Funktion F und ihre Umkehrfunktion beide
standardfunktionbasiert sind,
Das wäre dann einfach S in SB^o.
Post by IV
dann ist F eine Verkettung endlich vieler
Funktionen, von denen jede der Menge {einstellige algebraische
Funktionen, {einstellige einwertige Standardfunktionen}} entstammt.
Das wird dann wohl zu:

Behauptung (1):
Für ein s aus S sei K_s := K_0(s) (Rationale Funktionen in
je einer Standardfunktion).
Dann gilt:
Ist F in SB^o, so existiert ein n in IN
und a_i in K_s für i=1,...,n mit F = a_1 o ... o a_n.

Beweis: (?)

Das wäre nun ein Ansatz für ein Grundgerüst von Bezeichnungen
und Definitionen, auf dem man aufbauen könnte, wenn denn der
Ansatz auch so klappt (und nicht irgend welche Folgerungen daraus
zu Widersprüchen oder zum Kollaps der definierten Mengen zu irgend
etwas trivialem führt - was ich mir für gewisse S sehr gut vorstellen
kann).

Ich würde allerdings für die Behauptung (1) in der Allgemeinheit
eher nach einem Gegenbeispiel suchen als nach einem Beweis.

PS: Mir scheint, in Deiner eigentlichen Argumentation benutzt
Du die Aussage, daß die Verknüpfung mehrerer nicht elementarer
Funktionen wieder nicht elementar ist (das habe ich bereits
ausführlicher gepostet).

Das wäre gesondert zu zeigen (imo ist es aber einfach falsch).

Gruß,
Detlef
--
Dr. Detlef Müller,
http://www.mathe-doktor.de oder http://mathe-doktor.de
IV
2017-02-22 17:58:57 UTC
Permalink
Post by IV
Post by IV
Eine Funktion ist genau dann eine Standardfunktion, wenn definiert ist,
dass sie eine Standardfunktion ist.
Zur Definition gehört noch dazu, daß eine Standardfunktion F nur dann
eine standardfunktionbasierte Funktion ist, wenn die Umkehrfunktion von F
ebenfalls eine standardfunktionbasierte Funktion ist.
Gegeben sei eine Menge S von invertierbaren Funktionen, die also
abgeschlossen gegen die Inversenbildung ist. Diese Elemente dieser Menge S
nennen wir "Standardfunktionen".
Das könnte algebraisch fassbar sein, wenn man einen Funktionenkörper K mit
Konstantenkörper K_0 als Grundmenge hat.
Es gäbe dann eine zusätzliche Verknüpfung außer {+,*}, nämlich die
Komposition "o" und bezüglich dieser ein neutrales Element (die identische
Abbildung) - dann könnte man analog zu Einheiten in Ringen in K die
"bezüglich o invertierbaren" Elemente K^o auszeichnen (bijektive
Funktionen mit f^{-1} in K).
K^o ist dann kein Körper mehr (z. B. die 1 und andere Elemente aus K_0
sind nicht drin).
Wenn die Elementaren Funktionen und die Standardfunktionenbasierten
Funktionen abgehandelt sind, eröffnet sich noch eine Erweiterung:
Die Liouvilleschen Funktionen aus Liouvilles "Integration in endlichen
Ausdrücken" und die beiden eben genannten Funktionenklassen unterliegen dem
gleichen Konstruktionsprinzip: der Adjunktion von Funktionen. Und bei allen
drei dieser mathematischen Probleme kann man die Konstruktion der
Funktionenklasse sowohl einfach anhand einer Menge als auch anhand eines
Körpers darstellen.
Post by IV
Eine Funktion ist genau dann standardfunktionbasiert, also eine
standardfunktionbasierte Funktion, wenn sie eine Verkettung endlich
vieler Funktionen ist, von denen jede der Menge {ein- oder mehrstellige
algebraische Funktionen, {einstellige einwertige Standardfunktionen}}
entstammt.
Das würde ich dann versuchen in verschiedenen Stufen zu formulieren.
SB_{-1} := {}, (für spätere Defs nützlich)
SB_0 := K_0,
SB_1 := K_0(S),
SB_2 := K_0({f o t , t o f| f in SB_1, t in S}),
SB_3 := K_0({f o t , t o f| f in SB_2, t in S}),
...
und schließlich
sei SB die Vereinigung der SB_i, i in IN_0.
Damit hat man auch eine Art Abstufung, also jedem f in SB ist eine
"Komplexität" ord(f) zugeordnet (nämlich dasjenige ganzzahlige ord(f) mit
f in SB_{ord(f)} und f nicht in SB_{ord(f)-1}
Weiter dürfte SB ein Körper sein (wäre zu zeigen wäre) und ist bezgl. "o"
abgeschlossen (prüfen!).
Die Konstruktion der standardfunktionbasierten Funktionen wird wohl ganz
analog zu der der Elementaren Funktionen sein.
IV
2017-02-22 18:45:31 UTC
Permalink
Mir scheint, in Deiner eigentlichen Argumentation benutzt Du die Aussage,
daß die Verknüpfung mehrerer nicht elementarer Funktionen wieder nicht
elementar ist (das habe ich bereits ausführlicher gepostet).
Das wäre gesondert zu zeigen (imo ist es aber einfach falsch).
Auch für den Fall, daß meine Argumentation falsch ist, ist schon Einiges
gewonnen: Zumindest bei expln-Verkettungen, die nur eine einzige
nichttriviale algebraische Funktion enthalten, liefert meine Argumentation
eine Aussage. Ich will in diesem Post hier nur diesen einfachen Fall zeigen.

F: die gegebene bijektive Elementare Funktion, eine nicht-lineare
expln-Verkettung (= für F existiert keine lineare
expln-Verkettungsdarstellung)
A: eine algebraische Funktion
f1, f2, f3: jeweils eine lineare expln-Verkettung, alle drei algebraisch
unabhängig voneinander
gegeben: F(x) = A(f1(x),f2(x),f3(x))
y = F^-1(x)
Definitionsgleichung der Umkehrfunktion F^-1 von F:
A(f1(y),f2(y),f3(y)) = x
Um diese Gleichung nach y aufzulösen, muß man (lokale) Umkehrfunktionen A^-1
von A anwenden. A ist aber dreistellig, und jede (lokale) Umkehrfunktion von
A ist deshalb dreiwertig (Ist das richtig?).
Eine mehrwertige Funktion kann aber nicht elementar sein. Deshalb kann diese
Gleichung nicht nach y aufgelöst werden, F^-1 also als elementare Funktion
nicht aus dieser Gleichung berechnet werden.
Da für F nun zwar vielleicht andere expln-Verkettungsdarstellungen
existieren können, aber laut Voraussetzung eben nur nicht-lineare, kann es
_keine_ Definitionsgleichung der Umkehrfunktion F^-1 geben, die nach F^-1
als Elementare Funktion aufgelöst werden kann.
F^-1 ist also überhaupt nicht als Elementare Funktion bestimmbar, also kann
sie nicht als Elementare Funktion existieren. Deshalb kann F^-1 keine
Elementare Funktion sein.

Damit gezeigt, daß z. B. die bijektiven Funktionen mit dem Funktionsterm x +
e^x und die !bijektiven! Funktionen mit dem Funktionsterm x * e^x keine
Elementare Funktion als Umkehrfunktion haben können.
Detlef Müller
2017-02-23 09:44:49 UTC
Permalink
[...]
Post by IV
Auch für den Fall, daß meine Argumentation falsch ist, ist schon Einiges
gewonnen: Zumindest bei expln-Verkettungen, die nur eine einzige
nichttriviale algebraische Funktion enthalten, liefert meine
Argumentation eine Aussage. Ich will in diesem Post hier nur diesen
einfachen Fall zeigen.
F: die gegebene bijektive Elementare Funktion, eine nicht-lineare
expln-Verkettung (= für F existiert keine lineare
expln-Verkettungsdarstellung)
A: eine algebraische Funktion
f1, f2, f3: jeweils eine lineare expln-Verkettung, alle drei algebraisch
unabhängig voneinander
gegeben: F(x) = A(f1(x),f2(x),f3(x))
y = F^-1(x)
A(f1(y),f2(y),f3(y)) = x
Um diese Gleichung nach y aufzulösen, muß man (lokale) Umkehrfunktionen
A^-1 von A anwenden. A ist aber dreistellig, und jede (lokale)
Umkehrfunktion von A ist deshalb dreiwertig (Ist das richtig?).
So eine Funktion müsste dreiwertig sein, ja.
Post by IV
Eine mehrwertige Funktion kann aber nicht elementar sein.
Dies war per Definition gesetzt.
Was nicht verhindert, daß A^(-1) eine sehr einfach Funktion
sein kann.
Die Funktion A^(-1) hat drei (einwertige) Komponentenfunktionen
A^(-1)_1, A^(-1)_2, A^(-1)_3, ich sehe nicht, warum die nicht
elementar sein können.

Man erhält ein Gleichungssystem (GS)

(1) f1(y) = A^(-1)_1(x)
(2) f2(y) = A^(-1)_2(x)
(3) f3(y) = A^(-1)_3(x)
Post by IV
Deshalb kann
diese Gleichung nicht nach y aufgelöst werden, ...
Das ist nicht schlüssig.
Im Gleichungssystem (GS) hat man (es ist überbestimmt)
sogar drei Bestimmungsgleichungen zur Auswahl, wenn z.B.
eine der Funktionen f1, f2 oder f3 elementar umkehrbar ist,
können wir die Gleichung sehr wohl nach y auflösen.

Gruß,
Detlef
--
Dr. Detlef Müller,
http://www.mathe-doktor.de oder http://mathe-doktor.de
IV
2017-02-23 18:12:58 UTC
Permalink
Post by IV
F: die gegebene bijektive Elementare Funktion, eine nicht-lineare
expln-Verkettung (= für F existiert keine lineare
expln-Verkettungsdarstellung)
A: eine algebraische Funktion
f1, f2, f3: jeweils eine lineare expln-Verkettung, alle drei algebraisch
unabhängig voneinander
gegeben: F(x) = A(f1(x),f2(x),f3(x))
y = F^-1(x)
A(f1(y),f2(y),f3(y)) = x
Um diese Gleichung nach y aufzulösen, muß man (lokale) Umkehrfunktionen
A^-1 von A anwenden. A ist aber dreistellig, und jede (lokale)
Umkehrfunktion von A ist deshalb dreiwertig (Ist das richtig?).
...
Eine mehrwertige Funktion kann aber nicht elementar sein.
Dies war per Definition gesetzt. Was nicht verhindert, daß A^(-1) eine
sehr einfache Funktion sein kann.
Die Funktion A^(-1) hat drei (einwertige) Komponentenfunktionen A^(-1)_1,
A^(-1)_2, A^(-1)_3, ich sehe nicht, warum die nicht elementar sein können.
Für die einzelnen Funktionen haben wir doch folgende Zuordnungsvorschriften.
f1: x \mapsto f1(x); f1 ist eine Elementare Funktion.
f2: x \mapsto f2(x); f2 ist eine Elementare Funktion.
f3: c \mapsto f3(x); f3 ist eine Elementare Funktion.
A: (f1(x),f2(x),f3(x)) \mapsto A(f1(x),f2(x),f3(x)); A ist eine algebraische
Funktion, also eine Elementare Funktion
A^-1: A(f1(x),f2(x),f3(x)) \mapsto (f1(x),f2(x),f3(x)); A^-1 kann keine
Elementare Funktion sein, weil A^-1 mehrwertig ist.
Und jetzt die Komponentenfunktionen von A^-1:
A^-1_1: (f1(x),f2(x),f3(x)) \mapsto f1(x); kann eine Elementare Funktion
sein
A^-1_2: (f1(x),f2(x),f3(x)) \mapsto f2(x); kann eine Elementare Funktion
sein
A^-1_3: (f1(x),f2(x),f3(x)) \mapsto f3(x); kann eine Elementare Funktion
sein
Die Komponentenfunktionen von A^-1 können zwar Elementare Funktionen sein,
aber A^-1 kann keine Elementare Funktion sein.
Die Funktion A^-1 ist von der Stelligkeit und Wertigkeit her vergleichbar
mit der Funktion G in der Darstellung F(x) = A(G(x)). Die
Zuordnungsvorschrift von G ist: x \mapsto (f1(x),f2(x),f3(x)). Obwohl die
Funktion f1, f2 und f3 Elementare Funktionen sind, kann G keine Elementare
Funktion sein, weil G mehrwertig ist.

Diese Funktionen kann man ja mal als Verknüpfungen der jeweiligen Argumente
und Funktionswerte in einem Graphen darstellen/veranschaulichen. Ich nenne
solchen Graphen Kompositionsgraph/Verkettungsgraph, obwohl diese beiden
Begriffe bereits mehrere andere Bedeutungen haben. Ich hatte hier schonmal
so einen Graphen für die Funktionen in x \mapsto x+e^x gebracht - in der
Horizontalen geschrieben. Man kann den Graphen aber auch in der Vertikalen
schreiben.
Ist solch ein Graph vielleicht sogar ein Verband?

Zu Deinem Gleichungssystem später.
IV
2017-02-23 18:44:32 UTC
Permalink
Post by IV
Diese Funktionen kann man ja mal als Verknüpfungen der jeweiligen
Argumente und Funktionswerte in einem Graphen darstellen/veranschaulichen.
Verkettungsgraph für die Elementare Funktion F: x \mapsto F(x) = A(x,e^x) =
A(G(x) = x + e^x:
\mapsto x
x \mapsto x+e^x
\mapsto e^x
Darin kann man die Bezeichner für alle hierin irgendwie auftretenden
Funktionen und Umkehrfunktionen eintragen, und dazuschreiben, ob sie
elementar sind oder nicht-elementar.
Torn Rumero DeBrak
2017-02-23 19:27:53 UTC
Permalink
Post by IV
Post by IV
Diese Funktionen kann man ja mal als Verknüpfungen der jeweiligen
Argumente und Funktionswerte in einem Graphen
darstellen/veranschaulichen.
Kann man so wie unten gemact eben nicht.
Post by IV
Verkettungsgraph für die Elementare Funktion F: x \mapsto F(x) =
\mapsto x
x \mapsto x+e^x
\mapsto e^x
Was soll das heißen? Daß etwa x je nach Lust und Laune sich
aussucht, um auf x oder auf e^x abgebildet zu werden?

x wird auf ein TUPEL abgebildet, das wiederum auf die Summe
abgebildet wird. Da gibt es keine Verzweigung.
Post by IV
Darin kann man die Bezeichner für alle hierin irgendwie auftretenden
Funktionen und Umkehrfunktionen eintragen, und dazuschreiben, ob sie
elementar sind oder nicht-elementar.
Dunkel ist deiner Schrift Sinn.
Torn Rumero DeBrak
2017-02-23 19:22:25 UTC
Permalink
Post by IV
Post by IV
F: die gegebene bijektive Elementare Funktion, eine nicht-lineare
expln-Verkettung (= für F existiert keine lineare
expln-Verkettungsdarstellung)
A: eine algebraische Funktion
f1, f2, f3: jeweils eine lineare expln-Verkettung, alle drei algebraisch
unabhängig voneinander
gegeben: F(x) = A(f1(x),f2(x),f3(x))
y = F^-1(x)
A(f1(y),f2(y),f3(y)) = x
Um diese Gleichung nach y aufzulösen, muß man (lokale) Umkehrfunktionen
A^-1 von A anwenden. A ist aber dreistellig, und jede (lokale)
Umkehrfunktion von A ist deshalb dreiwertig (Ist das richtig?).
...
Eine mehrwertige Funktion kann aber nicht elementar sein.
Dies war per Definition gesetzt. Was nicht verhindert, daß A^(-1) eine
sehr einfache Funktion sein kann.
Die Funktion A^(-1) hat drei (einwertige) Komponentenfunktionen A^(-1)_1,
A^(-1)_2, A^(-1)_3, ich sehe nicht, warum die nicht elementar sein können.
Für die einzelnen Funktionen haben wir doch folgende
Zuordnungsvorschriften.
f1: x \mapsto f1(x); f1 ist eine Elementare Funktion.
f2: x \mapsto f2(x); f2 ist eine Elementare Funktion.
f3: c \mapsto f3(x); f3 ist eine Elementare Funktion.
A: (f1(x),f2(x),f3(x)) \mapsto A(f1(x),f2(x),f3(x));
Ja ist denn die ganze Diskussion an dir vorbeigelaufen
und liest du die Postings nicht??????


A: (f1(x),f2(x),f3(x)) \mapsto A(f1(x),f2(x),f3(x))
ist UNSINN, QUATSCH, FANTASIE, FALSCH!!

A ist eine Funktion von (x1, x2, x3) und nicht von
f1(x), f2(x) oder f3(x).
Du schreibst immer die Komposition der Funktionen

A: (x1, x2, x3) \mapsto A(x1, x2, x3) und der Funktion

f1 x f2 x f3: x \mapsto (f1(x), f2(x), f3(x))

dh. deine Schreibweise ist pure Faulheit (oder doch Dummheit?)
für

A o f1 x f2 x f3 : z \mapsto A(f1(z),f2(z),f3(z))

So gewinnst du keinen Blumentopf, wenn schon deine Schreibweise
ungenauer wohl nicht sein kann. Daraus kann keiner etwas sinnvolles
erarbeiten.
Loading...